NURS 122 practice questions

Réussis tes devoirs et examens dès maintenant avec Quizwiz!

The nurse is to give 750 mg PO, q8h x 10 days. The amount that is on hand is 0.5g/tablet. How many tablets should the nurse administer at each dose? Write your answer to the first decimal place. ____________ tablet(s)

1.5

steps of nociception

1: transduction 2:transmission 3: perception 4: modulation

A nurse must give 1 g of Keflex, PO, q 6 hr 3 days. The supply on hand is 500 mg/capsule. How many capsules should the nurse administer at each dose?

2

A 2-week-old infant can fixate on an object but cannot follow a light or bright toy. The nurse would: A) Consider this a normal finding. B) Assess the pupillary light reflex for possible blindness. C) Continue with the examination, and assess visual fields. D) Expect that a 2-week-old infant should be able to fixate and follow an object.

A

A 21-year-old patient has a head injury resulting from trauma and is unconscious. There are no other injuries. During the assessment what would the nurse expect to find when testing the patient's deep tendon reflexes? A) Reflexes will be normal. B) Reflexes cannot be elicited. C) All reflexes will be diminished but present. D) Some reflexes will be present, depending on the area of injury.

A

A 25-year-old woman in her fifth month of pregnancy has a blood pressure of 100/70 mm Hg. In reviewing her previous examination, the nurse notes that her blood pressure in her second month was 124/80 mm Hg. In evaluating this change, what does the nurse know to be true? A) This decline in blood pressure is the result of peripheral vasodilatation and is an expected change. B) Because of increased cardiac output, the blood pressure should be higher at this time. C) This change in blood pressure is not an expected finding because it means a decrease in cardiac output. D) This decline in blood pressure means a decrease in circulating blood volume, which is dangerous for the fetus.

A

A 31-year-old patient tells the nurse that he has noticed a progressive loss in his hearing. He says that it does seem to help when people speak louder or if he turns up the volume of a television or radio. The most likely cause of his hearing loss is: A) Otosclerosis. B) Presbycusis. C) Trauma to the bones. D) Frequent ear infections.

A

A 40-year-old patient who has just finished chemotherapy for breast cancer tells the nurse that she is concerned about her mouth. During the assessment the nurse finds areas of buccal mucosa that are raw and red with some bleeding, as well as other areas that have a white,cheesy coating. What do these findings indicate? A) Candidiasis B) Leukoplakia C) Koplik spots D) Aphthous ulcers

A

A 45-year-old farmer comes in for a skin evaluation and complains of hair loss on his head. He has noticed that his hair seems to be breaking off in patches and that he has some scaling on his head. The nurse would begin the examination suspecting: A) Tinea capitis. B) Folliculitis. C) Toxic alopecia. D) Seborrheic dermatitis.

A

A 59-year-old patient has a herniated intervertebral disc. Which of the following findings should the nurse expect to see on physical assessment of this individual? A) Hyporeflexia B) Increased muscle tone c) Positive Babinski sign D) Presence of pathologic reflexes

A

A 6-month-old infant has been brought to the well-child clinic for a check-up. She is currently sleeping. What should the nurse do first when beginning the examination? A) Auscultate the lungs and heart while the infant is still sleeping. B) Examine the infants hips, because this procedure is uncomfortable. C) Begin with the assessment of the eye, and continue with the remainder of the examination in a head-to-toe approach. D) Wake the infant before beginning any portion of the examination to obtain the most accurate assessment of body systems

A

A 67-year-old patient states that he recently began to have pain in his left calf when climbing the 10 stairs to his apartment. This pain is relieved by sitting for about 2 minutes; then he is able to resume his activities. The nurse interprets that this patient is most likely experiencing: A) Claudication B) Sore muscles. C) Muscle cramps. D) Venous insufficiency.

A

A 68-year-old woman is in the eye clinic for a checkup. She tells the nurse that she has been having trouble reading the paper, sewing, and even seeing the faces of her grandchildren. On examination, the nurse notes that she has some loss of central vision but her peripheral vision is normal. These findings suggest that she may have: A) Macular degeneration. B) Vision that is normal for someone her age. C) The beginning stages of cataract formation. D) Increased intraocular pressure or glaucoma.

A

A 70-year-old patient is being seen in the clinic for severe exacerbation of his heart failure. Which of these findings is the nurse most likely to observe in this patient? A) Shortness of breath, orthopnea, paroxysmal nocturnal dyspnea, and ankle edema B) Rasping cough, thick mucoid sputum, wheezing, and bronchitis C) Productive cough, dyspnea, weight loss, anorexia, and tuberculosis D) Fever, dry nonproductive cough, and diminished breath sounds

A

A 75-year-old woman is at the office for a preoperative interview. The nurse is aware that the interview may take longer than interviews with younger persons. What is the reason for this? A) An aged person has a longer story to tell. B) An aged person is usually lonely and likes to have someone with whom to talk. C) Aged persons lose much of their mental abilities and require longer time to complete an interview. D) As a person ages, he or she is unable to hear; thus the interviewer usually needs to repeat much of what is said

A

A 75-year-old woman who has a history of diabetes and peripheral vascular disease has been trying to remove a corn on the bottom of her foot with a pair of scissors. The nurse will encourage her to stop trying to remove the corn with scissors because: A) The woman could be at increased risk for infection and lesions because of her chronic disease. B) With her diabetes, she has increased circulation to her foot, and it could cause severe bleeding. C) She is 75 years old and is unable to see; consequently, she places herself at greater risk for self-injury with the scissors. D) With her peripheral vascular disease, her range of motion is limited and she may not be able to reach the corn safely.

A

A 76 year old patient is admitted to the hospital with a diagnosis of bacterial pneumonia. while obtaining the patient's health history, the nurse learns the patient has osteorthritis, follows a vegetarian diet, and is very concerned with cleanliness. Which patient information would most likely be a predisposing factor for the diagnosis of pneumonia? A) age B) osteoarthritis C) vegetarian diet D) daily bathing

A

A female nurse is interviewing a man who has recently immigrated. During the course of the interview, he leans forward and then finally moves his chair close enough that his knees are nearly touching the nurses knees. The nurse begins to feel uncomfortable with his proximity. Which statement most closely reflects what the nurse should do next? A) The nurse should try to relax; these behaviors are culturally appropriate for this person. B) The nurse should discreetly move his or her chair back until the distance is more comfortable, and then continue with the interview. C) These behaviors are indicative of sexual aggression, and the nurse should confront this person about his behaviors. D) The nurse should laugh but tell him that he or she is uncomfortable with his proximity and ask him to move away.

A

A female patient does not speak English well, and the nurse needs to choose an interpreter. Which of the following would be the most appropriate choice? A)Trained interpreter B) Male family member C) Female family member D) Volunteer college student from the foreign language studies department

A

A man is at the clinic for a physical examination. He states that he is very anxious about the physical examination. What steps can the nurse take to make him more comfortable? A) Appear unhurried and confident when examining him. B) Stay in the room when he undresses in case he needs assistance. C) Ask him to change into an examining gown and to take off his undergarments. D) Defer measuring vital signs until the end of the examination, which allows him time to become comfortable. A

A

A nurse is educating a patient about osteoarthritis and how to best manage the condition at home. which of the following statments made by the patient indicates that more teaching is necessary? A) i can sit at my computer and perform my data entry job like i usually do B) im going to work on losing weight C) i am going to quit smoking because it will help with my disease D) i play tennis, but am going to swtich to walking instead

A

A nurse notices that a patient has ascites, which indicates the presence of: A) Fluid B) Feces C) Flatus. D) Fibroid tumors.

A

A patient has been diagnosed with a ganglion cyst over the dorsum of his left wrist. He asks the nurse, "What is this thing?" The nurse's best answer would be, "It is: A) a common benign tumor." B) a tumor that will have to be watched because it may turn malignant." C) caused by chronic repetitive motion injury." D) a skin infection that will need to be drained."

A

A patient has hard, nonpitting edema of the left lower leg and ankle. The right leg has no edema. Based on these findings, the nurse recalls that: A) Nonpitting, hard edema occurs with lymphatic obstruction. B) Alterations in arterial function will cause edema. C) Phlebitis of a superficial vein will cause bilateral edema. D) Long-standing arterial obstruction will cause pitting edema.

A

A patient states, "I can hear a crunching or grating sound when I kneel." She also states that "it is very difficult to get out of bed in the morning because of stiffness and pain in my joints." The nurse should assess for signs of what problem? A) Crepitation B) A bone spur C) A loose tendon D) Fluid in the knee joint

A

A patient tells the nurse that he has noticed that one of his moles has started to burn and bleed. When assessing his skin, the nurse would pay special attention to the danger signs for pigmented lesions and would be concerned with which additional finding? A) Color variation B) Border regularity C) Symmetry of lesions D) Diameter of less than 6 mm

A

A patient tells the nurse that she is having a hard time bringing her hand to her mouth when she eats or tries to brush her teeth. The nurse knows that for her to move her hand to her mouth, she must perform which movement? A) flexion. B) abduction. C) adduction. D) extension.

A

A patient tells the nurse that she is having a hard time bringing her hand to her mouth when she tries to eat or brush her teeth. The nurse knows that for her to move her hand to her mouth, she must perform which movement? A) flexion b) abduction C) adduction D) extension

A

A patient who had a total hip replacement two days ago has developed an infection with a fever and profuse diaphoresis. the nurse establishes a goal to reduce the total fluid deficit. which outcome is the most appropriate> A) the patient will drink 2000mL of fluid per day B) the patient will verbalize how to manage their incision at home C) the patient's bed linens are changed to prevent moisture associated skin breakdown D) the patient's skin will remain cool throughout hospitalization

A

A patient will be ready to be discharged from the hospital soon, and the patient's family membersare concerned about whether the patient is able to walk outside alone safely. The nurse will perform which test to assess this? A) The Get Up and Go Test B) The Performance Activities of Daily Living C) The Physical Performance Test D) Tinetti Gait and Balance Evaluation

A

A patient with an indwelling catheter reports a need to void. What is the priority intervention for the nurse to perform? A) Check to see if the catheter is patent. B) Reassure the patient that it is not possible to void while catheterized. C) Catheterize the patient again with a larger-gauge catheter. D) Notify the primary care provider (PCP).

A

A patient's vision is recorded as 20/80 in each eye. The nurse interprets this finding to mean that the patient: A) Has poor vision. B) Has acute vision. C) Has normal vision. D) Is presbyopic.

A

A patients laboratory data reveal an elevated thyroxine (T4) level. The nurse would proceed with an examination of the _____ gland. A) Thyroid B) Parotid C) Adrenal D) Parathyroid

A

A patients uvula raises midline when she says ahh, and she has a positive gag reflex. The nurse has just tested which cranial nerves? A) IX and X B) IX and XII C) X and XII D) XI and XII

A

A wife of a 65-year old man tells the nurse that she is concerned because she has noticed a change in her husband's personality and ability to understand. he also cries very easily and becomes angry. what part of the cerebral lobe is responsible for these behaviors? A) frontal B) parietal C) occipital D) temporal

A

A woman who is 8 months pregnant comments that she has noticed a change in posture and is having lower back pain. The nurse tells her that during pregnancy women have a posture shift to compensate for the enlarging fetus. This shift in posture is known as: A) lordosis. B) scoliosis. C) ankylosis. D) kyphosis.

A

After the health history has been obtained and before beginning the physical examination, the nurse should first ask the patient to: A) Empty the bladder. B) Completely disrobe. C) Lie on the examination table. D) Walk around the room.

A

An 85-year-old man has been hospitalized after a fall at home, and his 86-year-old wife is at his bedside. She tells the nurse that she is his primary caregiver. The nurse should assess the caregiver for signs of possible caregiver burnout, such as: A) Depression. B) Weight gain. C) Hypertension. D) Social phobias

A

An adult patient with a history of allergies comes to the clinic complaining breathing when working in his yard. The assessment findings include tachypnea, the use of accessory neck of wheezing and difficulty in muscles, prolonged expiration, intercostal retractions, decreased breath sounds, and expiratory wheezes. The nurse interprets that these assessment findings are consistent with: A) Asthma B) Atelectasis. C) Lobar pneumonia. D) Heart failure.

A

An elderly patient has been ill with the flu, experiencing headache, fever, and chills. After 3 days, she developed a cough productive of yellow sputum. The nurse auscultates her lungs and hears diffuse crackles. How would the nurse best interpret these assessment findings? A) the patient is likely experiencing a secondary bacterial pneumonia B) the assessment findings are consistent with influenza; no further follow up is needed C) the patient is getting dehydrated and needs IV fluids D) the patient has not been taking her decongestants and bronchodilators

A

An older adult verbalizes to the nurse that he/she is having pain in his/her left wrist. Which of the following would be the most appropriate response by the nurse? A) "Tell me more about the pain." B) "Occasional pain is a normal part of getting older." C) "It's nothing to worry about." D) "We try to avoid prescribing pain medicine to older adults because they often become addicted."

A

As the nurse enters a patients room, the nurse finds her crying. The patient states that she has just found out that the lump in her breast is cancer and says, Im so afraid of, um, you know. The nurses most therapeutic response would be to say in a gentle manner: A) Youre afraid you might lose your breast? B) No, Im not sure what you are talking about. C) Ill wait here until you get yourself under control, and then we can talk. D) I can see that you are very upset. Perhaps we should discuss this later.

A

Before administering the evening dose of a prescribed medication, the nurse on the evening shift finds an unlabeled, filled syringe in the patient's medication drawer. What should the nurse in charge do? A) Discard the syringe to avoid a medication error. B) Obtain a label for the syringe from the pharmacy. C) Use the syringe because it looks like it contains the same medication the nurse was prepared to give. D) Call the day nurse to verify the contents of the syringe.

A

During a cardiovascular assessment, the nurse knows that a thrill is: A) Vibration that is palpable. B) Palpated in the right epigastric area. C) Associated with ventricular hypertrophy. D) Murmur auscultated at the third intercostal space.

A

During a checkup, a 22-year-old woman tells the nurse that she uses an over-the-counter nasal spray because of her allergies. She also states that it does not work as well as it used to when she first started using it. Which is the best response by the nurse? A) "You should never use over-the-counter nasal sprays because of the risk foraddiction." B) "You should try switching to another brand of medication to prevent this problem." C) "Continuing to use this spray is important to keep your allergies under control." D) "Frequent use of these nasal medications irritates the lining of the nose and may cause rebound swelling."

A

During a follow-up visit, the nurse discovers that a patient has not been taking his insulin on a regular basis. The nurse asks, Why havent you taken your insulin? Which statement is an appropriate evaluation of this question? A) This question may place the patient on the defensive. B) This question is an innocent search for information. C) Discussing his behavior with his wife would have been better. D) A direct question is the best way to discover the reasons for his behavior.

A

During a well-baby checkup, a mother is concerned because her 2-month-old infant cannot hold her head up when she is pulled to a sitting position. Which response by the nurse is appropriate? A) Head control is usually achieved by 4 months of age. B) You shouldnt be trying to pull your baby up like that until she is older. C) Head control should be achieved by this time. D) This inability indicates possible nerve damage to the neck muscles.

A

During an assessment of a 20-year-old patient with a 3-day history of nausea and vomiting,the nurse notices dry mucosa and deep vertical fissures in the tongue. What do these findingsindicate? A) Dehydration B) A normal oral assessment C) Irritation from gastric juices D) Side effects from nausea medication

A

During an assessment of a 62-year-old man the nurse notices the patient has a stooped posture, shuffling walk with short steps, flat facial expression, and pill-rolling finger movements. These findings would be consistent with: A) Parkinsonism. B) Cerebral palsy. C) Cerebellar ataxia. D) Muscular dystrophy.

A

During an assessment of a hospitalized patient, the nurse pinches a fold of skin under the clavicle or on the forearm to test: A) Mobility and turgor. B) The patient's response to pain. C) The percentage of the patient's fat-to-muscle ratio. D) The presence of edema.

A

During an assessment of a newborn infant, the nurse recalls that pyloric stenosis would be exhibited by: A) Projectile vomiting. B) Hypoactive bowel activity. C) Palpable olive-sized mass in the right lower quadrant. D) Pronounced peristaltic waves crossing from right to left.

A

During an examination of a 3-year-old child, the nurse notices a bruit over the left temporal area. The nurse should: A) Continue the examination because a bruit is a normal finding for this age. B) Check for the bruit again in 1 hour. C) Notify the parents that a bruit has been detected in their child. D) Stop the examination, and notify the physician.

A

During an examination, the nurse finds that a patient has excessive dryness of the skin. The best term to describe this condition is: A) Xerosis. B) Pruritus. C) Alopecia. D) Seborrhea.

A

During an examination, the nurse knows that the best way to palpate the lymph nodes in the neck is described by which statement? A) Using gentle pressure, palpate with both hands to compare the two sides. B) Using strong pressure, palpate with both hands to compare the two sides. C) Gently pinch each node between ones thumb and forefinger, and then move down the neck muscle. D) Using the index and middle fingers, gently palpate by applying pressure in a rotating pattern.

A

During an examination, the nurse notices that the patient stumbles a little while walking, and, when she sits down, she holds on to the sides of the chair. The patient states, "It feels like the room is spinning!" The nurse notices that the patient is experiencing: A) Objective vertigo. B) Subjective vertigo. C) Tinnitus. D) Dizziness.

A

During an examination, the patient tells the nurse that she sometimes feels as if objects are spinning around her. The nurse would document that she occasionally experiences: A) Vertigo. B) Tinnitus. C) Syncope. D) Dizziness.

A

During an inspection of a patients face, the nurse notices that the facial features are symmetric. This finding indicates which cranial nerve is intact? A) VII B) IX C) XI D) XII

A

During an oral examination of a 4-year-old American-Indian child, the nurse notices that heruvula is partially split. Which of these statements is accurate? A) A bifid uvula may occur in some American-Indian groups. B) This condition is a cleft palate and is common in American Indians. C) A bifid uvula is torus palatinus, which frequently occurs in American Indians. D) This condition is due to an injury and should be reported to the authorities.

A

During auscultation of breath sounds, the nurse should correctly use the stethoscope in which of the following ways? A) Listening to at least one full respiration in each location B) Listening as the patient inhales and then going to the next site during exhalation C) Instructing the patient breathe in and out rapidly while listening to the breath sounds D) If the patient is modest, listening to sounds over his or her clothing or hospital gown

A

During auscultation of the lungs, the nurse expects decreased breath sounds to be heard in which situation? A) When the bronchial tree is obstructed B) When adventitious sounds are present C) In conjunction with whispered pectoriloquy D) In conditions of consolidation, such as pneumonia

A

During reporting, the student nurse hears that a patient has hepatomegaly and recognizes that this term refers to: A) Enlarged liver .B) Enlarged spleen. C) Distended bowel. D) Excessive diarrhea

A

During the assessment of an 80-year-old patient, the nurse notices that his hands show tremors when he reaches for something and his head is always nodding. There is no associated rigidity with movement. Which of these statements is most accurate? A) These findings are normal, resulting from aging B) .These findings could be related to hyperthyroidism C) .These findings are the result of Parkinson disease. D) This patient should be evaluated for a cerebellar lesion.

A

During the history, a patient tells the nurse that "it feels like the room is spinning around me." The nurse would document this as: A) Vertigo. B) Syncope. C) Dizziness. D) Seizure activity.

A

How should the nurse document mild, slight pitting edema present at the ankles of a pregnant patient? A) 1+/0-4+ B) 3+/0-4+ C) 4+/0-4+ D) Brawny edema

A

In a person with an upper motor neuron lesion such as a cerebrovascular accident, which of these physical assessment findings should the nurse expect to see? A) Hyperreflexia B) Fasciculations C) Loss of muscle tone and flaccidity D) Atrophy and wasting of the muscles

A

In an interview, the nurse may find it necessary to take notes to aid his or her memory later. Which statement is true regarding note-taking? A) Note-taking may impede the nurses observation of the patients nonverbal behaviors. B) Note-taking allows the patient to continue at his or her own pace as the nurse records what is said. C) Note-taking allows the nurse to shift attention away from the patient, resulting in an increased comfort level. D) Note-taking allows the nurse to break eye contact with the patient, which may increase his or her level of comfort.

A

In previewing the medical record of a patient, you find documentation of pulsus alternans. On the basis of this fact, what do you expect when you assess the patient? A) Pulse with a regular rhythm, but the force of the pulse varies with alternating beats. B) Pulse with weaker amplitude with respiratory inspiration and stronger amplitude with expiration. C) Deficiency of oxygenated arterial blood to a body part. D) Pulse with coupled rhythm; every other beat is premature.

A

In the assessment of a 1-month-old infant, the nurse notices a lack of response to noise or stimulation. The mother reports that in the last week he has been sleeping all the time, and when he is awake all he does is cry. The nurse hears that the infant's cries are very high pitched and shrill. What should be the nurse's appropriate response to these findings? A) Refer the infant for further testing. B) Talk with the mother about eating habits. C) Do nothing; these are expected findings for an infant this age. D) Tell the mother to bring the baby back in 1 week for a recheck.

A

Just before going home, a new mother asks the nurse about the infants umbilical cord. Which of these statements is correct? A) It should fall off in 10 to 14 days. B) It will soften before it falls off. C) It contains two veins and one artery. D) Skin will cover the area within 1 week.

A

Mr. Kimbel is a 59-year-old patient who comes to the clinic for a routine health assessment at the request of his son. On examination, you note a positive profile sign. This indicates: A) early clubbing. B) the patency of the radial and ulnar arteries. C) the presence of thrombophlebitis. D) the degree of pedal edema.

A

Mr. Worrigan is a 67-year-old patient who comes with his son to the ambulatory health centre. On examination of Mr. Worrigan, you note a pulsus alternans. This is associated with: A) heart failure. B) pulmonary embolisms. C) hyperkinetic states. D) decreased cardiac output.

A

Neuropathic pain implies an abnormal A) processing of the pain message. B) degree of pain interpretation. C) modulation of pain signals. D) transmission of pain signals.

A

Nursing interventions for the patient who suffers from stress incontinence include A) kegel exercises. B) surgical interventions. C) bowel retraining. D) intermittent catheterization.

A

Of the 33 vertebrae in the spinal column, there are: A) 5 lumbar. B) 5 thoracic. C) 7 sacral. D) 12 cervical.

A

Of what does the patient believe the amulet is protective? A) the evil eye B) being kidnapped C) exposure to bacterial infections D) an unexpected fall.

A

The ankle joint is the articulation of the tibia, the fibula, and the: A) talus. B) cuboid. C) calcaneus. D) cuneiform bones.

A

The blood is returned to the heart through the veins by means of: A) All of the options. B) breathing. C) unidirectional valves. D) walking.

A

The functional units of the musculoskeletal system are the: A) joints. B) bones. C) muscles. D) tendons.

A

The home health nurse is called for a consult on a patient with memory problems who is having difficulty remembering to take multiple medications prescribed to be taken throughout the day. What can the nurse do to help the patient remember to take the medications as prescribed? A) Arrange for the medications to be put in a pill organizer by week. B) Make a chart showing times when medications should be taken. C) Ask a family member to come over each day to administer medications. D) Ask the patient to set an alarm clock for when medications are due.

A

The mother of a 10-month-old infant tells the nurse that she has noticed that her son becomes blue when he is crying and that the frequency of this is increasing. He is also not crawling yet. During the examination the nurse palpates a thrill at the left lower sternal border and auscultates a loud systolic murmur in the same area. What would be the most likely cause of these findings? A) Tetralogy of Fallot B) Atrial septal defect C) Patent ductus arteriosus D) Ventricular septal defect

A

The nurse assesses the hearing of a 7-month-old by clapping hands. What is the expected response? The infant: A) Turns his or her head to localize the sound. B) Shows no obvious response to the noise. C) Shows a startle and acoustic blink reflex. D) Stops any movement, and appears to listen for the sound.

A

The nurse has discovered decreased skin turgor in a patient and knows that this is an expected finding in which of these conditions? A) Severe obesity B) Childhood growth spurts C) Severe dehydration D) Connective tissue disorders such as scleroderma

A

The nurse is about to administer a new medication to a patient. Which action best demonstrates awareness of safe, proficient nursing practice? A) Identify the patient by comparing her name and birth date to the medication administration record (MAR). B) Determine whether the medication and dose are appropriate for the patient. C) Make sure the medication is in the medication cart. D) Check the accuracy of the dose with another nurse.

A

The nurse is assessing a patient who may have hearing loss. Which of these statements is true concerning air conduction? A) Air conduction is the normal pathway for hearing. B) Vibrations of the bones in the skull cause air conduction. C) Amplitude of sound determines the pitch that is heard. D) Loss of air conduction is called a conductive hearing loss.

A

The nurse is assessing an older adult's advanced activities of daily living, which would include: A) Recreational activities. B) Meal preparation. C) Balancing the checkbook. D) Self-grooming activities.

A

The nurse is assessing the apical pulse of a 3-month-old infant and finds that the heart rate is 135 beats per minute. The nurse interprets this result as: A) Normal for this age. B) Lower than expected. C) Higher than expected, probably as a result of crying. D) Higher than expected, reflecting persistent tachycardia.

A

The nurse is assessing the vital signs of a 3-year-old patient who appears to have an irregular respiratory pattern. How should the nurse assess this childs respirations? A) Respirations should be counted for 1 full minute, noticing rate and rhythm. B) Childs pulse and respirations should be simultaneously checked for 30 seconds. C) Childs respirations should be checked for a minimum of 5 minutes to identify any variations in his or her respiratory pattern. D) Patients respirations should be counted for 15 seconds and then multiplied by 4 to obtain the number of respirations per minute.

A

The nurse is auscultating the lungs of a patient who had been sleeping and notices short, popping, crackling sounds that stop after a few breaths. The nurse recognizes that these breath sounds are: A) Atelectatic crackles that do not have a pathologic cause. B) Fine crackles and may be a sign of pneumonia. C) Vesicular breath sounds. D) Fine wheezes.

A

The nurse is checking the range of motion in a patient's knee and knows that the knee is capable of which movement(s)? A) Flexion and extension B) Supination and pronation C) Circumduction D) Inversion and eversion

A

The nurse is examining a 6-month-old infant and places the infant's feet flat on the table and flexes his knees up. The nurse notes that the right knee is significantly lower than the left. Which of these statements is true of this finding? A) This is a positive Allis sign and suggests hip dislocation. B) The infant probably has a dislocated patella on the right. C) This is a normal finding for the Allis test for an infant of this age. D) The infant should return to the clinic in 2 weeks to see if this has changed.

A

The nurse is examining a patient who tells the nurse, "I sure sweat a lot, especially on my face and feet but it doesn't have an odor." The nurse knows that this could be related to: A) Eccrine glands. B) Apocrine glands. C) Disorder of the stratum corneum. D) Disorder of the stratum germinativum.

A

The nurse is examining a patient's retina with an ophthalmoscope. Which finding is considered normal? A) Optic disc that is a yellow-orange color B) Optic disc margins that are blurred around the edges C) Presence of pigmented crescents in the macular area D) Presence of the macula located on the nasal side of the retina

A

The nurse is inserting an indwelling catheter into a male patient. While initially passing the catheter through the urethra, resistance is met. What action should the nurse take next? A) Notify the primary care provider to place a coudé catheter. B) Straighten the penis and attempt to progress the catheter again. C) Remove the catheter and insert one with a smaller lumen. D) Inflate the balloon and wait for urine passage.

A

The nurse is interviewing a male patient who has a hearing impairment. What techniques would be most beneficial in communicating with this patient? A) Determine the communication method he prefers. B) Avoid using facial and hand gestures because most hearing-impaired people find this degrading. C) Request a sign language interpreter before meeting with him to help facilitate the communication. D) Speak loudly and with exaggerated facial movement when talking with him because doing so will help him lip read.

A

The nurse is listening to the breath sounds of a patient with severe asthma. Air passing through narrowed bronchioles would produce which of these adventitious sounds? A) Wheezes B) Bronchial sounds C) Bronchophony D) Whispered pectoriloquy

A

The nurse is observing the auscultation technique of another nurse. The correct method to use when progressing from one auscultatory site on the thorax to another is ___________comparison. A) Side-to-side B) Top-to-bottom C) Posterior-to-anterior D) Interspace-by-interspace

A

The nurse is percussing over the lungs of a patient with pneumonia. The nurse knows that percussion over an area of atelectasis in the lungs will reveal: A) Dullness B) Tympany. C) Resonance. D) Hyperresonance.

A

The nurse is percussing the seventh right intercostal space at the midclavicular line over the liver. Which sound should the nurse expect to hear? A) Dullness B) Тympany C) Resonance D) Hyperresonance

A

The nurse is performing a general survey. Which action is a component of the general survey? A) Observing the patients body stature and nutritional status B) Interpreting the subjective information the patient has reported C) Measuring the patients temperature, pulse, respirations, and blood pressure D) Observing specific body systems while performing the physical assessment

A

The nurse is performing a health interview on a patient who has a language barrier, and no interpreter is available. Which is the best example of an appropriate question for the nurse to ask in this situation? A) Do you take medicine? B) Do you sterilize the bottles? C) Do you have nausea and vomiting? D) You have been taking your medicine, havent you?

A

The nurse is performing an assessment on a 7-year-old child who has symptoms of chronic watery eyes, sneezing, and clear nasal drainage. The nurse notices the presence of a transverse line across the bridge of the nose, dark blue shadows below the eyes, and a double crease on the lower eyelids. These findings are characteristic of: A) Allergies. B) Sinus infection. C) Nasal congestion. D) Upper respiratory infection.

A

The nurse is performing an ear examination of an 80-year-old patient. Which of these findings would be considered normal? A) High-tone frequency loss B) Increased elasticity of the pinna C) Thin, translucent membrane D) Shiny, pink tympanic membrane

A

The nurse is performing an external eye examination. Which statement regarding the outer layer of the eye is true? A) The outer layer of the eye is very sensitive to touch. B) The outer layer of the eye is darkly pigmented to prevent light from reflecting internally. C) The trigeminal nerve (CN V) and the trochlear nerve (CN IV) are stimulated when the outer surface of the eye is stimulated. D) The visual receptive layer of the eye in which light waves are changed into nerve impulses is located in the outer layer of the eye.

A

The nurse is preparing a plan of care for a patient. What is the most appropriate goal for a patient related to medications? A) The patient will administer all medications correctly by discharge. B) The patient will be taught common side effects of prescribed medications. C) The patient will have a good understanding of prescribed medications. D) The patient will have all medications administered by staff as prescribed.

A

The nurse is preparing for a class on risk factors for hypertension and reviews recent statistics. Which racial group has the highest prevalence of hypertension in the world? A) Blacks B) Whites C) American Indians D) Hispanics

A

The nurse is preparing to do an otoscopic examination on a 2-year-old child. Which one of these reflects the correct procedure? A) Pulling the pinna down B) Pulling the pinna up and back C) Slightly tilting the child's head toward the examiner D) Instructing the child to touch his chin to his chest

A

The nurse is preparing to perform a functional assessment of an older patient and knows that a good approach would be to: A) Observe the patient's ability to perform the tasks. B) Ask the patient's wife how he does when performing tasks. C) Review the medical record for information on the patient's abilities. D) Ask the patient's physician for information on the patient's abilities.

A

The nurse is preparing to use a stethoscope for auscultation. Which statement is true regarding the diaphragm of the stethoscope? The diaphragm: A) Is used to listen for high-pitched sounds. B) Is used to listen for low-pitched sounds. C) Should be lightly held against the persons skin to block out low-pitched sounds. D) Should be lightly held against the persons skin to listen for extra heart sounds and murmurs.

A

The nurse is reviewing a patient's medical record and notes that he is in a coma. Using the Glasgow Coma Scale, which number indicates that the patient is in a coma? A) 6 B) 12 C) 15 D) 24

A

The nurse is reviewing percussion techniques with a newly graduated nurse. Which technique, if used by the new nurse, indicates that more review is needed? A) Percussing once over each area B) Quickly lifting the striking finger after each stroke C) Striking with the fingertip, not the finger pad D) Using the wrist to make the strikes, not the arm

A

The nurse is reviewing statistics for lactose intolerance. In the United States, the incidence of lactose intolerance is higher in adults of which ethnic group? A) Blacks B) Hispanics C) Whites D) Asians

A

The nurse is reviewing the principles of nociception. During which phase of nociception does the conscious awareness of a painful sensation occur? A)Perception B) Modulation C) Transduction D) Transmission

A

The nurse is reviewing venous blood flow patterns. Which of these statements best describes the mechanism(s) by which venous blood returns to the heart? A) Intraluminal valves ensure unidirectional flow toward the heart. B) Contracting skeletal muscles milk blood distally toward the veins. C) High-pressure system of the heart helps facilitate venous return. D) Increased thoracic pressure and decreased abdominal pressure facilitate venous return to the heart.

A

The nurse is teaching a class on osteoporosis prevention to a group of postmenopausal woman. A participant shows that she needs more instruction when she states, "I will: A) start swimming to increase my weight-bearing exercise." B) try to stop smoking as soon as possible." C) check with my doctor about taking calcium supplements." D) get a bone-density test soon."

A

The nurse is testing a patient's visual accommodation, which refers to which action? A) Pupillary constriction when looking at a near object B) Pupillary dilation when looking at a far object C) Changes in peripheral vision in response to light D) Involuntary blinking in the presence of bright light

A

The nurse is testing the deep tendon reflexes of a 30-year-old woman who is in the clinic for an annual physical examination. When striking the Achilles and quadriceps, the nurse is unable to elicit a reflex. The nurse's next response should be to: A) Ask the patient to lock her fingers and pull. B) Complete the examination, and then test these reflexes again. C) Refer the patient to a specialist for further testing. D) Document these reflexes as 0 on a scale of 0 to 4+.

A

The nurse is using an otoscope to assess the nasal cavity. Which of these techniques is correct? A) Avoiding touching the nasal septum with the speculum B) Inserting the speculum at least 3 cm into the vestibule C) Gently displacing the nose to the side that is being examined D) Keeping the speculum tip medial to avoid touching the floor of the nares

A

The nurse just noted from a patient's medical record that the patient has a lesion that is confluent in nature. On examination, the nurse would expect to find A) Lesions that run together. B) Annular lesions that have grown together. C) Lesions arranged in a line along a nerve route. D) Lesions that are grouped or clustered together.

A

The nurse knows that determining whether a person is oriented to his or her surroundings will test the functioning of which of these structures? A) Cerebrum B) Cerebellum C) CNs D)Medulla oblongata

A

The nurse knows that one advantage of the tympanic membrane thermometer (TMT) is that: A) Rapid measurement is useful for uncooperative younger children. B) Using the TMT is the most accurate method for measuring body temperature in newborn infants C) Measuring temperature using the TMT is inexpensive. D) Studies strongly support the use of the TMT in children under the age 6 years.

A

The nurse makes which adjustment in the physical environment to promote the success of an interview? A) Reduces noise by turning off televisions and radios B) Reduces the distance between the interviewer and the patient to 2 feet or less C) Provides a dim light that makes the room cozy and helps the patient relax D) Arranges seating across a desk or table to allow the patient some personal space

A

The nurse needs to pull the portion of the ear that consists of movable cartilage and skin down and back when administering eardrops. This portion of the ear is called the: A) Auricle. B) Concha. C) Outer meatus. D) Mastoid process.

A

The nurse should use which location for eliciting deep tendon reflexes? A) Achilles B) Femoral C) Scapular D) Abdominal

A

The nurse should use which test to check for large amounts of fluid around the patella? A) Ballottement B) Tinel sign C) Phalen's test D) McMurray's tes

A

The nurse suspects that a patient has hyperthyroidism, and the laboratory data indicate that the patients T4 and T3 hormone levels are elevated. Which of these findings would the nurse most likely find on examination? A)Tachycardia B) Constipation C) Rapid dyspnea D) Atrophied nodular thyroid gland

A

The nurse will use which technique of assessment to determine the presence of crepitus, swelling, and pulsations? A)Palpation B) Inspection C) Percussion D) Auscultation

A

The primary muscles of respiration include the: A) Diaphragm and intercostals. B) Sternomastoids and scaleni. C) Trapezii and rectus abdominis. D) External obliques and pectoralis major.

A

The sac that surrounds and protects the heart is called the: A)Pericardium. B) Myocardium. C) Endocardium. D) Pleural space.

A

The wife of a 65-year-old man tells the nurse that she is concerned because she has noticed a change in her husband's personality and ability to understand. He also cries and becomes angry very easily. The nurse recalls that the cerebral lobe responsible for these behaviors is the _____ lobe. A) Frontal B) Parietal C) Occipital D) Temporal

A

To assess a rectal temperature accurately in an adult, the nurse would: A) Use a lubricated blunt tip thermometer. B) Insert the thermometer 2 to 3 inches into the rectum. C) Leave the thermometer in place up to 8 minutes if the patient is febrile. D) Wait 2 to 3 minutes if the patient has recently smoked a cigarette.

A

To assess the head control of a 4-month-old infant, the nurse lifts the infant up in a prone position while supporting his chest. The nurse looks for what normal response? A) Raises the head, and arches the back. B) Extends the arms, and drops down the head. C) Flexes the knees and elbows with the back straight. D) Holds the head at 45 degrees, and keeps the back straight

A

To test for gross motor skill and coordination of a 6-year-old child, which of these techniques would be appropriate? A) Hop on one foot. B) Stand on his head. C) Touch his finger to his nose. D) Make "funny" faces at the nurse.

A

What should the nurse assess before entering the patient's room on morning rounds? A) Posted conditions, such as isolation precautions B) The patient's input and output chart from the previous shift C) The patient's general appearance D) The presence of any visitors in the room

A

What type of pain is short and self-limiting and dissipates after the injury heals? A) acute B) persistant C) chronic D) breakthrough

A

When assessing a patient's lungs, what should the nurse remember about the left lung? A) it consists of two lobes B) it is divided by a horizontal fissure C) it primarily consists of an upper lobe on the posterior chest D) is shorter than the right lung because of the underlying stomach

A

When assessing a patients lungs, the nurse recalls that the left lung: A) Consists of two lobes. B) Is divided by the horizontal fissure. C) Primarily consists of an upper lobe on the posterior chest D) Is shorter than the right lung because of underlying stomach.

A

When assessing a patients pulse, the nurse should also notice which of these characteristics? A) Force B) Pallor C) Capillary refill time D) Timing in the cardiac cycle

A

When assessing tactile fremitus, the nurse recalls that it is normal to feel tactile fremitus most intensely over which location? A) Between the scapulae B) Third intercostal space, MCL C) Fifth intercostal space, midaxillary line (MAL) D) Over the lower lobes, posterior side

A

When evaluating the temperature of older adults, the nurse should remember which aspect about an older adults body temperature? A) The body temperature of the older adult is lower than that of a younger adult. B) An older adults body temperature is approximately the same as that of a young child. C) Body temperature depends on the type of thermometer used. D) In the older adult, the body temperature varies widely because of less effective heat control mechanisms.

A

When examining a patients CN function, the nurse remembers that the muscles in the neck that are innervated by CN XI are the: A) Sternomastoid and trapezius. B) Spinal accessory and omohyoid. C) Trapezius and sternomandibular. D) Sternomandibular and spinal accessory.

A

When examining children affected with Down syndrome (trisomy 21), the nurse looks for the possible presence of: A) Ear dysplasia. B) Long, thin neck. C) Protruding thin tongue. D) Narrow and raised nasal bridge.

A

When examining the eye, the nurse notices that the patient's eyelid margins approximate completely. The nurse recognizes that this assessment finding: A) Is expected. B) May indicate a problem with extraocular muscles. C) May result in problems with tearing. D) Indicates increased intraocular pressure.

A

When listening to heart sounds, the nurse knows that S1: A) coincides with the carotid artery pulse B) signals the end of diastole C) produced by closure of the aortic and pulmonic valves D) is loudest at the third intercostal space midaxillary line

A

When palpating the abdomen of a 20-year-old patient, the nurse notices the presence of tenderness in the left upper quadrant with deep palpation. Which of these structures is most likely to be involved? A) Spleen B) Sigmoid colon C) Appendix D) Gallbladder

A

When percussing over the liver of a patient, the nurse notices a dull sound. The nurse should: A) Consider this a normal finding. B) Palpate this area for an underlying mass. C) Reposition the hands, and attempt to percuss in this area again. D) Consider this finding as abnormal, and refer the patient for additional treatment.

A

When performing a musculoskeletal assessment, the nurse knows that the correct approach for the examination should be: A) proximal to distal. B) distal to proximal. C) posterior to anterior. D) anterior to posterior.

A

When performing an otoscopic examination of a 5-year-old child with a history of chronic ear infections, the nurse sees that his right tympanic membrane is amber-yellow in color and that air bubbles are visible behind the tympanic membrane. The child reports occasional hearing loss and a popping sound with swallowing. The preliminary analysis based on this information is that the child: A) Most likely has serous otitis media. B) Has an acute purulent otitis media. C) Has evidence of a resolving cholesteatoma. D) Is experiencing the early stages of perforation.

A

When performing the corneal light reflex assessment, the nurse notes that the light is reflected at 2 o'clock in each eye. The nurse should: A) Consider this a normal finding. B) Refer the individual for further evaluation. C) Document this finding as an asymmetric light reflex. D) Perform the confrontation test to validate the findings.

A

When preparing to perform a physical examination on an infant, the nurse should: A) Have the parent remove all clothing except the diaper on a boy. B) Instruct the parent to feed the infant immediately before the examination. C) Encourage the infant to suck on a pacifier during the abdominal examination. D) Ask the parent to leave the room briefly when assessing the infants vital signs.

A

When providing his health history, Mr. Meier sates that his mother had lymphedema and then says, "Just what is that?" Your best reply would be that lymphedema is: A) the swelling of an extremity due to an obstructed lymph channel. B) an inflammation of the vein associated with thrombus formation. C) the indentation left after the examiner depresses the skin over swollen edematous tissue. D) a thickening and loss of elasticity of the arterial walls.

A

When taking a history from a newly admitted patient, the nurse notices that he often pauses and expectantly looks at the nurse. What would be the nurses best response to this behavior? A) Be silent, and allow him to continue when he is ready. B) Smile at him and say, Dont worry about all of this. Im sure we can find out why youre having these pains. C) Lean back in the chair and ask, You are looking at me kind of funny; there isnt anything wrong, is there? D) Stand up and say, I can see that this interview is uncomfortable for you. We can continue it another time.

A

Which best describes evidence based nursing practice? A) combining clinical expertise with the use of nursing research to provide the best care for patients while considering the patient's values and circumstances. B) appraising and looking at the implications of one or two articles as they relate to the culture and ethnicity of the patient C) completing a literature search to find relevant articles that use nursing research to encourage nurses to use good practices. D) finding value based resources to justify nursing actions when working with patients of diverse cultural backgrounds.

A

Which factor is identified as a priority influence on a patient's health status? A) poverty B) lifestyle factors C) legislative action D) occupational status

A

Which of these percussion findings would the nurse expect to find in a patient with a large amount of ascites? A) Dullness across the abdomen B) Flatness in the right upper quadrant C) Hyperresonance in the left upper quadrant D) Tympany in the right and left lower quadrants

A

Which of these statements is true regarding the vertebra prominens? The vertebra prominens is: A) The spinous process of C7. B) Usually nonpalpable in most individuals C) Opposite the interior border of the scapula. D) Located next to the manubrium of the sternum.

A

Which of these techniques uses the sense of touch to assess texture, temperature, moisture, and swelling when the nurse is assessing a patient? A) Palpation B) Inspection C) Percussion D) Auscultation

A

Which of these tests would the nurse use to check the motor coordination of an 11-month-old infant? A) Denver II B) Stereognosis C) Deep tendon reflexes D) Rapid alternating movements

A

Which organism is responsible for the majority of urinary tract infections in female patients? A) Escherichia coli B) Nesseria gonorrhea C) Candida albicans D) Haemophilus influenza

A

Which statement best describes religion? A) an organized system of beliefs concerning the cause, nature, and purpose, of the universe. B) belief in a divine and superhuman spirit to be obeyed and worshipped C) affiliation with one of the 1200 recognized religions in the United States D) The following of established rituals, especially in conjunction with health-seeking behaviors

A

Which technique is correct when the nurse is assessing the radial pulse of a patient?The pulse is counted for: A) 1 minute, if the rhythm is irregular. B) 15 seconds and then multiplied by 4, if the rhythm is regular. C) 2 full minutes to detect any variation in amplitude. D) 10 seconds and then multiplied by 6, if the patient has no history of cardiac abnormalities.

A

While discussing the history of a 6-month-old infant, the mother tells the nurse that she took a significant amount of aspirin while she was pregnant. What question would the nurse want to include in the history? A) "Does your baby seem to startle with loud noises?" B) "Has your baby had any surgeries on her ears?" C) "Have you noticed any drainage from her ears?" D) "How many ear infections has your baby had since birth?

A

While obtaining a history of a 3-month-old infant from the mother, the nurse asks about the infant's ability to suck and grasp the mother's finger. What is the nurse assessing? A) Reflexes B) Intelligence C) CNs D) Cerebral cortex function

A

While performing an assessment of a 65-year-old man with a history of hypertension and coronary artery disease, the nurse notices the presence of pitting edema in the lower legs bilaterally. The skin is puffy and tight but of normal color. There is no increased redness or tenderness over his lower legs, and the peripheral pulses are equal and strong. In this situation, the nurse suspects that the likely cause of the edema would be: A) Heart failure B) Venous thrombosis C) Local inflammation D) Blockage of lymphatic drainage

A

You are assigned to care for a patient with rectal bleeding admitted overnight. When you enter the patient's room to introduce yourself, you notice the patient is cool and clammy and their face is pale. Which type of physical assessment would be most appropriate for this patient? A) emergency assessment B) complete assessment C) focused assessment D) comprehensive assessment

A

You are reviewing concepts related to steps in the nursing process for determining prioritization and developing patient outcomes. To what are these actions attributed? A) planning B) assessment C) implementation D) diagnosis

A

the components of a nail exam should include: A) contour, consistency, and color B) size, shape, and surface C) texture, toughness, and translucency D) clubbing, pitting, and grooving

A

the nurse is performing a vision examination. Which of these charts is most widely used for vision examinations? A) Snellen B) Shetllen C) Smoollen D) Schwellon

A

the nurse is using an ostoscope to assess a patient's ears. which assessment technique is the nurse using? A) inspection B) percussion C) palpation D) auscultation

A

while taking a health history, the patient tells the nurse that "it feels like the room is spinning around me." How should the nurse document the finding? A) vertigo B) syncope C) dizziness D) seziure activity

A

Reflexive sympathetic dystrophy OR complex regional pain syndrome

A chronic progressive nerve condition, characterized by burning pain, swelling, stiffness. The nerve injury may modify usual pain pathway causing a "short circuit" -Innoculous stimulus can create intense pain.

The nurse is caring for a critically ill patient. What are the contraindications for administering medications by the oral route for this patient? (Select all that apply.) A) Vomiting B) Unconsciousness C) Fractured leg D) Penicillin allergy E) Family visitor F) Diarrhea

A, B

During an assessment, a patient mentions that "I just can't smell like I used to. I can barely smell the roses in my garden. Why is that?" For which possible causes of changes in the sense of smell will the nurse assess? (Select all that apply.) A) Aging B) Chronic allergies C) Cigarette smoking D) Chronic alcohol use E) Herpes simplex virus I F) Frequent episodes of strep throat

A, B, C

When emptying a patient's catheter drainage bag, the nurse notes that the urine appears to be discolored. The nurse understands that what factors may change the color of urine? (Select all that apply.) A) Taking the urinary tract analgesic phenazopyridine (Pyridium) B) A diet that includes a large amount of beets or blackberries C) An enlarged prostate or kidney stones D) High concentrations of bilirubin secondary to liver disease E) Increased carbohydrate intake

A, B, C, D

The nurse is assessing the intravenous (IV) infusion at the beginning of the shift. Which of these should be included in the assessment of the infusion? Select all that apply. A) Proper IV solution is infusing according to physician's orders. B) IV solution is infusing at the proper rate according to physician's orders. C) The infusion is proper according to the nurse's assessment of the patient's needs. D) Capillary refill in the fingers E) IV site date F) Whether the patient is voiding sufficiently

A, B, C, E

The nurse is completing an assessment on a patient who was just admitted from the emergency department. Which assessment findings would require immediate attention? (Select all that apply.) A) Sudden restlessness B) Temperature: 38.6°C C) Oxygen saturation: 95% D) Heart rate: 130 beats per minute E) Systolic blood pressure: 150 mm Hg F) Respiratory rate: 22 breaths per minute

A, B, D

The student nurse is correct in identifying which of the following as objective data? Select all that apply A) Respirations- 24 breaths/minute B) Hemoglobin- 13.5g/dL C) reports severe headache D) temperature- 98.4 deg F E) Wound size- 3 cm x 2cm

A, B, D, E

A patient has been admitted with chronic arterial symptoms. During the assessment, the nurse should expect which findings? Select all that apply. A) Patient has a history of diabetes and cigarette smoking. B) Skin of the patient is pale and cool. C) His ankles have two small, weeping ulcers. D) Patient works long hours sitting at a computer desk. E) He states that the pain gets worse when walking. F) Patient states that the pain is worse at the end of the day.

A, B, E

A nurse is performing an initial assessment on a client who is being admitted to the hospital for exacerbation of heart failure. During the client interview, the nurse wants to assess the client's background and health history. Which of the following are examples of leading questions that the nurse should avoid? Select all that apply A) you feel pretty good now, right? B) where are you working right now? C) You don't miss doses of your medications, do you? D) do you have a history of heart disease? E) who do you live with?

A, C

what instructions should the nurse give to a patient experiencing signs and symptoms related to decreased arterial insufficiency? Select all that apply A) avoid smoking and exposure to cold B) take acetaminophen if experiencing pain at night C) take aspirin or blood thinners as directed D) use additional blankets on the bed at night E) wear tight socks to keep feet warm

A, C, D

When considering cultural competence, the nurse must develop knowledge of discrete areas to understand the health care needs of others. These discrete areas include understanding of (Select all that apply.) A) his or her own heritage B) cultural and ethnic values C) heritage of the health system D) heritage of the nursing profession E) the heritage of the patient

A, C, D, E

which of the following assessment findings would be associated with a patient who is febrile? A) diaphoresis B) Pallor C) cyanosis D) erythema E) edema of the lower extremities F) hard, unmovable lymph nodes

A, D

A patient has been using herbal medication as part of her daily routine. Which actions should the nurse take? (Select all that apply.) A) Document the herbs as part of the medication history. B) Recommend a reputable company from which to buy herbs. C) Allow the patient to self-administer the herbs with her morning medications. D) Inform the health care provider of the findings E) Identify possible adverse effects of the herbal medications.

A, D, E

A patient is recovering from several hours of orthopedic surgery. During an assessment of the patient's lower legs, the nurse will monitor for signs of acute venous symptoms. Signs of acute venous symptoms include which of the following? Select all that apply. A) Intense, sharp pain, with the deep muscle tender to the touch B) Aching, tired pain, with a feeling of fullness C) Pain that is worse at the end of the day D) Sudden onset E) Warm, red, and swollen calf F) Pain that is relieved with elevation of the leg

A, D, E

The nurse is preparing for a certification course in skin care and needs to be familiar with the various lesions that may be identified on assessment of the skin. Which of the following definitions are correct? Select all that apply. A) Petechiae: Tiny punctate hemorrhages, 1 to 3 mm, round and discrete, dark red, purple, or brown in color B) Bulla: Elevated, circumscribed lesion filled with turbid fluid (pus) C) Papule: Hypertrophic scar D) Vesicle: Known as a friction blister E) Nodule: Solid, elevated, and hard or soft growth that is larger than 1 cm

A, D, E

During assessment of a patients pain, the nurse is aware that certain nonverbal behaviors are associated with chronic pain. Which of these behaviors are associated with chronic pain? Select all that apply. A) Sleeping B) Moaning C) Diaphoresis D) Bracing E) Restlessness F) Rubbing

A, D, F

Mr. Harris comes to the clinic for a follow-up appointment. On examination, you note a full "bounding" pulse. This is associated with: (Select all that apply.) A) anxiety. B) aortic valve stenosis. C) elevated temperature. D) hyperthyroidism. E) anemia. F) patent ductus arteriosus.

A, E, D, C

A patient with hypertension and heart failure demonstrates an understanding of patient discharge teaching by stating which of the following regarding dietary changes? Select all that apply A) I should restrict the amount of sodium I consume daily B) I should weight myself daily to determine if I', retaining fluid C) I should eat a high-calorie diet a few days a wekk D) I should check my blood sugar before each meal E) if i notice edema, I should restrict the amount of fluids I consume each day

A,B, E

The nurse is assessing voice sounds during a respiratory assessment. Which of these findings indicates a normal assessment? Select all that apply A) Voice sounds are faint, muffled, and almost inaudible when the patient whispers one, two, three in a very soft voice. B) As the patient repeatedly says ninety-nine, the examiner clearly hears the words ninety-nine. C) When the patient speaks in a normal voice, the examiner can hear a sound but cannot exactly distinguish what is being said. D) As the patient says a long ee-ee-ee sound, the examiner also hears a long ee-ee-ee sound.e. As the patient says a long ee-ee-ee sound, the examiner hears a long aaaaaa sound.

A,C,D

A 1-month-old infant has a head measurement of 34 cm and has a chest circumference of 32 cm. Based on the interpretation of these findings, the nurse would: A) Refer the infant to a physician for further evaluation. B) Consider these findings normal for a 1-month-old infant. C) Expect the chest circumference to be greater than the head circumference. D) Ask the parent to return in 2 weeks to re-evaluate the head and chest circumferences.

B

A 10-year-old is at the clinic for "a sore throat that has lasted 6 days." Which of these findingswould be consistent with an acute infection? A) Tonsils 3+/1-4+ with pale coloring B) Tonsils 3+/1-4+ with large white spots C) Tonsils 2+/1-4+ with small plugs of white debris D) Tonsils 1+/1-4+ and pink; the same color as the oral mucosa

B

A 13-year old girl is interested in obtaining information about the cause of her acne. The nurse would share with her that acne: A) Is contagious. B) Has no known cause. C) Is caused by increased sebum production. D) Has been found to be related to poor hygiene.

B

A 16-year-old boy has just been admitted to the unit for overnight observation after being in an automobile accident. What is the nurses best approach to communicating with him? A) Use periods of silence to communicate respect for him. B) Be totally honest with him, even if the information is unpleasant. C) Tell him that everything that is discussed will be kept totally confidential. D) Use slang language when possible to help him open up.

B

A 17-year-old student is a swimmer on her high school's swim team. She has had three bouts of otitis externa this season and wants to know what to do to prevent it. The nurse instructs her to: A) Use a cotton-tipped swab to dry the ear canals thoroughly after each swim. B) Use rubbing alcohol or 2% acetic acid eardrops after every swim. C) Irrigate the ears with warm water and a bulb syringe after each swim. D) Rinse the ears with a warmed solution of mineral oil and hydrogen peroxide.

B

A 22-year-old woman comes to the clinic because of a severe sunburn and states, "I was just out in the sun for a couple of minutes." The nurse begins a medication review with her, paying special attention to which medication class? A) Nonsteroidal antiinflammatory drugs for pain B) Tetracyclines for acne C) Proton pump inhibitors for heartburn D) Thyroid replacement hormone for hypothyroidism

B

A 31-year-old patient tells the nurse that he has noticed pain in his left ear when people speak loudly to him. The nurse knows that this finding: A) Is normal for people of his age. B) Is a characteristic of recruitment. C) May indicate a middle ear infection. D) Indicates that the patient has a cerumen impaction.

B

A 35-year-old man is seen in the clinic for an infection in his left foot. Which of these findings should the nurse expect to see during an assessment of this patient? A) Hard and fixed cervical nodes B) Enlarged and tender inguinal nodes C) Bilateral enlargement of the popliteal nodes D) Pelletlike nodes in the supraclavicular region

B

A 40-year-old man has come into the clinic with complaints of "extreme tenderness in my toes." The nurse notices that his toes are slightly swollen, reddened, and warm to the touch. His complaints would suggest: A) osteoporosis. B) acute gout. C) ankylosing spondylitis. D) degenerative joint disease.

B

A 40-year-old man states that his physician told him that has a hernia. He asks the nurse to explain what a hernia is. Which response by the nurse is appropriate? A) No need to worry. Most men your age develop hernias. B) A hernia is a loop of bowel protruding through a weak spot in the abdominal muscles. C) A hernia is the result of prenatal growth abnormalities that are just now causing problems. D) Ill have to have your physician explain this to you.

B

A 40-year-old woman reports a change in mole size, accompanied by color changes, itching, burning, and bleeding over the past month. She has a dark complexion and has no family history of skin cancer, but she has had many blistering sunburns in the past. The nurse would: A) Tell the patient to watch the lesion and report back in 2 months. B) Refer the patient because of the suggestion of melanoma on the basis of her symptoms. C) Ask additional questions regarding environmental irritants that may have caused this condition. D) Tell the patient that these signs suggest a compound nevus, which is very common in young to middle-aged adults.

B

A 52-year-old woman has a papule on her nose that has rounded, pearly borders and a central red ulcer. She said she first noticed it several months ago and that it has slowly grown larger. The nurse suspects which condition? A) Acne B) Basal cell carcinoma C) Melanoma D) Squamous cell carcinoma

B

A 70-year-old patient with a history of hypertension has a blood pressure of 180/100 mm Hg and a heart rate of 90 beats per minute. The nurse hears an extra heart sound at the apex immediately before the S1. The sound is heard only with the bell of the stethoscope while the patient is in the left lateral position. With these findings and the patients history, the nurse knows that this extra heart sound is most likely a(n): A) Split S1. B) Atrial gallop. C) Diastolic murmur. D) Summation sound.

B

A few days after a summer hiking trip, a 25-year-old man comes to the clinic with a rash. On examination, the nurse notes that the rash is red, macular, with a bull's eye pattern across his midriff and behind his knees. The nurse suspects: A) Rubeola. B) Lyme disease. C) Allergy to mosquito bites. D) Rocky Mountain spotted fever.

B

A male patient with a history of acquired immunodeficiency syndrome (AIDS) has come in for an examination and he states, I think that I have the mumps. The nurse would begin by examining the: A) Thyroid gland. B) Parotid gland. C) Cervical lymph nodes. D) Mouth and skin for lesions.

B

A mother brings her 28-month-old daughter into the clinic for a well-child visit. At the beginning of the visit, the nurse focuses attention away from the toddler, but as the interview progresses, the toddler begins to warm up and is smiling shyly at the nurse. The nurse will be most successful in interacting with the toddler if which is done next? A) Tickle the toddler, and get her to laugh. B) Stoop down to her level, and ask her about the toy she is holding. C) Continue to ignore her until it is time for the physical examination. D) Ask the mother to leave during the examination of the toddler, because toddlers often fuss less if their parent is not in view.

B

A mother brings her 3-month-old infant to the clinic for evaluation of a cold. She tells the nurse that he has had a runny nose for a week. When performing the physical assessment, the nurse notes that the child has nasal flaring and sternal and intercostal retractions. The nurses next action should be to: A) Assure the mother that these signs are normal symptoms of a cold B) Recognize that these are serious signs, and contact the physician. C) Ask the mother if the infant has had trouble with feedings D) Perform a complete cardiac assessment because these signs are probably indicative of early heart failure.

B

A mother brings her newborn in for an assessment and asks, Is there something wrong with my baby? His head seems so big. Which statement is true regarding the relative proportions of the head and trunk of the newborn? A) At birth, the head is one fifth the total length B) Head circumference should be greater than chest circumference at birth. C) The head size reaches 90% of its final size when the child is 3 years old. D) When the anterior fontanel closes at 2 months, the head will be more proportioned to the body.

B

A mother is concerned because her 18-month-old toddler has 12 teeth. She is wondering if this is normal for a child of this age. Which is the best response by the nurse? A) "How many teeth did you have at this age?" B) "This is a normal number of teeth for an 18 month old." C) "Normally, by age 2 1/2 years, 16 deciduous teeth are expected." D) "All 20 deciduous teeth are expected to erupt by age 4 years."

B

A mother of a 1-month-old infant asks the nurse why it takes so long for infants to learn to roll over. The nurse knows that the reason for this is that: A) A demyelinating process must be occurring with her infant. B) Myelin is needed to conduct the impulses, and the neurons of a newborn are not yet myelinated. C) The cerebral cortex is not fully developed; therefore, control over motor function gradually occurs. D) The spinal cord is controlling the movement because the cerebellum is not yet fully developed.

B

A nurse needs to auscultate posterior lung sounds in a patient. In which position would it be most appropriate to place the patient? A) semi-fowler's position B) lateral C) Sim's position D) Fowler's position

B

A patient admitted to the hospital with asthma has the following problems identified based on an admission health history and physical assessment. Which problem is a first-level priority? A) Ineffective self-health management B) Impaired gas exchange C) Readiness for enhanced spiritual well-being D) Risk for infection

B

A patient comes into the clinic complaining of pain in her right eye. On examination, the nurse sees a pustule at the lid margin that is painful to touch, red, and swollen. The nurse recognizes that this is a: A) Chalazion. B) Hordeolum (stye). C) Dacryocystitis. D) Blepharitis.

B

A patient comes to the clinic complaining of a cough that is worse at night but not as bad during the day. The nurse recognizes that this cough may indicate: A) Pneumonia. B) Postnasal drip or sinusitis. C) Exposure to irritants at work. D) Chronic bronchial irritation from smoking.

B

A patient comes to the clinic complaining of neck and shoulder pain and is unable to turn her head. The nurse suspects damage to CN ______ and proceeds with the examination by _____________. A) XI; palpating the anterior and posterior triangles B) XI; asking the patient to shrug her shoulders against resistance C) XII; percussing the sternomastoid and submandibular neck muscles D) XII; assessing for a positive Romberg sign

B

A patient comes to the emergency department after a boxing match, and his left eye is swollen almost shut. He has bruises on his face and neck. He says he is worried because he "can't see well" from his left eye. The physician suspects retinal damage. The nurse recognizes that signs of retinal detachment include: A) Loss of central vision. B) Shadow or diminished vision in one quadrant or one half of the visual field. C) Loss of peripheral vision. D) Sudden loss of pupillary constriction and accommodation.

B

A patient complains of leg pain that wakes him at night. He states that he "has been having problems" with his legs. He has pain in his legs when they are elevated that disappears when he dangles them. He recently noticed "a sore" on the inner aspect of the right ankle. On the basis of this history information, the nurse interprets that the patient is most likely experiencing: A) Pain related to lymphatic abnormalities. B) Problems related to arterial insufficiency. C) Problems related to venous insufficiency. D) Pain related to musculoskeletal abnormalities.

B

A patient has been admitted to a hospital after the staff in the nursing home noticed a pressure ulcer in his sacral area. The nurse examines the pressure ulcer and determines that it is a stage II ulcer. Which of these findings are characteristic of a stage II pressure ulcer? Select all that apply. A) Intact skin appears red but is not broken. B) Partial thickness skin erosion is observed with a loss of epidermis or dermis. C) Ulcer extends into the subcutaneous tissue. D) Localized redness in light skin will blanch with fingertip pressure. E) Open blister areas have a red-pink wound bed. F) Patches of eschar cover parts of the wound.

B

A patient has been diagnosed with osteoporosis and asks the nurse, "What is osteoporosis?" The nurse explains to the patient that osteoporosis is defined as: A) increased bone matrix B) Loss of bone density C) New, weaker bone growth D) Increased phagocytic

B

A patient has come in for an examination and states, I have this spot in front of my ear lobe on my cheek that seems to be getting bigger and is tender. What do you think it is? The nurse notes swelling below the angle of the jaw and suspects that it could be an inflammation of his: A) Thyroid gland. B) Parotid gland. C) Occipital lymph node. D) Submental lymph node.

B

A patient has hypoactive bowel sounds. The nurse knows that a potential cause of hypoactive bowel sounds is: A) Diarrhea. B) Peritonitis. C) Laxative use. D) Gastroenteritis.

B

A patient has suddenly developed shortness of breath and appears to be in significant respiratory distress. After calling the physician and placing the patient on oxygen, which of these actions is the best for the nurse to take when further assessing the patient? A) Count the patients respirations. B) Bilaterally percuss the thorax, noting any differences in percussion tones. C) Call for a chest x-ray study, and wait for the results before beginning an assessment. D)Inspect the thorax for any new masses and bleeding associated with respirations.

B

A patient in her first trimester of pregnancy is diagnosed with rubella. Which of these statements is correct regarding the significance of this in relation to the infant's hearing? A) Rubella may affect the mother's hearing but not the infant's. B) Rubella can damage the infant's organ of Corti, which will impair hearing. C) Rubella is only dangerous to the infant in the second trimester of pregnancy. D) Rubella can impair the development of CN VIII and thus affect hearing.

B

A patient in the emergency department with nausea and vomiting. Which would you include in the database? A) a complete health history and full physical examination B) a diet and GI history C) previously identified problems D) start collection of data in conjunction with livesaving measures.

B

A patient is able to flex his right arm forward without difficulty or pain but is unable to abduct his arm because of pain and muscle spasms; the nurse should suspect: A) crepitation. B) rotator cuff lesions. C) dislocated shoulder. D) rheumatoid arthritis.

B

A patient is in the primary care clinic for left sided weakness in their arm and leg for the past week. The nurse should perform which of the following nuerological exam? A) glasgow coma scale B) complete neurological exam C) screening neurologic exam D) neurologic recheck examination

B

A patient is scheduled for an intravenous pyelogram (IVP). Which piece of data would be most important to know before the procedure is carried out? A) Urinalysis showing negative results on testing for sugar and acetone B) History of allergies C) History of a recent thyroid scan D) Frequency of urination

B

A patient is suspected of having inflammation of the gallbladder, or cholecystitis. The nurse should conduct which of these techniques to assess for this condition? A) Obturator test B) Test for Murphy sign C) Assess for rebound tenderness D) Iliopsoas muscle test

B

A patient is unable to differentiate between sharp and dull stimulation to both sides of her face. The nurse suspects: A) Bell palsy. B) Damage to the trigeminal nerve. C) Frostbite with resultant paresthesia to the cheeks. D) Scleroderma.

B

A patient presents with acute pain of the abdomen. After the initial examination, how would you proceed? A) Withhold analgesic until diagnostic testing is completed B) Give patient medications as ordered C) Withhold analgesic until pain subsides. D) Determine what type of pain it is and proceed accordingly

B

A patient reports excruciating headache pain on one side of his head, especially around his eye, forehead, and cheek that has lasted approximately to 2 hours, occurring once or twice each day. The nurse should suspect. A) Hypertension. B) Cluster headaches. C) Tension headaches. D) Migraine headaches.

B

A patient says that she has recently noticed a lump in the front of her neck below her Adams apple that seems to be getting bigger. During the assessment, the finding that leads the nurse to suspect that this may not be a cancerous thyroid nodule is that the lump (nodule): A) Is tender. B) Is mobile and not hard. C) Disappears when the patient smiles. D) Is hard and fixed to the surrounding structures.

B

A patient tells the nurse that "all my life I've been called 'knock knees.'" The nurse knows that another term for "knock knees" is: A) genu varum. B) genu valgum. C) pes planus. D) metatarsus adductus.

B

A patient with pleuritis has been admitted to the hospital and complains of pain with breathing. What other key assessment finding would the nurse expect to find upon auscultation? A) Stridor B) Friction rub C) Crackles D) Wheezing

B

A patient's annual physical examination reveals a lateral curvature of the thoracic and lumbar segments of his spine; however, this curvature disappears with forward bending. The nurse knows that this abnormality of the spine is called: A) structural scoliosis. B) functional scoliosis. C) herniated nucleus pulposus. D) dislocated hip.

B

A patient's mother has noticed that her son, who has been to a new babysitter, has some blisters and scabs on his face and buttocks. On examination, the nurse notices moist, thin-roofed vesicles with a thin erythematous base and suspects: A) Eczema. B) Impetigo. C) Herpes zoster. D) Diaper dermatitis.

B

A patient's vision is recorded as 20/30 when the Snellen eye chart is used. The nurse interprets these results to indicate that: A) At 30 feet the patient can read the entire chart. B) The patient can read at 20 feet what a person with normal vision can read at 30 feet. C) The patient can read the chart from 20 feet in the left eye and 30 feet in the right eye. D) The patient can read from 30 feet what a person with normal vision can read from 20 feet.

B

A patients weekly blood pressure readings for 2 months have ranged between 124/84 mm Hg and 136/88 mm Hg, with an average reading of 126/86 mm Hg. The nurse knows that this blood pressure falls within which blood pressure category? A) Normal blood pressure B) Prehypertension C) Stage 1 hypertension D)Stage 2 hypertension

B

A pregnant woman states, I just know labor will be so painful that I wont be able to stand it. I know it sounds awful, but I really dread going into labor. The nurse responds by stating, Oh, dont worry about labor so much. I have been through it, and although it is painful, many good medications are available to decrease the pain. Which statement is true regarding this response? The nurses reply was a: A) Therapeutic response. By sharing something personal, the nurse gives hope to this woman. B) Nontherapeutic response. By providing false reassurance, the nurse actually cut off further discussion of the womans fears. C) Therapeutic response. By providing information about the medications available, the nurse is giving information to the woman. D) Nontherapeutic response. The nurse is essentially giving the message to the woman that labor cannot be tolerated without medication.

B

A semiconscious woman is brought to the emergency department after she was found on the floor in her kitchen. Her face, nail beds, lips, and oral mucosa are a bright cherry-red color. The nurse suspects that this coloring is due to: A) Polycythemia. B) Carbon monoxide poisoning. C) Carotenemia. D) Uremia.

B

A teenage patient comes to the emergency department with complaints of an inability to breathe and a sharp pain in the left side of his chest. The assessment findings include cyanosis, tachypnea, tracheal deviation to the right, decreased tactile fremitus on the left, hyperresonance on the left, and decreased breath sounds on the left. The nurse interprets that these assessment findings are consistent with: A) Bronchitis B) Pneumothorax C) Acute pneumonia. D) Asthmatic attack.

B

A woman is discussing the problems she is having with her 2-year-old son. She says, He wont go to sleep at night, and during the day he has several fits. I get so upset when that happens. The nurses best verbal response would be: A) Go on, Im listening. B) Fits? Tell me what you mean by this. C) Yes, it can be upsetting when a child has a fit. D) Dont be upset when he has a fit; every 2 year old has fits.

B

A woman who has had rheumatoid arthritis for years is starting to notice that her fingers are drifting to the side. The nurse knows that this condition is commonly referred to as: A) radial drift. B) ulnar deviation. C) swan neck deformity. D) Dupuytren's contracture.

B

After the examination of an infant, the nurse documents opisthotonos. The nurse recognizes that this finding often occurs with: A) Cerebral palsy. B) Meningeal irritation. C) Lower motor neuron lesion. D) Upper motor neuron lesion.

B

An 80-year-old woman is visiting the clinic for a checkup. She states, "I can't walk as much as I used to." The nurse is observing for motor dysfunction in her hip and should have her: A) internally rotate her hip while she is sitting. B) abduct her hip while she is lying on her back. C) adduct her hip while she is lying on her back. D) externally rotate her hip while she is standing.

B

An 85-year-old man has come in for a physical examination, and the nurse notices that he uses a cane. When documenting general appearance, the nurse should document this information under the section that covers: A) Posture. B) Mobility. C) Mood and affect. D) Physical deformity.

B

An 85-year-old patient comments during his annual physical that he seems to be getting shorter as he ages. The nurse should explain that decreased height occurs with aging because: A) Long bones tend to shorten with age B) The vertebral column C) Significant loss of subcutaneous fat occurs D) A thickening of the intervertebral disks develops

B

An elderly patient has been admitted to the intensive care unit (ICU) after falling at home. Within 8 hours, his condition has stabilized and he is transferred to a medical unit. The family is wondering whether he will be able to go back home. Which assessment instrument is most appropriate for the nurse to choose at this time? A) The Lawton IADL instrument B) Hospital Admission Risk Profile (HARP) C) The Mini-Cog D) The NEECHAM Confusion Scale

B

An elderly woman is brought to the emergency department after being found lying on the kitchen floor 2 days, and she is extremely dehydrated. What would the nurse expect to see upon examination? A) Smooth mucous membranes and lips B) Dry mucous membranes and cracked lips C) Pale mucous membranes D) White patches on the mucous membranes

B

An imaginary line connecting the highest point on each iliac crest would cross the _____ vertebra. A) first sacral B) fourth lumbar C) seventh cervical D) twelfth thoracic

B

An indwelling catheter is ordered for a postoperative patient who is unable to void. What is the primary concern of the nurse performing the procedure? A) Teaching deep-breathing techniques B) Maintaining strict aseptic technique C) Medicating the patient for pain before the procedure D) Positioning the patient for comfort during the procedure

B

An older patient has been diagnosed with pernicious anemia. The nurse knows that this condition could be related to: A) Increased gastric acid secretion. B) Decreased gastric acid secretion. C) Delayed gastrointestinal emptying time. D) Increased gastrointestinal emptying time.

B

Arteriosclerosis refers to: A) a deposition of fatty plaques along the intima of the arteries. B) thickening and loss of elasticity of the arterial walls. C) a sac formed by dilation in the arterial wall. D) a variation from the heart's normal rhythm.

B

Average urine pH is A) 4 B) 6 C) 7 D) 9

B

During a clinic visit, a woman in her seventh month of pregnancy complains that her legs feel "heavy in the calf" and that she often has foot cramps at night. The nurse notices that the patient has dilated, tortuous veins in her lower legs. Which condition is reflected by these findings? A) Deep-vein thrombophlebitis B) Varicose veins C) Lymphedema D) Raynaud phenomenon

B

During a complete health assessment, how would the nurse test the patients hearing? A) Observing how the patient participates in normal conversation B) Using the whispered voice test C) Using the Weber and Rinne tests D( Testing with an audiometer

B

During a functional assessment of an older person's home environment, which statement or question by the nurse is most appropriate regarding common environmental hazards? A) "These low toilet seats are safe because they are nearer to the ground in case of falls." B) "Do you have a relative or friend who can help to install grab bars in your shower?" C) "These small rugs are ideal for preventing you from slipping on the hard floor." D) "It would be safer to keep the lighting low in this room to avoid glare in your eyes."

B

During a physical education class, a student is hit in the eye with the end of a baseball bat. When examined in the emergency department, the nurse notices the presence of blood in the anterior chamber of the eye. This finding indicates the presence of: A) Hypopyon. B) Hyphema. C) Corneal abrasion. D) Pterygium.

B

During a prenatal check, a patient begins to cry as the nurse asks her about previous pregnancies. She states that she is remembering her last pregnancy, which ended in miscarriage. The nurses best response to her crying would be: A) Im so sorry for making you cry! B) I can see that you are sad remembering this. It is all right to cry. C) Why dont I step out for a few minutes until youre feeling better?D) I can see that you feel sad about this; why dont we talk about something else?

B

During a skin assessment, the nurse notices that a Mexican-American patient has skin that is yellowish-brown; however, the skin on the hard and soft palate is pink and the patient's scleras are not yellow. From this finding, the nurse could probably rule out: A) Pallor B) Jaundice C) Cyanosis D) Iron deficiency

B

During a visit to the clinic, a patient states, The doctor just told me he thought I ought to stop smoking. He doesnt understand how hard Ive tried. I just dont know the best way to do it. What should I do? The nurses most appropriate response in this case would be: A) Id quit if I were you. The doctor really knows what he is talking about. B) Would you like some information about the different ways a person can quit smoking? C) Stopping your dependence on cigarettes can be very difficult. I understand how you feel. D)Why are you confused? Didnt the doctor give you the information about the smoking cessation program we offer?

B

During an abdominal assessment, the nurse is unable to hear bowel sounds in a patients abdomen. Before reporting this finding as silent bowel sounds, the nurse should listen for at least A) 1 minute. B) 5 minutes. C) 10 minutes. D) 2 minutes in each quadrant.

B

During an abdominal assessment, the nurse would consider which of these findings as normal? A) Presence of a bruit in the femoral area B) Tympanic percussion note in the umbilical region C) Palpable spleen between the ninth and eleventh ribs in the left midaxillary line D) Dull percussion note in the left upper quadrant at the midclavicular line

B

During an assessment of a 22-year-old woman who has a head injury from a car accident 4 hours ago, the nurse notices the following change: pupils were equal, but now the right pupil is fully dilated and nonreactive, left pupil is 4 mm and reacts to light. What does finding this suggest? A) Injury to the right eye B) Increased intracranial pressure C) Test inaccurately performed D) Normal response after a head injury

B

During an assessment of a 68-year-old man with a recent onset of right-sided weakness, the nurse hears a blowing, swishing sound with the bell of the stethoscope over the left carotid artery. This finding would indicate: A) Valvular disorder. B) Blood flow turbulence. C) Fluid volume overload. D) Ventricular hypertrophy.

B

During an assessment of an adult, the nurse has noted unequal chest expansion and recognizes that this occurs in which situation? A) In an obese patient B) When part of the lung is obstructed or collapsed C) When bulging of the intercostal spaces is present D) When accessory muscles are used to augment respiratory effort

B

During an assessment of an infant, the nurse notes that the fontanels are depressed and sunken. The nurse suspects which condition? A) Rickets B) Dehydration C) Mental retardation D) Increased intracranial pressure

B

During an assessment of the cranial nerves, the nurse finds the following: asymmetry when the patient smiles or frowns, uneven lifting of eyebrows, sagging of the lower eyelids, and escape of air when the nurse presses against the right puffed cheek. This would indicate dysfunction of which of these cranial nerves? A) Motor component of CN IV B) Motor component of CN VII C) Motor and sensory components of CN XI D) Motor component of CN X and sensory component of CN VII

B

During an assessment the nurse has elevated a patient's legs 12 inches off the table and has had him wag his feet to drain off venous blood. After helping him to sit up and dangle his legs over the side of the table, the nurse should expect a normal finding at this point would be: A) Significant elevational pallor. B) Venous filling within 15 seconds. C) No change in the coloration of the skin. D) Color returning to the feet within 20 seconds of assuming a sitting position.

B

During an assessment, a patient tells the nurse that her fingers often change color when she goes out in cold weather. She describes these episodes as her fingers first turning white, then blue, then red with a burning, throbbing pain. The nurse suspects that she is experiencing: A) Lymphedema. B) Raynaud disease. C) Deep-vein thrombosis. D) Chronic arterial insufficiency.

B

During an assessment, the nurse notices that a patient's left arm is swollen from the shoulder down to the fingers, with nonpitting brawny edema. The right arm is normal. The patient had a left-sided mastectomy 1 year ago. The nurse suspects which problem? A) Venous stasis B) Lymphedema C) Arteriosclerosis D) Deep-vein thrombosis

B

During an assessment, the nurse uses the "profile sign" to detect: A) Pitting edema. B) Early clubbing. C) Symmetry of the fingers. D) Insufficient capillary refill.

B

During an examination of the anterior thorax, the nurse is aware that the trachea bifurcates anteriorly at the: A) Costal angle B) Sternal angle. C) Xiphoid process. D) Suprasternal notch.

B

During an examination, the nurse notices severe nystagmus in both eyes of a patient. Which of these conclusions by the nurse is correct? A) Is a normal occurrence B) May indicate disease of the cerebellum or brainstem. C) Is a sign that the patient is nervous about the examination. D) Indicates a visual problem, and a referral to an ophthalmologist is indicated.

B

During an interview the patient states, "I can feel this bump on the top of both of my shoulders—it doesn't hurt but I am curious about what it might be." The nurse should tell the patient, "That is: A) your subacromial bursa." B) your acromion process." C) your glenohumeral joint." D) the greater tubercle of your humerus."

B

During examination, the nurse finds that a patient is unable to distinguish objects placed in his hand. The nurse would document: A) Stereognosis. B) Astereognosis. C) Graphesthesia. D) Agraphesthesia.

B

During inspection of the posterior chest, the nurse should assess for: A) Symmetric expansion B). Symmetry of shoulders and muscles. C) Tactile fremitus. D) Diaphragmatic excursion.

B

During the neurologic assessment of a "healthy" 35-year-old patient, the nurse asks him to relax his muscles completely. The nurse then moves each extremity through full range of motion. Which of these results would the nurse expect to find? A) Firm, rigid resistance to movement B) Mild, even resistance to movement C) Hypotonic muscles as a result of total relaxation D) Slight pain with some directions of movement

B

Durning palpation of the anterior chest wall, the nurse notices a coarse, crackling sensation over the skin surface. On the basis of these findings, the nurse suspects: A) Tactile fremitus. B) Crepitus. C) Friction rub. D) Adventitious sounds.

B

Each culture has its own healers who usually A) own and operate specialty community clinics. B) cost less than traditional or biomedical providers. C) recommend folk practices that are dangerous. D) speak at least two languages.

B

For the patient who is experiencing expressive aphasia, which nursing intervention is most helpful in promoting ocmmunication? A) speaking loudly and slowly B) using a picture board for the patient to point to pictures C) writing directions so the patient can read them D) speaking in short sentences

B

In assessing a 70-year-old man, the nurse finds the following: blood pressure 140/100 mm Hg; heart rate 104 beats per minute and slightly irregular; and the split S2 heart sound. Which of these findings can be explained by expected hemodynamic changes related to age? A) Increase in resting heart rate B) Increase in systolic blood pressure C) Decrease in diastolic blood pressure D) Increase in diastolic blood pressure

B

In assessing for an S4 heart sound with a stethoscope, the nurse would listen with the: A) Bell of the stethoscope at the base with the patient leaning forward. B) Bell of the stethoscope at the apex with the patient in the left lateral position. C) Diaphragm of the stethoscope in the aortic area with the patient sitting. D) Diaphragm of the stethoscope in the pulmonic area with the patient supine.

B

In assessing the carotid arteries of an older patient with cardiovascular disease, the nurse would: A) Palpate the artery in the upper one third of the neck. B) Listen with the bell of the stethoscope to assess for bruits. C) Simultaneously palpate both arteries to compare amplitude. D) Instruct the patient to take slow deep breaths during auscultation.

B

In assessing the tonsils of a 30-year-old, the nurse notices that they are involuted, granular inappearance, and appear to have deep crypts. What is the correct response to these findings? A) Refer the patient to a throat specialist. B) No response is needed; this appearance is normal for the tonsils. C) Continue with the assessment, looking for any other abnormal findings. D) Obtain a throat culture on the patient for possible streptococcal (strep) infection

B

In obtaining a history on a 74-year-old patient the nurse notes that he drinks alcohol daily and that he has noticed a tremor in his hands that affects his ability to hold things. With this information, what should the nurse's response be? A) "Does your family know you are drinking every day?" B) "Does the tremor change when you drink alcohol?" C) "We'll do some tests to see what is causing the tremor." D) "You really shouldn't drink so much alcohol; it may be causing your tremor."

B

In performing a voice test to assess hearing, which of these actions would the nurse perform? A) Shield the lips so that the sound is muffled. B) Whisper a set of random numbers and letters, and then ask the patient to repeat them. C) Ask the patient to place his finger in his ear to occlude outside noise. D) Stand approximately 4 feet away to ensure that the patient can really hear at this distance.

B

Lymph nodes are palpable in: A) adults with infections. B) All of the options. C) children with infections. D) healthy children.

B

Mrs. Schneider comes to the office for a routine health assessment and without complaints. On examination, you note a water-hammer "Corrigan's" pulse. This is associated with: A) hyperkinetic states. B) aortic valve regurgitation. C) conduction disturbance. D) decreased cardiac output.

B

One of the leg's deep veins is the: A) great saphenous. B) popliteal. C) small saphenous. D) tibial.

B

Pain issues should be anticipated in a cognitively impaired older adult with a history of: A) Constipation B) Peripheral vascular disease C) COPD D) Parkinson Disease

B

Palatine, pharyngeal, and lingual are specific names for: A) cervical lymph nodes. B) tonsils. C) epitrochlear lymph nodes. D) axillary lymph nodes.

B

The ability that humans have to perform very skilled movements such as writing is controlled by the: A) Basal ganglia. B) Corticospinal tract. C) Spinothalamic tract. D) Extrapyramidal tract

B

The assessment of a 60-year-old patient has taken longer than anticipated. In testing his pain perception the nurse decides to complete the test as quickly as possible. When the nurse applies the sharp point of the pin on his arm several times, he is only able to identify these as one "very sharp prick." What would be the most accurate explanation for this? A) The patient has hyperesthesia as a result of the aging process. B) This response is most likely the result of the summation effect. C) The nurse was probably not poking hard enough with the pin in the other areas. D) The patient most likely has analgesia in some areas of arm and hyperalgesia in others.

B

The component of the conduction system referred to as the pacemaker of the heart is the: A) Atrioventricular (AV) node. B) Sinoatrial (SA) node. C) Bundle of His. D) Bundle branches.

B

The direction of blood flow through the heart is best described by which of these? A) Vena cava right atrium right ventricle lungs pulmonary artery left atrium left ventricle B) Right atrium right ventricle pulmonary artery lungs pulmonary vein left atrium left ventricle C) Aorta right atrium right ventricle lungs pulmonary vein left atrium left ventricle vena cava D) Right atrium right ventricle pulmonary vein lungs pulmonary artery left atrium left ventricle

B

The expert nurse differs from the novice nurse by acting without consciously thinking about the actions. This is referred to as A) deductive reasoning B) intuition C) the nursing process D) focused assessment

B

The nurse has administered a pain medication to a patient by an intravenous infusion. The nurse should reassess the patient's response to the pain medication within _____ minutes. A) 5 B) 15 C) 30 D) 60

B

The nurse has just completed a lymph node assessment on a 60-year-old healthy female patient. The nurse knows that most lymph nodes in healthy adults are normally: A) Shotty. B) Nonpalpable. C) Large, firm, and fixed to the tissue. D) Rubbery, discrete, and mobile.

B

The nurse has used interpretation regarding a patients statement or actions. After using this technique, it would be best for the nurse to: A) Apologize, because using interpretation can be demeaning for the patient. B) Allow time for the patient to confirm or correct the inference. C) Continue with the interview as though nothing has happened. D) Immediately restate the nurses conclusion on the basis of the patients nonverbal response.

B

The nurse is assessing a 16-year-old patient who has suffered head injuries from a recent motor vehicle accident. Which of these statements indicates the most important reason for assessing for any drainage from the ear canal? A) If the drum has ruptured, then purulent drainage will result. B) Bloody or clear watery drainage can indicate a basal skull fracture. C) The auditory canal many be occluded from increased cerumen. D) Foreign bodies from the accident may cause occlusion of the canal.

B

The nurse is assessing a patient who has liver disease for jaundice. Which of these assessment findings is indicative of true jaundice? A) Yellow patches in the outer sclera B) Yellow color of the sclera that extends up to the iris C) Skin that appears yellow when examined under low light D) Yellow deposits on the palms and soles of the feet where jaundice first appears

B

The nurse is assessing a patient with an indwelling catheter and finds that the catheter is not draining and the patient's bladder is distended. What action should the nurse take next? A) Notify the primary care provider (PCP). B) Assess the tubing for kinks and ensure downward flow. C) Change the catheter as soon as possible. D) Aspirate the stagnant urine in the catheter for culture.

B

The nurse is assessing a patient with possible cardiomyopathy and assesses the hepatojugular reflux. If heart failure is present, then the nurse should recognize which finding while pushing on the right upper quadrant of the patients abdomen, just below the rib cage? A) The jugular veins will rise for a few seconds and then recede back to the previous level if the heart is properly working. B) The jugular veins will remain elevated as long as pressure on the abdomen is maintained. C) An impulse will be visible at the fourth or fifth intercostal space at or inside the midclavicular line. D) The jugular veins will not be detected during this maneuver.

B

The nurse is assessing a patient's ischial tuberosity. To palpate the ischial tuberosity, the nurse knows that it is best to have the patient: A) standing B) flexing the hip C) flexing the knee D) Lying in the supine position

B

The nurse is assessing a patients skin during an office visit. What part of the hand and technique should be used to best assess the patients skin temperature? A) Fingertips; they are more sensitive to small changes in temperature. B) Dorsal surface of the hand; the skin is thinner on this surface than on the palms. C) Ulnar portion of the hand; increased blood supply in this area enhances temperature sensitivity. D) Palmar surface of the hand; this surface is the most sensitive to temperature variations because of its increased nerve supply in this area.

B

The nurse is assessing the abdomen of a pregnant woman who is complaining of having acid indigestion all the time. The nurse knows that esophageal reflux during pregnancy can cause: A) Diarrhea. B) Руrosis. C) Dysphagia D) Constipation.

B

The nurse is assessing the abdomen of an older adult. Which statement regarding the older adult and abdominal assessment is true? A) Abdominal tone is increased. B) Abdominal musculature is thinner. C) Abdominal rigidity with an acute abdominal condition is more common. D) The older adult with an acute abdominal condition complains more about pain than the younger person.

B

The nurse is assessing the neurologic status of a patient who has a late-stage brain tumor. With the reflex hammer, the nurse draws a light stroke up the lateral side of the sole of the foot and inward, across the ball of the foot. In response, the patient's toes fan out, and the big toe shows dorsiflexion. The nurse interprets this result as: A) Negative Babinski sign, which is normal for adults. B) Positive Babinski sign, which is abnormal for adults. C) Clonus, which is a hyperactive response. D) Achilles reflex, which is an expected response.

B

The nurse is assessing the skin of a patient who has AIDS and notices multiple patch-like lesions on the temple and beard area that are faint pink in color. The nurse recognizes these lesions as: A) Measles (rubeola). B) Kaposis sarcoma. C) Angiomas. D) Herpes zoster.

B

The nurse is assessing the vital signs of a 20-year-old male marathon runner and documents the following vital signs: temperature36 C; pulse48 beats per minute; respirations14 breaths per minute; blood pressure104/68 mm Hg. Which statement is true concerning these results? A) The patient is experiencing tachycardia. B) These are normal vital signs for a healthy, athletic adult. C) The patients pulse rate is not normalhis physician should be notified. D) On the basis of these readings, the patient should return to the clinic in 1 week.

B

The nurse is attempting to assess the femoral pulse in an obese patient. Which of these actions would be most appropriate? A) The patient is asked to assume a prone position. B) The patient is asked to bend his or her knees to the side in a froglike position. C) The nurse firmly presses against the bone with the patient in a semi-Fowler position. D) The nurse listens with a stethoscope for pulsations; palpating the pulse in an obese person is extremely difficult.

B

The nurse is conducting a visual examination. Which of these statements regarding visual pathways and visual fields is true? A) The right side of the brain interprets the vision for the right eye. B) The image formed on the retina is upside down and reversed from its actual appearance in the outside world. C) Light rays are refracted through the transparent media of the eye before striking the pupil. D) Light impulses are conducted through the optic nerve to the temporal lobes of the brain.

B

The nurse is conducting an interview with a woman who has recently learned that she is pregnant and who has come to the clinic today to begin prenatal care. The woman states that she and her husband are excited about the pregnancy but have a few questions. She looks nervously at her hands during the interview and sighs loudly. Considering the concept of communication, which statement does the nurse know to be most accurate? The woman is: A) Excited about her pregnancy but nervous about the labor. B)Exhibiting verbal and nonverbal behaviors that do not match. C) Excited about her pregnancy, but her husband is not and this is upsetting to her. D) Not excited about her pregnancy but believes the nurse will negatively respond to her if she states this.

B

The nurse is examining a patient who is complaining of feeling cold. Which is a mechanism of heat loss in the body? A)Exercise B) Radiation C) Metabolism D) Food digestion

B

The nurse is examining an infant and prepares to elicit the Moro reflex at which time during the examination? A) When the infant is sleeping B) At the end of the examination C) Before auscultation of the thorax D) Halfway through the examination

B

The nurse is examining the hip area of a patient and palpates a flat depression on the upper, lateral side of the thigh when the patient is standing. The nurse interprets this finding as the: A) ischial tuberosity. B) greater trochanter. C) iliac crest. D) gluteus maximus muscle.

B

The nurse is examining the lymphatic system of a healthy 3-year-old child. Which finding should the nurse expect? A) Excessive swelling of the lymph nodes B) Presence of palpable lymph nodes C) No palpable nodes because of the immature immune system of a child D) Fewer numbers and a smaller size of lymph nodes compared with those of an adult

B

The nurse is listening to bowel sounds. Which of these statements is true of bowel sounds? Bowel sounds: A) Are usually loud, high-pitched, rushing, and tinkling sounds. B) Are usually high-pitched, gurgling, and irregular sounds. C) Sound like two pieces of leather being rubbed together. D) Originate from the movement of air and fluid through the large intestine.

B

The nurse is nearing the end of an interview. Which statement is appropriate at this time? A) Did we forget something? B) Is there anything else you would like to mention? C) I need to go on to the next patient. Ill be back. D) While Im here, lets talk about your upcoming surgery.

B

The nurse is palpating the sinus areas. If the findings are normal, then the patient shouldreport which sensation? A) No sensation B) Firm pressure C) Pain during palpation D) Pain sensation behind eyes

B

The nurse is performing a middle ear assessment on a 15-year-old patient who has had a history of chronic ear infections. When examining the right tympanic membrane, the nurse sees the presence of dense white patches. The tympanic membrane is otherwise unremarkable. It is pearly, with the light reflex at 5 o'clock and landmarks visible. The nurse should: A) Refer the patient for the possibility of a fungal infection. B) Know that these are scars caused from frequent ear infections. C) Consider that these findings may represent the presence of blood in the middle ear. D) Be concerned about the ability to hear because of this abnormality on the tympanic membrane.

B

The nurse is performing an eye assessment on an 80-year-old patient. Which of these findings is considered abnormal? A) Decrease in tear production B) Unequal pupillary constriction in response to light C) Presence of arcus senilis observed around the cornea D) Loss of the outer hair on the eyebrows attributable to a decrease in hair follicles

B

The nurse is performing the diagnostic positions test. Normal findings would be which of these results? A) Convergence of the eyes B) Parallel movement of both eyes C) Nystagmus in extreme superior gaze D) Slight amount of lid lag when moving the eyes from a superior to an inferior position

B

The nurse is placing an indwelling catheter in a female patient. She inserts the catheter into the vagina. What is the next action for the nurse to implement? A) Collect a urine specimen and notify the PCP. B) Leave the catheter in place and insert a new catheter into the urethra. C) Remove the catheter from the vagina and place it into the urethra. D) Ask another nurse to attempt the catheterization of the patient.

B

The nurse is preparing to auscultate for heart sounds. Which technique is correct? A) Listening to the sounds at the aortic, tricuspid, pulmonic, and mitral areas B) Listening by inching the stethoscope in a rough Z pattern, from the base of the heart across and down, then over to the apex C) Listening to the sounds only at the site where the apical pulse is felt to be the strongest D) Listening for all possible sounds at a time at each specified area

B

The nurse is preparing to examine a 4-year-old child. Which action is appropriate for this age group? A) Explain the procedures in detail to alleviate the childs anxiety. B) Give the child feedback and reassurance during the examination. C) Do not ask the child to remove his or her clothes because children at this age are usually very private. D) Perform an examination of the ear, nose, and throat first, and then examine the thorax and abdomen

B

The nurse is preparing to examine a patient who has been complaining of right lower quadrant pain. Which technique is correct during the assessment? The nurse should: A) Examine the tender area first. B) Examine the tender area last. C) Avoid palpating the tender area. D) Palpate the tender area first, and then auscultate for bowel sounds.

B

The nurse is preparing to perform a physical assessment. Which statement is true about the physical assessment? The inspection phase: A) Usually yields little information. B) Takes time and reveals a surprising amount of information. C) May be somewhat uncomfortable for the expert practitioner. D) Requires a quick glance at the patients body systems before proceeding with palpation.

B

The nurse is preparing to use the Lawton IADL instrument as part of an assessment. Which statement about the Lawton IADL instrument is true? A) The nurse uses direct observation to implement this tool. B) It is designed as a self-report measure of performance rather than ability. C) It is not useful in the acute hospital setting. D) It is best used for those residing in an institutional setting

B

The nurse is reviewing an assessment of a patient's peripheral pulses and notices that the documentation states that the radial pulses are "2+." The nurse recognizes that this reading indicates what type of pulse? A) Bounding B) Normal C) Weak D) Absent

B

The nurse is reviewing in age-related changes in the eye for a class. Which of these physiologic changes is responsible for presbyopia? A) Degeneration of the cornea B) Loss of lens elasticity C) Decreased adaptation to darkness D) Decreased distance vision abilities

B

The nurse is reviewing risk factors for venous disease. Which of these situations best describes a person at highest risk for development of venous disease? A) Woman in her second month of pregnancy B) Person who has been on bed rest for 4 days C) Person with a 30-year, 1 pack per day smoking habit D) Older adult taking anticoagulant medication

B

The nurse is teaching a class on basic assessment skills. Which of these statements is true regarding the stethoscope and its use? A) Slope of the earpieces should point posteriorly (toward the occiput). B) Although the stethoscope does not magnify sound, it does block out extraneous room noise. C) Fit and quality of the stethoscope are not as important as its ability to magnify sound. D)Ideal tubing length should be 22 inches to dampen the distortion of sound.

B

The nurse is teaching a review class on the lymphatic system. A participant shows correct understanding of the material with which statement? A) "Lymph flow is propelled by the contraction of the heart." B) "The flow of lymph is slow, compared with that of the blood." C) "One of the functions of the lymph is to absorb lipids from the biliary tract." D) "Lymph vessels have no valves; therefore, lymph fluid flows freely from the tissue spaces into the bloodstream."

B

The nurse is using the pads of their fingers to assess for subcutaneous emphysema after chest tube placement. which assessment technique is the nurse using? A) inspection B) palpation C) percussion D) auscultation

B

The nurse is watching a new graduate nurse perform auscultation of a patients abdomen. Which statement by the new graduate shows a correct understanding of the reason auscultation precedes percussion and palpation of the abdomen? A) We need to determine the areas of tenderness before using percussion and palpation. B) Auscultation prevents distortion of bowel sounds that might occur after percussion and palpation. C) Auscultation allows the patient more time to relax and therefore be more comfortable with the physical examination. D) Auscultation prevents distortion of vascular sounds, such as bruits and hums, that might occur after percussion and palpation.

B

The nurse keeps in mind that a thorough skin assessment is very important because the skin holds information about a person's: A) Support systems. B) Circulatory status. C) Socioeconomic status. D) Psychological wellness

B

The nurse keeps in mind that the most important reason to share information and to offer brief teaching while performing the physical examination is to help the: A) Examiner feel more comfortable and to gain control of the situation. B) Examiner to build rapport and to increase the patients confidence in him or her. C) Patient understand his or her disease process and treatment modalities. D) Patient identify questions about his or her disease and the potential areas of patient education.

B

The nurse knows that a normal finding when assessing the respiratory system of an older adult is: A) Increased thoracic expansion. B) Decreased mobility of the thorax. C) Decreased anteroposterior diameter. D) Bronchovesicular breath sounds throughout the lungs.

B

The nurse knows that during an abdominal assessment, deep palpation is used to determine: A) Bowel motility. B) Enlarged organs C) Superficial tenderness. D) Overall impression of skin surface and superficial musculature.

B

The nurse knows that normal splitting of the S2 is associated with: A) Expiration. B) Inspiration. C) Exercise state. D) Low resting heart rate.

B

The nurse knows that testing kinesthesia is a test of a person's: A) Fine touch. B) Position sense. C) Motor coordination. D) Perception of vibration.

B

The nurse places a key in the hand of a patient and he identifies it as a penny. What term would the nurse use to describe this finding? A) Extinction B) Astereognosis C) Graphesthesia D) Tactile discrimination

B

The nurse suspects that a patient has otitis media. Early signs of otitis media include which of these findings of the tympanic membrane? A) Red and bulging B) Hypomobility C) Retraction with landmarks clearly visible D) Flat, slightly pulled in at the center, and moves with insufflation

B

The nurse would use bimanual palpation technique in which situation? A) Palpating the thorax of an infant B) Palpating the kidneys and uterus C) Assessing pulsations and vibrations D) Assessing the presence of tenderness and pain

B

The physician reports that a patient with a neck tumor has a tracheal shift. The nurse is aware that this means that the patients trachea is: A) Pulled to the affected side. B) Pushed to the unaffected side. C) Pulled downward. D) Pulled downward in a rhythmic pattern.

B

Two parts of the nervous system are the: A) Motor and sensory. B) Central and peripheral. C) Peripheral and autonomic. D) Hypothalamus and cerebral.

B

What is the yin/yang theory of health? A) Health exists in the absence of illness. B) Health exists when all aspects of the person are in perfect balance. C) Health exists when physical, psychological, spiritual, and social needs are met. D) Health exists when there is optimal functioning.

B

What occurs during transduction (the first phase of nociceptive pain)? A) Chemical mediators are neutralized to decrease the perception of pain. B) Pain signals move from the site of origin to the spinal cord. C) The brain interprets the pain signal. D) The pain impulse moves from the spinal cord to the brain.

B

What should the nurse do first when preparing to administer medications to a patient? A) Check the medication expiration date. B) Check the medication administration record (MAR). C) Call the pharmacy for administration instructions. D) Check the patient's name band.

B

What type of database is most appropriate for an individual who is admitted to a long-term care facility? A) focused B) complete C) emergency D) follow up

B

When assessing a patient in the hospital setting, the nurse knows that which statement is true? A) The patient will need a brief assessment at least every 4 hours. B) The patient will need a consistent, specialized examination every 8 hours that focuses on certain parameters. C) The patient will need a complete head-to-toe physical examination every 24 hours. D) Most patients require a minimal examination each shift unless they are in critical condition.

B

When assessing a patient the nurse notes that the left femoral pulse as diminished, 1+/4+. What should the nurse do next? A) Document the finding. B) Auscultate the site for a bruit. C) Check for calf pain. D) Check capillary refill in the toes.

B

When assessing a patients pain, the nurse knows that an example of visceral pain would be: A) Hip fracture. B) Cholecystitis. C) Second-degree burns. D) Pain after a leg amputation.

B

When assessing an older adult, which vital sign changes occur with aging? A) Increase in pulse rate B) Widened pulse pressure C) Increase in body temperature D) Decrease in diastolic blood pressure

B

When assessing the intensity of a patients pain, which question by the nurse is appropriate? A) What makes your pain better or worse? B) How much pain do you have now? C) How does pain limit your activities? D) What does your pain feel like?

B

When assessing the pulse of a 6-year-old boy, the nurse notices that his heart rate varies with his respiratory cycle, speeding up at the peak of inspiration and slowing to normal with expiration. The nurses next action would be to: A) Immediately notify the physician. B) Consider this finding normal in children and young adults. C) Check the childs blood pressure, and note any variation with respiration. D) Document that this child has bradycardia, and continue with the assessment.

B

When beginning to assess a person's spirituality, which question by the nurse would be most appropriate? A) "Do you believe in God?" B) "How does your spirituality relate to your health care decisions?" C) "What religious faith do you follow?" D) "Do you believe in the power of prayer?"

B

When completing a health assessment, which of the following actions most demonstrates cultural competence? A) Ask about family history of diseases. B) Ask about use of traditional, herbal, or folk remedies. C) Make sure the blood pressure cuff fits appropriately. D) Measure height and weight in a private room.

B

When examining the ear with an otoscope, the nurse notes that the tympanic membrane should appear: A) Light pink with a slight bulge. B) Pearly gray and slightly concave. C) Pulled in at the base of the cone of light. D) Whitish with a small fleck of light in the superior portion.

B

When examining the mouth of an older patient, the nurse recognizes which finding is due to the aging process? A) Teeth appearing shorter B) Tongue that looks smoother in appearance C) Buccal mucosa that is beefy red in appearance D) Small, painless lump on the dorsum of the tongue

B

When performing a physical assessment, the first technique the nurse will always use is: A)Palpation. B) Inspection. C) Percussion. D) Auscultation.

B

When performing a physical examination, safety must be considered to protect the examiner and the patient against the spread of infection. Which of these statements describes the most appropriate action the nurse should take when performing a physical examination? A) Washing ones hands after removing gloves is not necessary, as long as the gloves are still intact. B) Hands are washed before and after every physical patient encounter. C) Hands are washed before the examination of each body system to prevent the spread of bacteria from one part of the body to another. D) Gloves are worn throughout the entire examination to demonstrate to the patient concern regarding the spread of infectious diseases.

B

When using the various instruments to assess an older person's activities of daily living (ADLs), the nurse needs to remember that a disadvantage of these instruments includes: A) The reliability of the tools. B) Self or proxy report of functional activities. C) Lack of confidentiality during the assessment. D) Insufficient detail about the deficiencies identified.

B

Which health belief practice is associated with patients who are of American Indian heritage? A) wearing bangle bracelets to ward off evil spirits B) believing that forces of nature must be kept in natural balance C) using swamp root as a traditional home remedy D) believing in a shaman as a traditional healer.

B

Which nursing instruction is correct when a urine specimen is collected for culture and sensitivity testing from a patient without a urinary catheter? A) Tell the patient to void and pour the urine into a labeled specimen container. B) Ask the patient to void first into the toilet, stop midstream, and finish voiding into the sterile specimen container. C) Instruct the patient to discard the first void and collect the next void for the specimen. D) Have the patient keep all voided urine for 24 hours in a chilled, opaque collection container.

B

Which of the following is an example of subjective data? A) Blood glucose 126 md/dL B) Pain rated at 7 out of 10 C) Heart rate of 76 bpm D) Bruising on lower leg

B

Which of the following is the most reliable indicator for chronic pain? A) blood drug levels B) patient self report C) Magnetic resonance imaging (MRI) results D) Tissue enzyme levels

B

Which of these actions illustrates the correct technique the nurse should use when assessing oral temperature with a mercury thermometer? A) Wait 30 minutes if the patient has ingested hot or iced liquids. B) Leave the thermometer in place 3 to 4 minutes if the patient is afebrile. C) Place the thermometer in front of the tongue, and ask the patient to close his or her lips. D) Shake the mercury-in-glass thermometer down to below 36.6 C before taking the temperature.

B

Which of these assessment findings would the nurse expect to see when examining the eyes of a black patient? A) Increased night vision B) Dark retinal background C) Increased photosensitivity D) Narrowed palpebral fissures

B

Which of these statements concerning areas of the brain is true? A) The cerebellum is the center for speech and emotions. B) The hypothalamus controls body temperature and regulates sleep. C) The basal ganglia are responsible for controlling voluntary movements. D) Motor pathways of the spinal cord and brainstem synapse in the thalamus.

B

Which of these statements is true regarding the arterial system? A) Arteries are large-diameter vessels. B) The arterial system is a high-pressure system. C) The walls of arteries are thinner than those of the veins. D) Arteries can greatly expand to accommodate a large blood volume increase.

B

Which statement about apices of the lungs is true? The apices of the lungs: A) Are at the level of the second rib anteriorly B) Extend 3 to 4 cm above the inner third of the clavicles. C) Are located at the sixth rib anteriorly and the eighth rib laterally D) Rest on the diaphragm at the fifth intercostal space in the midclavicular line (MCL).

B

Which statement best illustrates the difference between religion and spirituality? A) religion reflects an individual's reaction to life events whereas spirituality is based on whether the individual attends religious services. B) religion is characterized by identification of a higher being shaping one's destiny, whereas spirituality reflects an individual's perception of one's life having worth or meaning. D) religion is the active interpretation of one's spirituality.

B

Which statement is true in regards to pain? A) Nurse's attitudes toward their patients' pain are unrelated to their own experiences with pain B) The cultural background of a patient is important in a nurse's assessment of that patient's pain C) A nurse's area of clinical practice is most likely to determine his or her assessment of a patient's pain D) A nurse's years of clinical experience and current position are a strong indicator of his or her response to patient pain.

B

Which statement is true regarding the complete physical assessment? A) The male genitalia should be examined in the supine position. B) The patient should be in the sitting position for examination of the had and neck. C) The vital signs, height, and weight should be obtained at the end of the examination. D) To promote consistency between patients, the examiner should not vary the order of the assessment.

B

Which theory has been expanded in an attempt to study the degree to which a person's lifestyle reflects his or her traditional heritage? A) Congruence mechanism B) Heritage consistency C) Behavior theory D) Socialization experience

B

While assessing the carotid arteries of an older adult, the nurse would need further instruction if they: A) use the bell of the stethoscope to asses for murmurs and bruits B) document the normal pulses as 4+ bilaterally bi C) assess each carotid pulse individually D) use a doppler to assess 2+ pulses

B

While auscultating heart sounds on a 7-year-old child for a routine physical examination, the nurse hears an S3, a soft murmur at the left midsternal border, and a venous hum when the child is standing. What would be a correct interpretation of these findings? A) S3 is indicative of heart disease in children. B) These findings can all be normal in a child. C) These findings are indicative of congenital problems. D) The venous hum most likely indicates an aneurysm.

B

While evaluating the health history, the nurse determines that the patient subscribes to the hot/cold theory of health. Which of the following would most likely describe this patient's view of wellness? A) The phlegm will be replaced with dryness. B) The humors must be balanced. C) Good is hot. D) Evil is hot.

B

While examining the broken arm of a 4 year old boy, select the appropriate assessment tool to evaluate his pain status. A) 0 to 10 numeric rating scale B) Wong-Baker scale C) simple descriptor scale D) 0 to 5 numeric rating scale

B

While obtaining a health history, a patient tells the nurse that he has frequent nosebleeds and asks the best way to get them to stop. What would be the nurse's best response? A) "While sitting up, place a cold compress over your nose." B) "Sit up with your head tilted forward and pinch your nose." C) "Allow the bleeding to stop on its own, but don't blow your nose." D) "Lie on your back with your head tilted back and pinch your nose."

B

While performing an assessment of the mouth, the nurse notices that the patient has a 1-cm ulceration that is crusted with an elevated border and located on the outer third of the lower lip. What other information would be most important for the nurse to assess? A) Nutritional status B) When the patient first noticed the lesion C) Whether the patient has had a recent cold D) Whether the patient has had any recent exposure to sick animals

B

While performing the otoscopic examination of a 3-year-old boy who has been pulling on his left ear, the nurse finds that his left tympanic membrane is bright red and that the light reflex is not visible. The nurse interprets these findings to indicate a(n): A) Fungal infection. B) Acute otitis media. C) Perforation of the eardrum. D) Cholesteatoma.

B

While taking a history, the patient describes a burning, painful sensation that moves around the toes and bottoms of the feet. These symptoms suggest: A) Nociceptive pain B) Neuropathic Pain C) Visceral Pain D) Muscular Pain

B

While taking a history, the patient describes a burning, painful sensation that moves around the toes and bottoms of the feet. These symptoms suggest: A) nociceptive pain B) neuropathic pain C) visceral pain D) muscular pain

B

With which of these patients would it be most appropriate for the nurse to use games during the assessment, such as having the patient blow out the light on the penlight? A) Infant B) Preschool child C) School-age child D) Adolescent

B

a patient presents with acute pain of the abdomen. After the initial examination, how would you proceed? A) withhold analgesic until diagnostic testing is completed B) give pain medications as ordered C) withhold analgesic until pain subsides D) determine what type of pain it is and proceed accordingly

B

during the assessment of a healthy thirty year old patient, the nurse asks him to reflex his muscles completely. the nurse then moves each extremity through full ROM. which of these are expected findings? A) firm, rigid resistance to movement B) mild, even resistance to movement C) slight pain with some directions of movement D) hypotonic muscles as a result of total relaxation

B

pain issues should be anticipated in a cognitively impaired older adult with a history of: A) constipation B) peripheral vascular disease C) COPD D) parkinson disease

B

the nurse is assessing a patient for neurologic impairment after a total hip replacement. Which finding indicates impairment in the affected extremity? A) decreased distal pulse B) inability to move C) decreased capillary refill D) coolness on palpation

B

the nurse knows that during an abdominal assessment, deep palpation is used to determine: A) bowel motility B) enlarged organs C) superficial tenderness D) overall impression of skin surface and superficial musculature

B

the patient admitted for acute pancreatitis has an elevated white blood cell count, elevated amylase and lipase levels, and is complaining of 10/10 abdominal pain. On assessment, the nurse notes ecchymosis in the periumbilical area, and correctly documents it as which of the following? A) gray turner sign B) cullen's sign C) Brudizinki sign D) battle sign

B

the student nurse notes jugular vein distention in the 75 year old patient. Jugular vein distention is an indicator of which of the following? A) conduction efficiency B) the heart's ability as a pump C) heart rate and rhythm D) extent of aortic failure

B

which group is the greatest risk for developing Reynaud's phenomenon? A) young men B) young women C) old men D) old women

B

which of the following is not a cause of abdominal distention? A) ascites B) pyrosis C) obesity D) flatulence E) peritonitis

B

which of the following screening tools is used to assess a patient's risk for developing a pressure ulcer? A) Rowling scale B) Braden scale C) brandon scale D) pressure injury assessment scale

B

The nurse is assessing the joints of a woman who has stated, "I have a long family history of arthritis, and my joints hurt." The nurse suspects that she has osteoarthritis. Which of these are symptoms of osteoarthritis? Select all that apply. A) symmetric joint involvement B) asymmetric joint involvement C) pain with motion of affected joints D) affected joints are swollen with hard, bony protuberances E) affected joint may have heat, redness, and swelling

B, C, D

During an admission assessment of a patient with dementia, the nurse assesses for pain because the patient has recently had several falls. Which of these are appropriate for the nurse to assess in a patient with dementia? Select all that apply. A) Ask the patient, Do you have pain? B) Assess the patients breathing independent of vocalization. C) Note whether the patient is calling out, groaning, or crying. D) Have the patient rate pain on a 1-to-10 scale. E) Observe the patients body language for pacing and agitation.

B, C, E

Which of the following actions/behaviors in the critical-thinking process are important for the novice nurse to remember? (Select all that apply.) A) disregard initial cues B) Approach assessment with a nonjudgmental attitude C) Cluster associated assessment data D) Perform assessment in whatever manner works for you E) Avoid making assumptions

B, C, E

when assessing a patient with multiple sclerosis for potential complications, the nurse should assess for which of the following? Select all that apply A) dehydration B) falls C) skin breakdown D) seizures E) fatigue

B, C, E

The nurse is presenting a class on risk factors for cardiovascular disease. Which of these are considered modifiable risk factors for MI? Select all that apply. A) Ethnicity B) Abnormal lipids C) Smoking D) Gender E) Hypertension F) Diabetes G) Family history

B, C, E, F

The nurse is assessing the abilities of an older adult. Which of these following activities are considered instrumental activities of daily living? Select all that apply. A) Feeding oneself B) Preparing a meal C) Balancing a checkbook D) Walking E) Toileting F) Grocery shopping

B, C, F

The nurse suspects that a patient has appendicitis. Which of these procedures are appropriate for use when assessing for appendicitis or a perforated appendix? Select all that apply. A) Test for the Murphy sign B) Test for the Blumberg sign C) Test for shifting dullness D) Perform the iliopsoas muscle test E) Test for fluid wave

B, D

During an examination, a patient states that she was diagnosed with open-angle glaucoma 2 years ago. The nurse assesses for characteristics of open-angle glaucoma. Which of these are characteristics of open-angle glaucoma? Select all that apply. A) Patient may experience sensitivity to light, nausea, and halos around lights. B) Patient experiences tunnel vision in the late stages. C) Immediate treatment is needed. D) Vision loss begins with peripheral vision. E) Open-angle glaucoma causes sudden attacks of increased pressure that cause blurred vision. F) Virtually no symptoms are exhibited.

B, D, F

The nurse is assessing a 1-month-old infant at his well-baby checkup. Which assessment findings are appropriate for this age? Select all that apply. A) Head circumference equal to chest circumference B) Head circumference greater than chest circumference C) Head circumference less than chest circumference D) Fontanels firm and slightly concave E) Absent tonic neck reflex F) Nonpalpable cervical lymph nodes

B, D, F

The nurse is testing the hearing of a 78-year-old man and is reminded of the changes in hearing that occur with aging that include which of the following? Select all that apply. A) Hearing loss related to aging begins in the mid 40s. B) Progression of hearing loss is slow. C) The aging person has low-frequency tone loss. D) The aging person may find it harder to hear consonants than vowels. E) Sounds may be garbled and difficult to localize. F) Hearing loss reflects nerve degeneration of the middle ear.

B,D,E

A 69-year-old patient has been admitted to an adult psychiatric unit because his wife thinks he is getting more and more confused. He laughs when he is found to be forgetful, saying "I'm just getting old!" After the nurse completes a thorough neurologic assessment, which findings would be indicative of Alzheimer's disease? Select all that apply. A) Occasionally forgetting names or appointments B) Difficulty performing familiar tasks, such as placing a telephone call C) Misplacing items, such as putting dish soap in the refrigerator D) Sometimes having trouble finding the right word E) Rapid mood swings, from calm to tears, for no apparent reason F) Getting lost in one's own neighborhood

B. C, E, F

A 14-year-old boy who has been diagnosed with Osgood-Schlatter disease reports painful swelling just below the knee for the past 5 months. Which response by the nurse is appropriate? A) "If these symptoms persist, you may need arthroscopic surgery." B) "You are experiencing degeneration of your knee, which may not resolve." C) "Your disease is due to repeated stress on the patellar tendon. It is usually self-limited, and your symptoms should resolve with rest." D) "Increasing your activity and performing knee-strengthening exercises will help to decrease the inflammation and maintain mobility in the knee."

C

A 17-year-old single mother is describing how difficult it is to raise a 3-year-old child by herself. During the course of the interview she states, I cant believe my boyfriend left me to do this by myself! What a terrible thing to do to me! Which of these responses by the nurse uses empathy? A) You feel alone. B) You cant believe he left you alone? C) It must be so hard to face this all alone. D) I would be angry, too; raising a child alone is no picnic.

C

A 2-year-old child has been brought to the clinic for a well-child checkup. The best way for the nurse to begin the assessment is to: A) Ask the parent to place the child on the examining table. B) Have the parent remove all of the childs clothing before the examination. C) Allow the child to keep a security object such as a toy or blanket during the examination. D) Initially focus the interactions on the child, essentially ignoring the parent until the childs trust has been obtained.

C

A 30 year old woman reports having persistent intense pain in her right arm related to trauma sustained from a car accident 5 months ago. She states that the slightest touch or clothing can exacerbate the pain. This report is suggestive of: A) Referred Pain B) Psychogenic Pain C) Complex Regional Pain I D) Cutaneous pain

C

A 30-year old woman reports having persistent intense pain in her right arm related to trauma sustained from a car accident 5 months ago. She states that the slightest touch or clothing can exacerbate the pain. This report is suggestive of: A) referred pain B) psychogenic pain C) complex regional pain I D) cutaneous pain

C

A 30-year-old woman tells the nurse that she has been very unsteady and has had difficulty in maintaining her balance. Which area of the brain would the nurse be concerned about with these findings? A) Thalamus B) Brainstem C) Cerebellum D) Extrapyramidal tract

C

A 35-year-old pregnant woman comes to the clinic for a monthly appointment. During the assessment, the nurse notices that she has a brown patch of hyperpigmentation on her face. The nurse continues the skin assessment aware that another finding may be: A) Keratoses. B) Xerosis. C) Chloasma. D) Acrochordons.

C

A 35-year-old recent immigrant is being seen in the clinic for complaints of a cough that is associated with rust-colored sputum, low-grade afternoon fevers, and night sweats for the past 2 months. The nurses preliminary analysis, based on this history, is that this patient may be suffering from: A) Bronchitis. B) Pneumonia. C) Tuberculosis. D) Pulmonary edema.

C

A 42-year-old female patient complains that she has noticed several small, slightly raised, bright red dots on her chest. On examination, the nurse expects that the spots are probably: A) Anasarca. B) Scleroderma. C) Senile angiomas. D) Latent myeloma.

C

A 45-year-old man is in the clinic for a routine physical examination. During the recording of his health history, the patient states that he has been having difficulty sleeping. Ill be sleeping great, and then I wake up and feel like I cant get my breath. The nurses best response to this would be: A) When was your last electrocardiogram? B) Its probably because its been so hot at night. C) Do you have any history of problems with your heart? D) Have you had a recent sinus infection or upper respiratory infection?

C

A 60-year-old male patient has been treated for pneumonia for the past 6 weeks. He is seen today in the clinic for an unexplained weight loss of 10 pounds over the last 6 weeks. The nurse knows that: A) Weight loss is probably the result of unhealthy eating habits. B) Chronic diseases such as hypertension cause weight loss. C) Unexplained weight loss often accompanies short-term illnesses. D) Weight loss is probably the result of a mental health dysfunction.

C

A 60-year-old man is at the clinic for an eye examination. The nurse suspects that he has ptosis of one eye. How should the nurse check for this? A) Perform the confrontation test. B) Assess the individual's near vision. C) Observe the distance between the palpebral fissures. D) Perform the corneal light test, and look for symmetry of the light reflex.

C

A 65-year-old patient is experiencing pain in his left calf when he exercises that disappears after resting for a few minutes. The nurse recognizes that this description is most consistent with _________ the left leg. A) Venous obstruction of B) Claudication due to venous abnormalities in C) Ischemia caused by a partial blockage of an artery supplying D) Ischemia caused by the complete blockage of an artery supplying

C

A 65-year-old patient with a history of heart failure comes to the clinic with complaints of being awakened from sleep with shortness of breath. Which action by the nurse is most appropriate? A) Obtaining a detailed health history of the patients allergies and a history of asthma B) Telling the patient to sleep on his or her right side to facilitate ease of respirations C) Assessing for other signs and symptoms of paroxysmal nocturnal dyspnea D) Assuring the patient that paroxysmal nocturnal dyspnea is normal and will probably resolve within the next week

C

A 68-year-old woman has come in for an assessment of her rheumatoid arthritis, and the nurse notices raised, firm, nontender nodules at the olecranon bursa and along the ulna. These nodules are most commonly diagnosed as: A) epicondylitis. B) gouty arthritis. C) olecranon bursitis. D) subcutaneous nodules.

C

A 70-year-old patient tells the nurse that he has noticed that he is having trouble hearing, especially in large groups. He says that he "can't always tell where the sound is coming from" and the words often sound "mixed up." What might the nurse suspect as the cause for this change? A) Atrophy of the apocrine glands B) Cilia becoming coarse and stiff C) Nerve degeneration in the inner ear D) Scarring of the tympanic membrane

C

A 70-year-old woman who loves to garden has small, flat, brown macules over her arms and hands. She asks, "What causes these liver spots?" The nurse tells her, "They are: A) Signs of decreased hematocrit related to anemia. B) Due to the destruction of melanin in your skin from exposure to the sun. C) Clusters of melanocytes that appear after extensive sun exposure. D) Areas of hyperpigmentation related to decreased perfusion and vasoconstriction.

C

A black patient is in the intensive care unit because of impending shock after an accident. The nurse would expect to find what characteristics in this patient's skin? A) Ruddy blue. B) Generalized pallor. C) Ashen, gray, or dull. D) Patchy areas of pallor.

C

A man who was found wandering in a park at 2 AM has been brought to the emergency department for an examination because he said he fell and hit his head. During the examination, the nurse asks him to use his index finger to touch the nurse's finger, then his own nose, then the nurse's finger again (which has been moved to a different location). The patient is clumsy, unable to follow the instructions, and overshoots the mark, missing the finger. The nurse should suspect which of the following? A) Cerebral injury B) Cerebrovascular accident C) Acute alcohol intoxication D)Peripheral neuropathy

C

A mother asks when her newborn infant's eyesight will be developed. The nurse should reply: A) "Vision is not totally developed until 2 years of age." B) "Infants develop the ability to focus on an object at approximately 8 months of age." C) "By approximately 3 months of age, infants develop more coordinated eye movements and can fixate on an object." D) "Most infants have uncoordinated eye movements for the first year of life."

C

A mother brings her newborn baby boy in for a checkup; she tells the nurse that he doesn't seem to be moving his right arm as much as his left and that he seems to have pain when she lifts him up under the arms. The nurse suspects a fractured clavicle and would observe for: A) a negative Allis test. B) a positive Ortolani's sign. C) limited range of motion during the Moro's reflex. D) limited range of motion during Lasègue's test

C

A mother brings in her newborn infant for an assessment and tells the nurse that she has noticed that whenever her newborns head is turned to the right side, she straightens out the arm and leg on the same side and flexes the opposite arm and leg. After observing this on examination, the nurse tells her that this reflex is: A) Abnormal and is called the atonic neck reflex. B) Normal and should disappear by the first year of life. C) Normal and is called the tonic neck reflex, which should disappear between 3 and 4 months of age. D) Abnormal. The baby should be flexing the arm and leg on the right side of his body when the head is turned to the right.

C

A newborn infant is in the clinic for a well-baby check. The nurse observes the infant for the possibility of fluid loss because of which of these factors? A) Subcutaneous fat deposits are high in the newborn. B) Sebaceous glands are over productive in the newborn. C) The newborns skin is more permeable than that of the adult. D) The amount of vernix caseosa dramatically rises in the newborn.

C

A nurse is taking complete health histories on all of the patients attending a wellness workshop. On the history form, one of the written questions asks, You dont smoke, drink, or take drugs, do you? This question is an example of: A) Talking too much. B) Using confrontation. C) Using biased or leading questions. D) Using blunt language to deal with distasteful topics.

C

A nurse precepting a student nurse asks, "What's the most important step in the critical-thinking process?" A) clustering subjective and objective data B) analyzing health data C) using evidence based assessment techniques D) prioritizing health concerns

C

A patient comes in for a physical, and she complains of "freezing to death" while waiting for her examination. The nurse notes that her skin is pale and cool and attributes this finding to: A) Venous pooling. B) Peripheral vasodilation. C) Peripheral vasoconstriction. D) Decreased arterial perfusion.

C

A patient comes to the clinic and tells the nurse that he has been confined to his recliner chair for about 3 days with his feet down and he wants the nurse to evaluate his feet. During the assessment, the nurse might expect to find: A) Pallor B) Coolness C) Distended veins D) Prolonged capillary filling time

C

A patient has a severed spinal nerve as a result of trauma. Which of these statements is true in this situation? A) Because there are 31 pairs of spinal nerves, no effect results if only one nerve is severed. B) The dermatome served by this nerve will no longer experience any sensation. C) The adjacent spinal nerves will continue to carry sensations for the dermatome served by the severed nerve. D) A severed spinal nerve will only affect motor function of the patient because spinal nerves have no sensory component

C

A patient has been admitted after an accident at work. During the assessment, the patient is having trouble hearing and states, "I don't know what the matter is. All of a sudden, I can't hear you out of my left ear!" What should the nurse do next? A) Make note of this finding for the report to the next shift. B) Prepare to remove cerumen from the patient's ear. C) Notify the patient's health care provider. D) Irrigate the ear with rubbing alcohol.

C

A patient has been admitted for severe psoriasis. The nurse can expect to see what finding in the patient's fingernails? A) Splinter hemorrhages B) Paronychia C) Pitting D) Beau lines

C

A patient has been admitted to the emergency department for a suspected drug overdose. His respirations are shallow, with an irregular pattern, with a rate of 12 respirations per minute. The nurse interprets this respiration pattern as which of the following? A) Bradypnea B) Cheyne-Stokes respirations C) Hypoventilation D) Chronic obstructive breathing

C

A patient has been admitted to the emergency department with a possible medical diagnosis of pulmonary embolism. The nurse expects to see which assessment findings related to this condition? A) Absent or decreased breath sounds B) Productive cough with thin, frothy sputum C) Chest pain that is worse on deep inspiration and dyspnea D) Diffuse infiltrates with areas of dullness upon percussion

C

A patient has been shown to have a sensorineural hearing loss. During the assessment, it would be important for the nurse to: A) Speak loudly so the patient can hear the questions. B) Assess for middle ear infection as a possible cause. C) Ask the patient what medications he is currently taking.. D) Look for the source of the obstruction in the external ear.

C

A patient has recently received health insurance and would like to know how often he should visit the provider. How do you respond? A) "It would be most efficient if you visit on an annual basis" B) "There is no recommendation for the frequency of health care visits." C) "Your visits may vary, depending on your level of wellness" D) "Your visits will be based on your preference"

C

A patient is being assessed for range of joint movement. The nurse asks him to move his arm in toward the center of his body. This movement is called: A) flexion. B) abduction. C) adduction. D) extension.

C

A patient is complaining of a sharp pain along the costovertebral angles. The nurse is aware that this symptom is most often indicative of: A) Ovary infection. B) Liver enlargement. C) Kidney inflammation. D) Spleen enlargement.

C

A patient is complaining of pain in his joints that is worse in the morning, is better after he has moved around for awhile, and then gets worse again if he sits for long periods of time. The nurse should assess for other signs of what problem? A) Tendinitis B) Osteoarthritis C) Rheumatoid arthritis D) Intermittent claudication

C

A patient is especially worried about an area of skin on her feet that has turned white. The health care provider has told her that her condition is vitiligo. The nurse explains to her that vitiligo is: A) Caused by an excess of melanin pigment B) Caused by an excess of apocrine glands in her feet C) Caused by the complete absence of melanin pigment D) Related to impetigo and can be treated with an ointment

C

A patient is having difficulty swallowing medications and food. The nurse would document that this patient has: A) Aphasia. B) .Dysphasia. C) Dysphagia D) Anorexia

C

A patient is not able to perform rapid alternating movements such as patting her knees rapidly. The nurse should document this as: A) Ataxia. B) Astereognosis. C) Presence of dysdiadochokinesia. D) Loss of kinesthesia.

C

A patient is unable to shrug her shoulders against the nurses resistant hands. What cranial nerve is involved with successful shoulder shrugging? A) VII B) IX C) XI D) XII

C

A patient is visiting the clinic for an evaluation of a swollen, painful knuckle. The nurse notices that the knuckle above his ring on the left hand is swollen and that he is unable to remove his wedding ring. This joint is called the _____ joint. A) interphalangeal B) tarsometatarsal C) metacarpophalangeal D) tibiotalar

C

A patient says she is very nervous and nauseated, and she feels as if she will vomit. This data would be what type of data? A) objective B) reflective C) subjective D) introspective

C

A patient states that the pain medication is not working and rates his postoperative pain at a 10 on a 1-to-10 scale. Which of these assessment findings indicates an acute pain response to poorly controlled pain? A)Confusion B) Hyperventilation C) Increased blood pressure and pulse D) Depression

C

A patient tells the nurse, Sometimes I wake up at night and I have real trouble breathing. I have to sit up in bed to get a good breath. When documenting this information, the nurse would note: A) Orthopnea B) Acute emphysema. C) Paroxysmal nocturnal dyspnea. D) Acute shortness of breath episode.

C

A patient who has had rheumatoid arthritis for years comes to the clinic to ask about changes in her fingers. The nurse will assess for signs of what problems? A) Heberden's nodes B) Bouchard's nodules C) Swan neck deformities D) Dupuytren's contractures

C

A patient with a middle ear infection asks the nurse, "What does the middle ear do?" The nurse responds by telling the patient that the middle ear functions to: A) Maintain balance. B) Interpret sounds as they enter the ear. C) Conduct vibrations of sounds to the inner ear. D) Increase amplitude of sound for the inner ear to function.

C

A patient with a spinal cord injury who has been active in sports and outdoor activities talks almost obsessively about his past activities. in tears, one day he asks the nurse "why am I unable to stop talking about these things? I know those days are over." which response by the nurse conveys the best understanding of the patient's behavior? A) be patient. it takes time to adjust to such a massive loss B) talking about the past is a form of denial. we have to help you focus on today C) reviewing your losses is a way to help you work through your grief and loss D) it is an escape mechanism to go back and live again in happier times

C

A patient with lack of oxygen to his heart will have pain in his chest and possibly the shoulder, arms, or jaw. The nurse knows that the statement that best explains why this occurs is which of these? A) A problem exists with the sensory cortex and its ability to discriminate the location. B) The lack of oxygen in his heart has resulted in decreased amount of oxygen to the areas experiencing the pain. C) The sensory cortex does not have the ability to localize pain in the heart; consequently, the pain is felt elsewhere. D) A lesion has developed in the dorsal root, which is preventing the sensation from being transmitted normally.

C

A patient, an 85-year-old woman, is complaining about the fact that the bones in her face have become more noticeable. What explanation should the nurse give her? A) Diets low in protein and high in carbohydrates may cause enhanced facial bones. B) Bones can become more noticeable if the person does not use a dermatologically approved moisturizer. C) More noticeable facial bones are probably due to a combination of factors related to aging, such as decreased elasticity, subcutaneous fat, and moisture in her skin D) Facial skin becomes more elastic with age. This increased elasticity causes the skin to be more taught, drawing attention to the facial bones.

C

A patients blood pressure is 118/82 mm Hg. He asks the nurse, What do the numbers mean? The nurses best reply is: A) The numbers are within the normal range and are nothing to worry about. B) The bottom number is the diastolic pressure and reflects the stroke volume of the heart. C) The top number is the systolic blood pressure and reflects the pressure of the blood against the arteries when the heart contracts. D) The concept of blood pressure is difficult to understand. The primary thing to be concerned about is the top number, or the systolic blood pressure.

C

A patients thyroid gland is enlarged, and the nurse is preparing to auscultate the thyroid gland for the presence of a bruit. A bruit is a __________ sound that is heard best with the __________ of the stethoscope. A) Low gurgling; diaphragm B) Loud, whooshing, blowing; bell C) Soft, whooshing, pulsatile; bell D) High-pitched tinkling; diaphragm

C

A physician has diagnosed a patient with purpura. After leaving the room, a nursing student asks the nurse what the physician saw that led to that diagnosis. The nurse should say, "The physician is referring to: A) Blue dilation of blood vessels in a star-shaped linear pattern on the legs. B) Fiery red, star-shaped marking on the cheek that has a solid circular center. C) Confluent and extensive patch of petechiae and ecchymoses on the feet. D) Tiny areas of hemorrhage that are less than 2 mm, round, discrete, and dark red in color.

C

A physician tells the nurse that a patients vertebra prominens is tender and asks the nurse to reevaluate the area in 1 hour. The area of the body the nurse will assess is: A) Just above the diaphragm. B) Just lateral to the knee cap. C) At the level of the C7 vertebra. D) At the level of the T11 vertebra.

C

A teenage girl has arrived complaining of pain in her left wrist. She was playing basketball when she fell and landed on her left hand. The nurse examines her hand and would expect a fracture if the girl complains: A) of a dull ache. B) that the pain in her wrist is deep. C) of sharp pain that increases with movement. D) of dull throbbing pain that increases with rest.

C

A visitor from Poland who does not speak English seems to be somewhat apprehensive about the nurse examining his neck. He would probably be more comfortable with the nurse examining his thyroid gland from: A) Behind with the nurses hands placed firmly around his neck. B) The side with the nurses eyes averted toward the ceiling and thumbs on his neck. C) The front with the nurses thumbs placed on either side of his trachea and his head tilted forward. D) The front with the nurses thumbs placed on either side of his trachea and his head tilted backward.

C

A woman comes to the clinic and states, Ive been sick for so long! My eyes have gotten so puffy, and my eyebrows and hair have become coarse and dry. The nurse will assess for other signs and symptoms of: A) Cachexia. B) Parkinson syndrome. C) Myxedema. D) Scleroderma

C

A woman in her 26th week of pregnancy states that she is not really short of breath but feels that she is aware of her breathing and the need to breathe. What is the nurses best reply? A) The diaphragm becomes fixed during pregnancy, making it difficult to take in a deep breath. B) The increase in estrogen levels during pregnancy often causes a decrease in the diameter of the rib cage and makes it difficult to breathe. C) What you are experiencing is normal. Some women may interpret this as shortness of breath, but it is a normal finding and nothing is wrong. D) This increased awareness of the need to breathe normal as the fetus grows because of the increased oxygen demand on the mothers body, which results in an increased respiratory rate.

C

An aneurysm is: A) a fatty plaque deposited in the intima of the arteries. B) a thickening and loss of elasticity of the arterial walls. C) a sac formed by dilation in the arterial wall. D) a variation from the heart's normal rhythm.

C

An example of objective data is A) a report of impaired mobility from left knee pain as evidenced by an inability to walk, swelling, and pain on passive range of motion. B) a complaint of left knee pain. C) crepitation in the left knee joint. D) left knee has been swollen and hot for the past 3 days.

C

An unlicensed assisitve personnel (UAP) is providing care to a patient with left-side paralysis. Which action by the UAP requires the nurse to provide further instruction A) providing a passive range of motion exercises to the left extremities during bed bath B) elevating the head of the bed to reduce swelling and edema C) pulling the patient up by the left shoulder to get out of bed D) putting high top tennis shoes on the patient after bathing

C

At the beginning of rounds, when the nurse enters the room, what should the nurse do first? A) Check the intravenous infusion site for swelling or redness. B) Check the infusion pump settings for accuracy. C) Make eye contact with the patient and introduce himself or herself as the patient's nurse. D) Offer the patient something to drink

C

At which phase does the individual become aware of a painful sensation? A) Modulation B) Transduction C) Perception D) Transmission

C

At which phase does the individual become aware of a painful sensation? A) modulation B) transduction C) perception D) transmission

C

Atherosclerosis is defined as: A) a swooshing sound heard through a stethoscope when an artery is partially occluded. B) a thickening and loss of elasticity of the arterial walls. C) plaques of fatty deposits forming in the intima of the arteries D) .a sac formed by dilation in the arterial wall.

C

During a cardiovascular assessment, the nurse knows that an S4 heart sound is: A) Heard at the onset of atrial diastole. B) Usually a normal finding in the older adult. C) Heard at the end of ventricular diastole. D) Heard best over the second left intercostal space with the individual sitting upright.

C

During a health history, the patient tells the nurse, I have pain all the time in my stomach. Its worse 2 hours after I eat, but it gets better if I eat again! Based on these symptoms, the nurse suspects that the patient has which condition? A) Appendicitis B) Gastric ulcer C) Duodenal ulcer D) Cholecystitis

C

During a morning assessment, the nurse notices that a patient's urine output is below the expected amount. What should the nurse do next? A) Obtain an order for a Foley catheter. B) Obtain an order for a straight catheter. C) Perform a bladder scan test. D) Refer the patient to a urologist.

C

During a morning assessment, the nurse notices that an older patient is less attentive and is unable to recall yesterday's events. Which test is appropriate for assessing the patient's mental status? A) Geriatric Depression Scale, Short Form B) The Physical Performance Test C) Mini-Cog D) The Get Up and Go Test

C

During a neonatal examination, the nurse notices that the newborn infant has six toes. This finding is documented as: A) unidactyly. B) syndactyly. C) polydactyly. D) multidactyly.

C

During a routine office visit, a patient takes off his shoes and shows the nurse "this awful sore that won't heal." On inspection, the nurse notes a 3-cm round ulcer on the left great toe, with a pale ischemic base, well-defined edges, and no drainage. The nurse should assess for other signs and symptoms of: A) Varicosities. B) Venous stasis ulcer. C) Arterial ischemic ulcer. D) Deep-vein thrombophlebitis.

C

During a well-baby checkup, the nurse notices that a 1-week-old infants face looks small compared with his cranium, which seems enlarged. On further examination, the nurse also notices dilated scalp veins and downcast or setting sun eyes. The nurse suspects which condition? A)Craniotabes B)Microcephaly C) Hydrocephalus D) Caput succedaneum

C

During an abdominal assessment, The nurse elicits tenderness on light palpation in the right lower quadrant. The nurse interprets that this finding could indicate a disorder of which of these structures? A) Spleen B) Sigmoid C) Appendix D) Gallbladder

C

During an admission assessment, the nurse notices that a male patient has an enlarged and rather thick skull. The nurse suspects acromegaly and would further assess for: A) Exophthalmos. B) Bowed long bones. C) Coarse facial features. D) Acorn-shaped cranium.

C

During an assessment of a 20-year-old Asian patient, the nurse notices that he has dry, flaky cerumen in his canal. What is the significance of this finding? This finding: A) Is probably the result of lesions from eczema in his ear. B) Represents poor hygiene. C) Is a normal finding, and no further follow-up is necessary. D) Could be indicative of change in cilia; the nurse should assess for hearing loss

C

During an assessment of a 26-year-old for "a spot on my lip I think is cancer," the clinic nurse notices a group of clear vesicles with an erythematous base around them located at the lip-skin border. The patient mentions that she just returned from Hawaii. What is the most appropriate action by the nurse? A) Tell the patient she needs to see a skin specialist. B) Discuss the benefits of having a biopsy performed on any unusual lesion. C) Tell the patient that these vesicles are indicative of herpes simplex I or cold sores and that they will heal in 4 to 10 days. D) Tell the patient that these vesicles are most likely the result of a riboflavin deficiency and discuss nutrition.

C

During an assessment of a 80 year old patient, the nurse notices the following: inability to identify vibrations at her ankle, a slower and more deliberate gait, and a slightly impaired tactile sensation. All other neurologic findings are within normal limited (WNL), How should the nurse interpret these findings? A) CNS dysfunction B) lesion in the cerebral cortex C) normal changes attributed to aging D) demyelination of nerves attributable to a lesion

C

During an assessment of an 80-year-old patient, the nurse notices the following: inability to identify vibrations at the ankle and to identify position of big toe, slower and more deliberate gait, and slightly impaired tactile sensation. All other neurologic findings are normal. The nurse should interpret that these findings indicate: A) CN dysfunction. B) Lesion in the cerebral cortex. C) Normal changes attributable to aging. D) Demyelination of nerves attributable to a lesion.

C

During an assessment of an older adult, the nurse should expect to notice which finding as a normal physiologic change associated with the aging process? A) Hormonal changes causing vasodilation and a resulting drop in blood pressure B) Progressive atrophy of the intramuscular calf veins, causing venous insufficiency C) Peripheral blood vessels growing more rigid with age, producing a rise in systolic blood pressure D) Narrowing of the inferior vena cava, causing low blood flow and increases in venous pressure resulting in varicosities

C

During an assessment, the nurse knows that expected assessment findings in the normal adult lung include the presence of A) Adventitious sounds and limited chest expansion. B) Increased tactile fremitus and dull percussion tones. C) Muffled voice sounds and symmetric tactile fremitus. D) Absent voice sounds and hyperresonant percussion tones.

C

During an assessment, the nurse notices that a patients umbilicus is enlarged and everted. It is positioned midline with no change in skin color. The nurse recognizes that the patient may have which condition? A) Intra-abdominal bleeding B) Constipation C) Umbilical hernia D) Abdominal tumor

C

During an assessment, the nurse notices that an older adult patient has tears rolling down his face from his left eye. Closer examination shows that the lower lid is loose and rolling outward. The patient complains of his eye feeling "dry and itchy." Which action by the nurse is correct? A) Assessing the eye for a possible foreign body B) Documenting the finding as ptosis C) Assessing for other signs of ectropion D) Contacting the prescriber; these are signs of basal cell carcinoma

C

During an examination of a female patient, the nurse notes lymphadenopathy and suspects an acute infection. Acutely infected lymph nodes would be: A) Clumped. B) Unilateral. C) Firm but freely movable. D) Firm and nontender

C

During an examination of a patient in her third trimester of pregnancy, the nurse notices that the patients thyroid gland is slightly enlarged. No enlargement had been previously noticed. The nurse suspects that the patient: A) Has an iodine deficiency. B) Is exhibiting early signs of goiter. C) Is exhibiting a normal enlargement of the thyroid gland during pregnancy. D) Needs further testing for possible thyroid cancer.

C

During an examination, the nurse asks a patient to bend forward from the waist and notices that the patient has lateral tilting. When his leg is raised straight up, he complains of a pain going down his buttock into his leg. The nurse suspects: A) scoliosis. B) meniscus tear. C) herniated nucleus pulposus. D) spasm of paravertebral muscles.

C

During an examination, the nurse finds that a patients left temporal artery is tortuous and feels hardened and tender, compared with the right temporal artery. The nurse suspects which condition? A)Crepitation B) Mastoiditis C) Temporal arteritis D) Bell palsy

C

During an examination, the nurse notices that a patients legs turn white when they are raised above the patients head. The nurse should suspect: A) Lymphedema. B) Raynaud disease. C) Chronic arterial insufficiency. D) Chronic venous insufficiency.

C

During an examination, the patient states he is hearing a buzzing sound and says that it is "driving me crazy!" The nurse recognizes that this symptom indicates: A) Vertigo. B) Pruritus. C) Tinnitus. D) Cholesteatoma.

C

During an interview, a woman says, I have decided that I can no longer allow my children to live with their fathers violence, but I just cant seem to leave him. Using interpretation, the nurses best response would be: A)You are going to leave him? B) If you are afraid for your children, then why cant you leave? C) It sounds as if you might be afraid of how your husband will respond. D)It sounds as though you have made your decision. I think it is a good one.

C

During an interview, the nurse would expect that most of the interview will take place at what distance? A) Intimate zone B) Personal distance C) Social distance D) Public distance

C

During an oral assessment of a 30-year-old black patient, the nurse notices bluish lips and adark line along the gingival margin. What action would the nurse perform in response to thisfinding? A) Check the patient's Hb for anemia. B) Assess for other signs of insufficient oxygen supply. C) Proceed with the assessment, this appearance is a normal finding. D) Ask if he has been exposed to an excessive amount of carbon monoxide.

C

During auscultation of the lungs of an adult patient, the nurse notices the presence of bronchophony, The nurse should assess for signs of which condition? A) Airway obstruction B) Emphysema C) Pulmonary consolidation D) Asthma

C

During the evaluation phase of the nursing process, which action would be included? A) validating the nursing diagnosis B) establishing priorities related to patient care C) including the patient and family members D) establishing a timeline for planned outcomes

C

During the examination of a patient, the nurse notices that the patient has several small, flat macules on the posterior portion of her thorax. These macules are less than 1 cm wide. Another name for these macules is A) Warts. B) Bullae. C) Freckles. D) Papules.

C

During the examination, offering some brief teaching about the patients body or the examiners findings is often appropriate. Which one of these statements by the nurse is most appropriate? A) Your atrial dysrhythmias are under control. B) You have pitting edema and mild varicosities. C) Your pulse is 80 beats per minute, which is within the normal range. D) Im using my stethoscope to listen for any crackles, wheezes, or rubs.

C

During the physical examination, your patient is diaphoretic and pale and complains of a dull pain in the LUQ of the abdomen. This is what type of pain? A) cutaneous B) somatic C) visceral D) psychogenic

C

During the physical examination, your patient is diaphoretic and pale and complains of dull pain in the LUQ of the abdomen. This is what type of pain? A) Cutaneous Pain B) Somatic Pain C) Visceral Pain D) Psychogenic Pain

C

During the precordial assessment on an patient who is 8 months pregnant, the nurse palpates the apical impulse at the fourth left intercostal space lateral to the midclavicular line. This finding would indicate: A) Right ventricular hypertrophy. B) Increased volume and size of the heart as a result of pregnancy. C) Displacement of the heart from elevation of the diaphragm. D) Increased blood flow through the internal mammary artery.

C

Following a stroke, a patient has dysphagia and left-sided facial paralysis. Which feeding technique will be most helpful at this time A) encourage sipping diluted liquid meal supplements from a straw B) postion the patient with the head of the bed at 30 degrees C) offer solid foods from the unaffected side of the mouth D) feed the patient a soft diet from a spoon into the left side of the mouth

C

If the nurse records the results to the Hirschberg test, the nurse has: A) Tested the patellar reflex. B) Assessed for appendicitis. C) Tested the corneal light reflex. D) Assessed for thrombophlebitis.

C

In a patient who has anisocoria, the nurse would expect to observe: A) Dilated pupils. B) Excessive tearing. C) Pupils of unequal size. D) Uneven curvature of the lens.

C

In assessing a patients major risk factors for heart disease, which would the nurse want to include when taking a history? A) Family history, hypertension, stress, and age B) Personality type, high cholesterol, diabetes, and smoking C) Smoking, hypertension, obesity, diabetes, and high cholesterol D) Alcohol consumption, obesity, diabetes, stress, and high cholesterol

C

In pulsus bigeminus: A) there is a deficiency of oxygenated arterial blood to a body part. B) the rhythm is regular, but the force of the pulse varies with alternating beats. C) the rhythm is coupled-every other beat is premature. D) beats have weaker amplitude with respiratory inspiration and stronger amplitude with expiration.

C

In using the ophthalmoscope to assess a patient's eyes, the nurse notices a red glow in the patient's pupils. On the basis of this finding, the nurse would: A) Suspect that an opacity is present in the lens or cornea. B) Check the light source of the ophthalmoscope to verify that it is functioning. C) Consider the red glow a normal reflection of the ophthalmoscope light off the inner retina. D) Continue with the ophthalmoscopic examination, and refer the patient for further evaluation.

C

Mr. Duguay is a 68-year-old man who comes to the clinic for a routine health assessment. In the older adult: A) the peripheral vessels become less rigid B) .the number of lymph nodes increases. C) the lymphatic tissue decreases. D) the intramuscular calf veins shrink.

C

Mrs. Lukianchuk is a 65-year-old patient who presents to the ambulatory health centre with a complaint of bilateral foot pain. On examination, you note delayed venous filling. This occurs with: A) incompetent valves. B) anemia. C) arterial insufficiency. D) aortic valve stenosis.

C

Normal age related finding in the lower extremities of an 80 year old woman would be: A) crepitus B) joint swelling C) diminished strength bilaterally D) unilateral muscle atrophy

C

Normal age- related finding in the lower extremities of an 80-year-old woman would be: A) Crepitus B) Joint Swelling C) Diminished strength bilaterally D) Unilateral muscle atrophy

C

Receiving is a part of the communication process. Which receiver is most likely to misinterpret a message sent by a health care professional? A)Well-adjusted adolescent who came in for a sports physical B) Recovering alcoholic who came in for a basic physical examination C) Man whose wife has just been diagnosed with lung cancer D)Man with a hearing impairment who uses sign language to communicate and who has an interpreter with him

C

The area of the nervous system that is responsible for mediating reflexes is the: A) Medulla. B) Cerebellum. C) Spinal cord. D) Cerebral cortex.

C

The articulation of the mandible and the temporal bone is known as the: A) intervertebral foramen. B) condyle of the mandible. C) temporomandibular joint D) zygomatic arch of the temporal bone.

C

The cervical nodes drain the: A) upper arm and breast. B) external genitalia. C) head and neck. D) hand and lower arm.

C

The concept of health and healing has evolved in recent years. Which is the best description of health? A) Health is the absence of disease. B) Health is a dynamic process toward optimal functioning C) Health depends on an interaction of mind, body, and spirit within the environment. D) Health is the prevention of disease.

C

The health care provider prescribes a medication that is administered transdermally. The nurse understands what feature of the transdermal route? A) It is inhaled into the respiratory tract. B) It is dissolved inside the cheek. C) It is absorbed through the skin. D) It is inserted into the vaginal cavity.

C

The major factor contributing to the need for cultural care in nursing is: A) an increasing birth rate B) limited access to health care service C) demographic change D) a decreasing rate of immigration

C

The nurse auscultating the chest in an adult. Which technique is correct? A) Instructing the patient to take deep, rapid breaths B) Instructing the patient to breathe in and out through his or her nose C) Firmly holding the diaphragm of the stethoscope against the chest D) Lightly holding the bell of the stethoscope against the chest to avoid friction

C

The nurse educator is preparing an education module for the nursing staff on the epidermal layer of skin. Which of these statements would be included in the module? The epidermis is: A) Contains mostly fat cells. B) Consists mostly of keratin. C) Is replaced every 4 weeks. D) Contains sensory receptors.

C

The nurse has just recorded a positive iliopsoas test on a patient who has abdominal pain. This test is used to confirm a(n): A) Inflamed liver. B) Perforated spleen. C) Perforated appendix. D) Enlarged gallbladder

C

The nurse is administering a test that is timed over 15 minutes and assesses a patient's upper body fine motor and coarse motor activities, balance, mobility, coordination, and endurance. During this test, activities such as dressing and stair climbing are timed. Which test is described by these activities? A) The Get Up and Go Test B) The Performance Activities of Daily Living C) The Physical Performance Test D) Tinetti Gait and Balance Evaluation

C

The nurse is assessing a 1-week-old infant and is testing his muscle strength. The nurse lifts the infant with hands under the axillae and notices that the infant starts to "slip" between the hands. The nurse should: A) suspect a fractured clavicle. B) suspect that the infant may have a deformity of the spine. C) suspect that the infant may have weakness of the shoulder muscles. D) consider this a normal finding because the musculature of an infant this age is undeveloped.

C

The nurse is assessing a 3-year-old for "drainage from the nose." On assessment, a purulent drainage that has a very foul odor is noted from the left naris and no drainage is observed from the right naris. The child is afebrile with no other symptoms. What should the nurse do next? A) Refer to the physician for an antibiotic order. B) Have the mother bring the child back in 1 week. C) Perform an otoscopic examination of the left nares. D) Tell the mother that this drainage is normal for a child of this age.

C

The nurse is assessing a patient in the hospital who has received numerous antibiotics for a lung infection and notices that his tongue appears to be black and hairy. In response to his concern, what would the nurse say? A) "We will need to get a biopsy to determine the cause." B) "This is an overgrowth of hair and will go away in a few days." C) "Black, hairy tongue is a fungal infection caused by all the antibiotics you have received" D) "This is probably caused by the same bacteria you had in your lungs."

C

The nurse is assessing a patient with COPD. Which requires immediate intervention A) distant heart sounds B) diminished lung sounds C) inability to speak D) pursed-lip breathing

C

The nurse is assessing an 80-year-old male patient. Which assessment findings would be considered normal? A) Increase in body weight from his younger years B) Additional deposits of fat on the thighs and lower legs C) Presence of kyphosis and flexion in the knees and hips D) Change in overall body proportion, including a longer trunk and shorter extremities

C

The nurse is assessing an 80-year-old patient. Which of these findings would be expected forthis patient? A) Hypertrophy of the gums B) Increased production of saliva C) Decreased ability to identify odors D) Finer and less prominent nasal hair

C

The nurse is assessing color vision of a male child. Which statement is correct? The nurse should: A) Check color vision annually until the age of 18 years. B) Ask the child to identify the color of his or her clothing. C) Test for color vision once between the ages of 4 and 8 years. D) Begin color vision screening at the child's 2-year checkup.

C

The nurse is assessing for inflammation in a dark-skinned person. Which is the best technique? A) Assessing the skin for cyanosis and swelling B) Assessing the oral mucosa for generalized erythema C) Palpating the skin for edema and increased warmth D) Palpating for tenderness and local areas of ecchymosis

C

The nurse is assessing the level of consciousness of a patient with a head injury who has been unresponsibe for the last eight hours. Using the glasgow coma scale, the nurse notes that the patient opens their eyes only as a response to pain, responds with sounds that are not understandable, and has abnormal extension of the extremities. What should the nurse do A) attempt to arouse the patient B) reposition the patient with the extremities in normal alignment C) chart the patient's level of consciousness as coma D) notify the physician

C

The nurse is assessing the pulses of a patient who has been admitted for untreated hyperthyroidism. The nurse should expect to find a(n) _____ pulse. A)Normal B) Absent C) Bounding D) Weak, thready

C

The nurse is caring for a patient who has just had neurosurgery. To assess for increased intracranial pressure, what would the nurse include in the assessment? A) CNs, motor function, and sensory function B) Deep tendon reflexes, vital signs, and coordinated movements C) Level of consciousness, motor function, pupillary response, and vital signs D) Mental status, deep tendon reflexes, sensory function, and pupillary response

C

The nurse is caring for a patient who is unable to hold a cup or spoon. How should the nurse administer oral medications to the patient? A) Crush the pills and mix them in pudding before administering B) Ask the pharmacist to change all of the medications to a liquid form C). Use a small paper cup to put the pills into the patient's mouth. D) Place the pills on the table and have the patient take the pills by hand.

C

The nurse is caring for a patient with an infiltrated intravenous catheter. The patient's right arm is warm to the touch with 3+ nonpitting edema. which of the following nursing interventions is the priority when caring for the patient? A) placing the patient in high Fowler's position B) administer salty snacks to promote fluid reabsorption C) elevate the affected extremity D) administer ordered pain medications

C

The nurse is completing an assessment of a nonverbal patient who had an appendectomy the previous day. The patient is restless, holding his hand over his abdomen, sweating, and his vital signs are: heart rate 100 bpm, respirations 20, blood pressure 135/90. Which of the following would be the most appropriate action by the nurse? A) Teach the patient the importance about getting out of bed and walking after surgery. B) Offer to turn on healing music. C) Ask the patient to nod "yes" or "no" and ask him or her if he or she is experiencing pain. D) Chart vital signs and assessment and determine normal findings.

C

The nurse is describing a weak, thready pulse on the documentation flow sheet. Which statement is correct? A) "Is easily palpable; pounds under the fingertips." B) "Has greater than normal force, then suddenly collapses." C) "Is hard to palpate, may fade in and out, and is easily obliterated by pressure." D) "Rhythm is regular, but force varies with alternating beats of large and small amplitude."

C

The nurse is doing an assessment on a 29-year-old woman who visits the clinic complaining of "always dropping things and falling down." While testing rapid alternating movements, the nurse notices that the woman is unable to pat both her knees. Her response is very slow and she misses frequently. What should the nurse suspect? A) Vestibular disease B) Lesion of CN IX C) Dysfunction of the cerebellum D) Inability to understand directions

C

The nurse is examining a patient who has possible cardiac enlargement. Which statement about percussion of the heart is true? A) Percussion is a useful tool for outlining the hearts borders. B) Percussion is easier in patients who are obese. C) Studies show that percussed cardiac borders do not correlate well with the true cardiac border. D) Only expert health care providers should attempt percussion of the heart.

C

The nurse is examining a patient's ears and notices cerumen in the external canal. Which of these statements about cerumen is correct? A) Sticky honey-colored cerumen is a sign of infection. B) The presence of cerumen is indicative of poor hygiene. C) The purpose of cerumen is to protect and lubricate the ear. D) Cerumen is necessary for transmitting sound through the auditory canal.

C

The nurse is examining a patients lower leg and notices a draining ulceration. Which of these actions is most appropriate in this situation? A) Washing hands, and contacting the physician B) Continuing to examine the ulceration, and then washing hands C) Washing hands, putting on gloves, and continuing with the examination of the ulceration D) Washing hands, proceeding with rest of the physical examination, and then continuing with the examination of the leg ulceration

C

The nurse is explaining the mechanism of the growth of long bones to a mother of a toddler. Where does lengthening of the bones occur? A) Bursa B) Calcaneus C) Epiphyses D) Tuberosities

C

The nurse is performing a cardiac assessment on a 65-year-old patient 3 days after her myocardial infarction (MI). Heart sounds are normal when she is supine, but when she is sitting and leaning forward, the nurse hears a high-pitched, scratchy sound with the diaphragm of the stethoscope at the apex. It disappears on inspiration. The nurse suspects: A) Increased cardiac output. B) Another MI. C) Inflammation of the precordium. D) Ventricular hypertrophy resulting from muscle damage.

C

The nurse is performing a neurologic assessment on a 41-year-old woman with a history of diabetes. When testing her ability to feel the vibrations of a tuning fork, the nurse notices that the patient is unable to feel vibrations on the great toe or ankle bilaterally, but she is able to feel vibrations on both patellae. Given this information, what would the nurse suspect? A).Hyperalgesia B) Hyperesthesia C) Peripheral neuropathy D).Lesion of sensory cortex

C

The nurse is performing a well-child check on a 5-year-old boy. He has no current history that would lead the nurse to suspect illness. His medical history is unremarkable, and he received immunizations 1 week ago. Which of these findings should be considered normal in this situation? A) Enlarged, warm, and tender nodes B) Lymphadenopathy of the cervical nodes C) Palpable firm, small, shotty, mobile, and nontender lymph nodes D) Firm, rubbery, and large nodes, somewhat fixed to the underlying tissue

C

The nurse is performing an assessment on a 21-year-old patient and notices that his nasalmucosa appears pale, gray, and swollen. What would be the most appropriate question to askthe patient? A) "Have you had any symptoms of a cold?" B) "Do you have an elevated temperature?" C) "Are you aware of having any allergies?" D) "Have you been having frequent nosebleeds?"

C

The nurse is performing an assessment on a 65-year-old man. He reports a crusty nodule behind the pinna. It intermittently bleeds and has not healed over the past 6 months. On physical assessment, the nurse finds an ulcerated crusted nodule with an indurated base. The preliminary analysis in this situation is that this: A) Is most likely a benign sebaceous cyst. B) Is most likely a keloid. C) Could be a potential carcinoma, and the patient should be referred for a biopsy. D) Is a tophus, which is common in the older adult and is a sign of gout.

C

The nurse is performing an assessment on an adult. The adult's vital signs are normal and capillary refill time is 5 seconds. What should the nurse do next? A) Ask the patient about a history of frostbite. B) Suspect that the patient has venous insufficiency. C) Consider this a delayed capillary refill time, and investigate further. D) Consider this a normal capillary refill time that requires no further assessment.

C

The nurse is performing an otoscopic examination on an adult. Which of these actions is correct? A) Tilting the person's head forward during the examination B) Once the speculum is in the ear, releasing the traction C) Pulling the pinna up and back before inserting the speculum D) Using the smallest speculum to decrease the amount of discomfort

C

The nurse is performing percussion during an abdominal assessment. Percussion notes heard during the abdominal assessment may include: A) Flatness, resonance, and dullness. B) Resonance, dullness, and tympany. C) Tympany, hyperresonance, and dullness D) Resonance, hyperresonance, and flatness.

C

The nurse is preparing to assess an older adult and discovers that the older adult is in severe pain.Which statement about pain and the older adult is true? A) Pain is inevitable with aging. B) Older adults with cognitive impairments feel less pain. C) Alleviating pain should be a priority over other aspects of the assessment. D) The assessment should take priority so that care decisions can be made.

C

The nurse is preparing to assess the visual acuity of a 16-year-old patient. How should the nurse proceed? A) Perform the confrontation test. B) Ask the patient to read the print on a handheld Jaeger card. C) Use the Snellen chart positioned 20 feet away from the patient. D) Determine the patient's ability to read newsprint at a distance of 12 to 14 inches.

C

The nurse is preparing to measure the length, weight, chest, and head circumference of a 6-month-old infant. Which measurement technique is correct? A) Measuring the infants length by using a tape measure B) Weighing the infant by placing him or her on an electronic standing scale C) Measuring the chest circumference at the nipple line with a tape measure D) Measuring the head circumference by wrapping the tape measure over the nose and cheekbones

C

The nurse is preparing to percuss the abdomen of a patient. The purpose of the percussion is to assess the __________ of the underlying tissue. A) Turgor B) Texture C) Density D) Consistency

C

The nurse is preparing to perform a modified Allen test. Which is an appropriate reason for this test? A) To measure the rate of lymphatic drainage B) To evaluate the adequacy of capillary patency before venous blood draws C) To evaluate the adequacy of collateral circulation before cannulating the radial artery D) To evaluate the venous refill rate that occurs after the ulnar and radial arteries are temporarily occluded

C

The nurse is preparing to perform an otoscopic examination of a newborn infant. Which statement is true regarding this examination? A) Immobility of the drum is a normal finding. B) An injected membrane would indicate an infection. C) The normal membrane may appear thick and opaque. D) The appearance of the membrane is identical to that of an adult.

C

The nurse is reviewing the assessment of an aortic aneurysm. Which of these statements is true regarding an aortic aneurysm? A) A bruit is absent. B) Femoral pulses are increased. C) A pulsating mass is usually present. D) Most are located below the umbilicus.

C

The nurse is reviewing the blood supply to the arm. The major artery supplying the arm is the _____ artery. A) Ulnar B) Radial C) Brachial D) Deep palmar

C

The nurse is reviewing the characteristics of breath sounds. Which statement about bronchovesicular breath sounds is true? Bronchovesicular breath sounds are: A) Musical in quality. B) Usually caused by a pathologic disease. C) Expected near the major airways. D) Similar to bronchial sounds except shorter in duration.

C

The nurse is reviewing the function of the cranial nerves (CNs). Which CN is responsible for conducting nerve impulses to the brain from the organ of Corti? A) I B) III C) VIII D)XI

C

The nurse is reviewing the technique of palpating for tactile fremitus with a new graduate. Which statement by the graduate nurse reflects a correct understanding of tactile fremitus? Tactile fremitus: A) Is caused by moisture in the alveoli. B) Indicates that air is present in the subcutaneous tissues. C) Is caused by sounds generated from the larynx. D) Reflects the blood flow through the pulmonary arteries.

C

The nurse is selecting a site to administer a medication by the intramuscular route. The nurse chooses to avoid which site due to the high risk for injury? A) Vastus lateralis B) Ventrogluteal C) Dorsogluteal D) Deltoid

C

The nurse is taking temperatures in a clinic with a TMT. Which statement is true regarding use of the TMT? A) A tympanic temperature is more time consuming than a rectal temperature. B) The tympanic method is more invasive and uncomfortable than the oral method. C) The risk of cross-contamination is reduced, compared with the rectal route. D) The tympanic membrane most accurately reflects the temperature in the ophthalmic artery.

C

The nurse is teaching a patient about how to take a sublingual nitroglycerin tablet. Which statement by the patient best demonstrates understanding of the teaching? A) "I will hold the tablet next to my skin." B) "I will put the tablet inside my cheek." C) "I will put the tablet under my tongue." D) "I will place the tablet in the lower lid of my eye."

C

The nurse is testing superficial reflexes on an adult patient. When stroking up the lateral side of the sole and across the ball of the foot, the nurse notices the plantar flexion of the toes. How should the nurse document this finding? A) Positive Babinski sign B) Plantar reflex abnormal C) Plantar reflex present D) Plantar reflex 2+ on a scale from "0 to 4+"

C

The nurse is unable to identify any changes in sound when percussing over the abdomen of an obese patient. What should the nurse do next? A) Ask the patient to take deep breaths to relax the abdominal musculature. B) Consider this finding as normal, and proceed with the abdominal assessment. C) Increase the amount of strength used when attempting to percuss over the abdomen. D) Decrease the amount of strength used when attempting to percuss over the abdomen.

C

The nurse is unable to palpate the right radial pulse on a patient. The best action would be to: A) Auscultate over the area with a fetoscope. B) Use a goniometer to measure the pulsations. C) Use a Doppler device to check for pulsations over the area. D) Check for the presence of pulsations with a stethoscope.

C

The nurse knows that auscultation of fine crackles would most likely be noticed in: A) A healthy 5-year-old child. B) A pregnant woman. C) The immediate newborn period. D) Association with a pneumothorax.

C

The nurse knows that which statement is true regarding the pain experienced by infants? A) Pain in infants can only be assessed by physiologic changes, such as an increased heart rate. B) The FPS-R can be used to assess pain in infants. C) A procedure that induces pain in adults will also induce pain in the infant. D) Infants feel pain less than do adults.

C

The nurse makes this comment to a patient, I know it may be hard, but you should do what the doctor ordered because she is the expert in this field. Which statement is correct about the nurses comment? A) This comment is inappropriate because it shows the nurses bias. B) This comment is appropriate because members of the health care team are experts in their area of patient care. C) This type of comment promotes dependency and inferiority on the part of the patient and is best avoided in an interview situation. D) Using authority statements when dealing with patients, especially when they are undecided about an issue, is necessary at times.

C

The nurse needs to assess a patient's ability to perform activities of daily living and should choose which tool for this assessment? A) Direct Assessment of Functional Abilities (DAFA) B) Lawton IADL C) Barthel Index D) Older Americans Resources and Services Multidimensional Functional Assessment Questionnaire-IADL (OARS-IADL)

C

The nurse needs to palpate the temporomandibular joint for crepitation. This joint is located just below the temporal artery and anterior to the: A) Hyoid bone B) Vagus nerve. C) Tragus D) Mandible.

C

The nurse notes redness on the patient's left forearm. Which of the following is the correct follow-up action? A) ask the patient what happened B) notify the physician C) compare to the patient's right arm D0 chart the findings

C

The nurse notices that a patient has had a black, tarry stool and recalls that a possible cause would be: A) Gallbladder disease. B) Overuse of laxatives. C) Gastrointestinal bleeding. D) Localized bleeding around the anus.

C

The nurse notices that a patient has ulcerations on the tips of the toes and on the lateral aspect of the ankles. This finding indicates: A) Lymphedema. B) Raynaud disease. C) Arterial insufficiency. D) Venous insufficiency.

C

The nurse notices that a patients palpebral fissures are not symmetric. On examination, the nurse may find that damage has occurred to which cranial nerve (CN)? A) III B) V C) VII D) VIII

C

The nurse notices that an infant has a large, soft lump on the side of his head and that his mother is very concerned. She tells the nurse that she noticed the lump approximately 8 hours after her babys birth and that it seems to be getting bigger. One possible explanation for this is: A) Hydrocephalus. B) Craniosynostosis. C) Cephalhematoma. D) Caput succedaneum.

C

The nurse notices that the mother of a 2-year-old boy brings him into the clinic quite frequently for various injuries and suspects there may be some child abuse involved. What should the nurse look for during an inspection of this child's mouth? A) Swollen, red tonsils B) Ulcerations on the hard palate C) Bruising on the buccal mucosa or gums D) Small yellow papules along the hard palate

C

The nurse notices the presence of periorbital edema when performing an eye assessment on a 70-year-old patient. The nurse should: A) Check for the presence of exophthalmos. B) Suspect that the patient has hyperthyroidism. C) Ask the patient if he or she has a history of heart failure. D) Assess for blepharitis, which is often associated with periorbital edema.

C

The nurse reviews a physician's order and finds that the medication amount is greater than the standard dose. What should the nurse do? A) Give the standard dose rather than the one that is ordered. B) Inform the nursing supervisor. C) Call the physician to discuss the order. D) Give the drug as ordered by the physician.

C

The nurse should measure rectal temperatures in which of these patients? A) School-age child B) Older adult C) Comatose adult D) Patient receiving oxygen by nasal cannula

C

The nurse should wear gloves for which of these examinations? A) Measuring vital signs B) Palpation of the sinuses C) Palpation of the mouth and tongue D) Inspection of the eye with an ophthalmoscope

C

The nurse suspects that a patient has carpal tunnel syndrome and wants to perform the Phalen's test. To perform this test, the nurse should instruct the patient to: A) dorsiflex the foot. B) plantarflex the foot. C) hold both hands back to back while flexing the wrists 90 degrees for 60 seconds. D) hyperextend the wrists with the palmar surface of both hands touching and wait for 60 seconds.

C

The nurse will measure a patients near vision with which tool? A) Snellen eye chart with letters B) Snellen E chart C) Jaeger card D) Ophthalmoscope

C

To determine if a darker-skinned patient is experiencing cyanosis or pallor, the nurse should look where? A) earlobes B) axillae C) conjunctiva D) palms of hands

C

To palpate the temporomandibular joint, the nurse's fingers should be placed in the depression _____ of the ear. A) distal to the helix B) proximal to the helix C) anterior to the tragus D) posterior to the tragus

C

What are the projections in the nasal cavity that increase the surface area are called? A) Meatus B) Septum C) Turbinates D) Kiesselbach plexus

C

What can be determined when the nurse clusters data as part of the critical thinking process? A) this step identifies problems that may be urgent and require immediate action. B) this step involves making assumptions in the data. C) the nurse recognizes relevant information among the data. D) risk factors can be determined so the nurse knows how to offer health teaching.

C

What is the primary purpose of the ciliated mucous membrane in the nose? A) To warm the inhaled air B) To filter out dust and bacteria C) To filter coarse particles from inhaled air D) To facilitate the movement of air through the nares

C

What self-care measure is most important for the nurse to include in the teaching plan for a patient who will be discharged with a urostomy? A) Change the appliance before going to bed. B) Cut the wafer 1 inch larger than the stoma. C) Cleanse the peristomal skin with mild soap and water. D) Use firm pressure to attach the wafer to the skin.

C

When a light is directed across the iris of a patient's eye from the temporal side, the nurse is assessing for: A) Drainage from dacryocystitis. B) Presence of conjunctivitis over the iris. C) Presence of shadows, which may indicate glaucoma. D) Scattered light reflex, which may be indicative of cataracts.

C

When assessing a newborn infant who is 5 minutes old, the nurse knows which of these statements to be true? A) The left ventricle is larger and weighs more than the right ventricle. B) The circulation of a newborn is identical to that of an adult. C) Blood can flow into the left side of the heart through an opening in the atrial septum. D)The foramen ovale closes just minutes before birth, and the ductus arteriosus closes immediately after.

C

When assessing a patient's general appearance, the nurse should include which of these questions? A) Is the patient's muscle strength equal in both arms? B) Is ptosis or facial droop present? C) Does the patient respond appropriately to questions? D) Are the pupils equal in reaction and size?

C

When assessing the force, or strength, of a pulse, the nurse recalls that the pulse: A) Is usually recorded on a 0- to 2-point scale. B) Demonstrates elasticity of the vessel wall. C) Is a reflection of the hearts stroke volume. D) Reflects the blood volume in the arteries during diastole.

C

When assessing the neurologic system of a hospitalized patient during morning rounds, the nurse should include which of these during the assessment? A) Blood pressure B) The patient's rating of pain on a 1 to 10 scale C) The patient's ability to communicate D) The patient's personal hygiene level

C

When assessing the pupillary light reflex, the nurse should use which technique? A) Shine a penlight from directly in front of the patient, and inspect for pupillary constriction. B) Ask the patient to follow the penlight in eight directions, and observe for bilateral pupil constriction. C) Shine a light across the pupil from the side, and observe for direct and consensual pupillary constriction. D) Ask the patient to focus on a distant object. Then ask the patient to follow the penlight to approximately 7 cm from the nose.

C

When assessing the respiratory system of a 4-year-old child, which of these findings would the nurse expect? A) Crepitus palpated at the costochondral junctions B) No diaphragmatic excursion as a result of a childs decreased inspiratory volume C) Presence of bronchovesicular breath sounds in the peripheral lung fields D) Irregular respiratory pattern and a respiratory rate of 40 breaths per minute at rest

C

When auscultating the lungs of an adult patient, the nurse notes that low-pitched, soft breath sounds are heard over the posterior lower lobes, with inspiration being longer than expiration. The nurse interprets that these sounds are: A) Normally auscultated over the trachea. B) Bronchial breath sounds and normal in that location C) Vesicular breath sounds and normal in that location D) Bronchovesicular breath sounds and normal in that location.

C

When evaluating a patients pain, the nurse knows that an example of acute pain would be: A) Arthritic pain. B) Fibromyalgia. C) Kidney stones. D) Low back pain.

C

When examining a patient's eyes, the nurse recalls that stimulation of the sympathetic branch of the autonomic nervous system: A) Causes pupillary constriction. B) Adjusts the eye for near vision. C) Elevates the eyelid and dilates the pupil. D) Causes contraction of the ciliary body.

C

When examining the face of a patient, the nurse is aware that the two pairs of salivary glands that are accessible to examination are the ___________ and ___________ glands. A) Occipital; submental B) Parotid; jugulodigastric C) Parotid; submandibular D) Submandibular; occipital

C

When listening to heart sounds, the nurse knows that the S1: A) Is louder than the S2 at the base of the heart. B) Indicates the beginning of diastole. C) Coincides with the carotid artery pulse. D) Is caused by the closure of the semilunar valves.

C

When listening to heart sounds, the nurse knows the valve closures that can be heard best at the base of the heart are: A) Mitral and tricuspid. B) Tricuspid and aortic. C) Aortic and pulmonic. D) Mitral and pulmonic.

C

When measuring a patients body temperature, the nurse keeps in mind that body temperature is influenced by: A) Constipation. B) Patients emotional state. C) Diurnal cycle. D) Nocturnal cycle.

C

When performing a peripheral vascular assessment on a patient, the nurse is unable to palpate the ulnar pulses. The patient's skin is warm and capillary refill time is normal. The nurse should next: A) Check for the presence of claudication. B) Refer the individual for further evaluation. C) Consider this finding as normal, and proceed with the peripheral vascular evaluation. D) Ask the patient if he or she has experienced any unusual cramping or tingling in the arm.

C

When performing respiratory assessment on a patient, the nurse notices a costal angle of approximately 90 degrees. This characteristic is: A) Observed in patients with kyphosis. B) Indicative of pectus excavatum. C) A normal finding in a healthy adult. D) An expected finding in a patient with a barrel chest.

C

When taking the history on a patient with a seizure disorder, the nurse assesses whether the patient has an aura. Which of these would be the best question for obtaining this information? A)"Does your muscle tone seem tense or limp?" B) "After the seizure, do you spend a lot of time sleeping?" C) "Do you have any warning sign before your seizure starts?" D) "Do you experience any color change or incontinence during the seizure?"

C

When the nurse is auscultating the carotid artery for bruits, which of these statements reflects the correct technique? A) While listening with the bell of the stethoscope, the patient is asked to take a deep breath and hold it. B) While auscultating one side with the bell of the stethoscope, the carotid artery is palpated on the other side to check pulsations. C) While lightly applying the bell of the stethoscope over the carotid artery and listening, the patient is asked to take a breath, exhale, and briefly hold it. D) While firmly placing the bell of the stethoscope over the carotid artery and listening, the patient is asked to take a breath, exhale, and briefly hold it.

C

When the nurse is testing the triceps reflex, what is the expected response A) Flexion of the hand B) Pronation of the hand C) Extension of the forearm D) Flexion of the forearmse?

C

When using a Doppler ultrasonic stethoscope, the nurse recognizes venous flow when which sound is heard? A) Low humming sound B) Regular "lub, dub" pattern C) Swishing, whooshing sound D) Steady, even, flowing sound

C

Which culture would describe illness as hot and cold imbalance? A) Asian-American heritage B) African-American heritage C) Hispanic-American heritage D) American Indian heritage

C

Which direction would you give a patient to assist you with palpating the femoral pulse? A) "Push your leg down against the examining table." B) "Relax your leg and turn your foot and knee inward." C) "Bend your knee outward like a frog." D) "Straighten your leg and point your toe."

C

Which of the following actions by the nurse will facilitate relaxation of the abdominal muscles in preparation for the abdominal exam? A) asking the patient to inhale deeply and hold their breath B) asking the patient to guard their abdomen C) asking the patient to flex their legs by placing a pillow under the knees D) placing the patient in the prone position

C

Which of the following has been found to influence pain sensitivity in women? A) age B) parity C) hormonal changes D) weight

C

Which of the following is a function of the skin? A) secretion of sebum B) vitamin B production and synthesis C) temperature regulation D) melanocyte production

C

Which of the following is an example of objective data? A) a sore throat B) an earache C) alert and oriented D) dizziness

C

Which of the following statements regarding language barriers and health care is true? A) English proficiency is associated with a lower quality of care. B) Patients with language barriers have a decreased risk for nonadherence to medication regimens. C) Standards have been identified that are important to eliminate health disparities. D) LEP is associated with a higher quality of care.

C

Which of the following symptoms is greatly influenced by a person's cultural heritage? A) food intolerance B) hearing loss C) pain D) breast lump

C

Which of these findings would the nurse expect to notice during a cardiac assessment on a 4-year-old child? A) S3 when sitting up B) Persistent tachycardia above 150 beats per minute C) Murmur at the second left intercostal space when supine D) Palpable apical impulse in the fifth left intercostal space lateral to midclavicular line

C

Which of these is included in an assessment of general appearance? A) Height B) Weight C) Skin color D) Vital signs

C

Which of these statements describes the closure of the valves in a normal cardiac cycle? A) The aortic valve closes slightly before the tricuspid valve. B) The pulmonic valve closes slightly before the aortic valve. C)The tricuspid valve closes slightly later than the mitral valve. D) Both the tricuspid and pulmonic valves close at the same time.

C

Which statement best describes ethnocentrism? A) the government description of various cultures B) a central belief that accepts all cultures as one's own C) the tendency to view your own way of life as the most desirable. D) the tendency to impose your beliefs, values, and patterns of behavior on an individual from another culture.

C

Which statement best reflects the magicoreligious causation of illness? A) each being is but part of a larger structure in the world of nature as it relates to health and illness B) causality relationships exist, leading to expression of illness C) belief in the struggle between good and evil is reflected in the regulation of health and illness D) illness occurs as a result of disturbances between hot and cold reactions.

C

Which statement by the patient about herbs and prescription medications demonstrates understanding of education by the nurse? A) "I can stop taking my prescription medication when I begin an herbal preparation." B) "I know that herbal preparations are highly regulated in this country to ensure no interactions with prescription medications." C) "I should check with my physician before beginning an herbal preparation." D) "I cannot ever take an herbal preparation while I am using prescription medication."

C

Which statement indicates that the nurse understands the pain experienced by an older adult? A) Older adults must learn to tolerate pain. B) Pain is a normal process of aging and is to be expected. C) Pain indicates a pathologic condition or an injury and is not a normal process of aging. D) Older individuals perceive pain to a lesser degree than do younger individuals.

C

Which would be considered a risk diagnosis? A) identifying existing levels of wellness B) evaluating previous problems and goals C) Identifying potential problems the individual may develop D) focusing on strengths and reflecting an individual's transition to higher levels of wellness

C

Which would be included in a holistic model of assessment? A) nursing goals for the patient B) anticipated growth and development patterns C) a patient's perception of his or her health status D) the nurse's perception of disease related to the patient

C

While assessing a 7-month-old infant, the nurse makes a loud noise and notices the following response: abduction and flexion of arms and legs; fanning of fingers, and curling of index and thumb in a C position followed by infant bringing in arms and legs to body. What does the nurse know about this response? A).This response could indicate brachial nerve palsy. B) This reaction is an expected startle response at this age. C) This reflex should have disappeared between 1 and 4 months of age. D) This response is normal as long as the movements are bilaterally symmetric.

C

While counting the apical pulse on a 16-year-old patient, the nurse notices an irregular rhythm. His rate speeds up on inspiration and slows on expiration. What would be the nurses response? A) Talk with the patient about his intake of caffeine. B) Perform an electrocardiogram after the examination. C) No further response is needed because sinus arrhythmia can occur normally. D) Refer the patient to a cardiologist for further testing.

C

While examining a patient, the nurse observes abdominal pulsations between the xiphoid process and umbilicus. The nurse would suspect that these are: A) Pulsations of the renal arteries. B) Pulsations of the inferior vena cava. C) Normal abdominal aortic pulsations. D) Increased peristalsis from a bowel obstruction.

C

While gathering equipment after an injection, a nurse accidentally received a prick from an improperly capped needle. To interpret this sensation, which of these areas must be intact? A) Corticospinal tract, medulla, and basal ganglia B) Pyramidal tract, hypothalamus, and sensory cortex C) Lateral spinothalamic tract, thalamus, and sensory cortex D) Anterior spinothalamic tract, basal ganglia, and sensory cortex

C

While performing a well-child assessment on a 5 year old, the nurse notes the presence of palpable, bilateral, cervical, and inguinal lymph nodes. They are approximately 0.5 cm in size, round, mobile, and nontender. The nurse suspects that this child: A) Has chronic allergies. B) May have an infection. C) Is exhibiting a normal finding for a well child of this age. D) Should be referred for additional evaluation.

C

While recording in a patients medical record, the nurse notices that a patients Hematest results are positive. This finding means that there is(are): A) Crystals in his urine B) Parasites in his stool. C) Occult blood in his stool. D) Bacteria in his sputum.

C

You are reviewing assessment data of a 45- year - old male patient and note pain of 8 on a scale of 10, labored breathing, and pale skin color on the electronic health record. This documentation is an example of A) hypothetical reasoning B) diagnostic reasoning C) data cluster D) signs and symptoms

C

You are the triage nurse in the emergency department and perform the initial intake assessment on a patient who does not speak English. Based on your understanding of linguistic competence, which action would present as a barrier to effective communication? A) Maintaining a professional respectful demeanor B) allowing for additional time to complete the process C) providing the patient with a paper and pencil so he or she can write down the answers to the questions you are going to ask D) seeing is there are any family members present who may assist with the interview process.

C

following a motor vehicle accident, a patient arrives in the emergency department (ED) with massive right lower extremity swelling and 10/10 pain. which of the following assessments would take priority? A) gait assessment B) romberg test C) checking pulses in the affected extremity D) assessment of motor sensation and function

C

mmediately after birth, the nurse is unable to suction the nares of a crying newborn. Anattempt is made to pass a catheter through both nasal cavities with no success. What should the nurse do next? A) Attempt to suction again with a bulb syringe. B) Wait a few minutes, and try again once the infant stops crying. C) Recognize that this situation requires immediate intervention. D) Contact the physician to schedule an appointment for the infant at his or her nexthospital visit.

C

what are expected findings in an adult with a normal lung and thorax examination A) adventitious breath sounds and limited chest expansion B) muffled voice sounds and symmetrical tactile fremitus C) increased tactile fremitus and dull percussion tones D) absent voice sounds and hyperrresonant percussion tones

C

which of the following assessment data items would warrant follow-up by the nurse when assessing a 72 year old patient admitted for a heart failure exacerbation? A) capillary refill of 2 seconds B) 160 degree nail angle C) supraclavicular tenting >3 seconds D) longitudinal ridging on some nails

C

which of the following assessment findings should the nurse report to the provider immediately in the patient admitted after surgical repair of a pelvic fracture? A) serous drainage on the patient's dressing B) 10/10 pelvis pain C) absent bowel sounds D) abdominal and pelvic brusing

C

which of the following best describes the rationale for the correct order of abdominal assessment? A) palpation should occur first to determine area of tenderness before asucultation B) percussion should occur before palpation to avoid injury to the patient C) auscultation should occur before palpation to avoid distortion of bowel sounds D) inspection of the abdomen should occur last so that they can observe any possible changes

C

which of the following findings require immediate attention from the nurse conducting a peripheral vascular assessment? A) 2/10 pain when walking B) hairless, shiny calves C) change in pulse intensity from baseline D) stage 2 ulcer on the patient's right great toe

C

which of the following nursing diagnosis would take priority for the patient admitted for an open reduction and internal fixation (ORIF) of a fractured tibia A) risk for constipation related to prolonged bed reset B) activity intolerance related to reconditioning C) risk for infection related to placement of external fixation device D) impaired active range of motion associated with knee and ankle discomfort as evidenced by hesitant, unsteady gait

C

while assessing a 36 year old patient admitted for pneumonia, the patient suddenly begins to complain of chest pain. What is the nurse's best response? A) notify the physician B) complete a focused cardiac assessment C) assess the patient's pain D) administer ordered pain medications

C

A 19-year-old college student is brought to the emergency department with a severe headache he describes as, Like nothing I've ever had before. His temperature is 40 C, and he has a stiff neck. The nurse looks for other signs and symptoms of which problem? A) Head injury B) Cluster headache C) Migraine headache D) Meningeal inflammation

D

A 22-year-old man comes to the clinic for an examination after falling off his motorcycle and landing on his left side on the handle bars. The nurse suspects that he may have injured his spleen. Which of these statements is true regarding assessment of the spleen in this situation? A) The spleen can be enlarged as a result of trauma. B) The spleen is normally felt on routine palpation. C) If an enlarged spleen is noted, then the nurse should thoroughly palpate to determine its size. D) An enlarged spleen should not be palpated because it can easily rupture.

D

A 30-year-old woman with a history of mitral valve problems states that she has been very tired. She has started waking up at night and feels like her heart is pounding. During the assessment, the nurse palpates a thrill and lift at the fifth left intercostal space midclavicular line. In the same area, the nurse also auscultates a blowing, swishing sound right after the S1. These findings would be most consistent with: A) Heart failure. B) Aortic stenosis. C) Pulmonary edema. D) Mitral regurgitation.

D

A 32-year-old woman is at the clinic for "little white bumps in my mouth." During theassessment, the nurse notes that she has a 0.5-cm white, nontender papule under her tongueand one on the mucosa of her right cheek. What would the nurse tell the patient? A) "These spots indicate an infection such as strep throat." B) "These bumps could be indicative of a serious lesion, so I will refer you to aspecialist." C) "This condition is called leukoplakia and can be caused by chronic irritation suchas with smoking." D) "These bumps are Fordyce granules, which are sebaceous cysts and are not aserious condition."

D

A 32-year-old woman tells the nurse that she has noticed "very sudden, jerky movements" mainly in her hands and arms. She says, "They seem to come and go, primarily when I am trying to do something. I haven't noticed them when I'm sleeping." This description suggests: A) Tics. B) Athetosis. C) Myoclonus. D) Chorea.

D

A 45-year-old man is in the clinic for a physical examination. During the abdominal assessment, the nurse percusses the abdomen and notices an area of dullness above the right costal margin of approximately 11 cm. The nurse should: A) Document the presence of hepatomegaly. B) Ask additional health history questions regarding his alcohol intake. C) Describe this dullness as indicative of an enlarged liver, and refer him to a physician. D) Consider this finding as normal, and proceed with the examination.

D

A 50-year-old woman is in the clinic for weakness in her left arm and leg that she has noticed for the past week. The nurse should perform which type of neurologic examination? A) Glasgow Coma Scale B) Neurologic recheck examination C) Screening neurologic examination D) Complete neurologic examination

D

A 52-year-old patient describes the presence of occasional floaters or spots moving in front of his eyes. The nurse should: A) Examine the retina to determine the number of floaters. B) Presume the patient has glaucoma and refer him for further testing. C) Consider these to be abnormal findings, and refer him to an ophthalmologist. D) Know that floaters are usually insignificant and are caused by condensed vitreous fibers.

D

A 60-year-old woman has developed reflexive sympathetic dystrophy after arthroscopic repair of her shoulder. A key feature of this condition is that the: A) Affected extremity will eventually regain its function. B) Pain is felt at one site but originates from another location. C) Patients pain will be associated with nausea, pallor, and diaphoresis. D) Slightest touch, such as a sleeve brushing against her arm, causes severe and intense pain.

D

A 65-year-old man with emphysema and bronchitis has come to the clinic for a follow-up appointment. On assessment, the nurse might expect to see which assessment finding? A) Anasarca B) Scleroderma C) Pedal erythema D) Clubbing of the nails

D

A 70-year-old patient is scheduled for open-heart surgery. The surgeon plans to use the great saphenous vein for the coronary bypass grafts. The patient asks, "What happens to my circulation when the veins are removed?" The nurse should reply: A) "Venous insufficiency is a common problem after this type of surgery." B) "Oh, you have lots of veins—you won't even notice that it has been removed." C) "You will probably experience decreased circulation after the vein is removed." D) "This vein can be removed without harming your circulation because the deeper veins in your leg are in good condition."

D

A 70-year-old woman tells the nurse that every time she gets up in the morning or after she's been sitting she gets "really dizzy" and feels like she is going to fall over. The nurse's best response would be: A) "Have you been extremely tired lately?" B) "You probably just need to drink more liquids." C) "I'll refer you for a complete neurologic examination." D) "You need to get up slowly when you've been lying down or sitting."

D

A 72-year-old patient has a history of hypertension and chronic lung disease. Which is animportant question for the nurse to include in this patient's health history? A) "Do you use a fluoride supplement?" B) "Have you had tonsillitis in the last year?" C) "At what age did you get your first tooth?" D) "Have you noticed any dryness in your mouth?"

D

A 75 year old women has come in for an assessment of her rheumatoid arthritis. the nurse notes raise, firm, non-tender nodules at the olecranon bursa and along the ulna. What is the appropriate term for these nodules A) epicondylitis B) gouty arthritis C) olecranon bursitis D) subcutaneous nodules

D

A 78-year-old man has a history of a cerebrovascular accident. The nurse notes that when he walks his left arm is immobile against the body with flexion of the shoulder, elbow, wrist, and fingers and adduction of the shoulder. His left leg is stiff and extended and circumducts with each step. What type of gait disturbance is this individual experiencing? A) Scissors gait B) Cerebellar ataxia C) Parkinsonian gait D) Spastic hemiparesis

D

A complete database is A) used to collect data rapidly and is often compiled concurrently with lifesaving measures. B) used to evaluate the cause or etiology of disease. C) used for a limited or short-term problem usually consisting of one problem, one cue complex, or one body system. D) used to perform a thorough or comprehensive health history and physical examination.

D

A father brings in his 2-month-old infant to the clinic because the infant has had diarrhea for the last 24 hours. He says that his baby has not been able to keep any formula down and that the diarrhea has been at least every 2 hours. The nurse suspects dehydration. The nurse should test skin mobility and turgor in this infant over the: A) Sternum. B) Forehead. C) Forearms. D) Abdomen.

D

A female American Indian has come to the clinic for follow-up diabetic teaching. During the interview, the nurse notices that she never makes eye contact and speaks mostly to the floor. Which statement is true regarding this situation? A) The woman is nervous and embarrassed. B) She has something to hide and is ashamed. C) The woman is showing inconsistent verbal and nonverbal behaviors. D) She is showing that she is carefully listening to what the nurse is saying.

D

A female nurse is interviewing a male patient who is near the same age as the nurse. During the interview, the patient makes an overtly sexual comment. The nurses best reaction would be: A) Stop that immediately! B) Oh, you are too funny. Lets keep going with the interview. C) Do you really think I would be interested? D) It makes me uncomfortable when you talk that way. Please stop.

D

A female patient has had frequent urinary tract infections. Which statement by the patient indicates that the nurse's teaching on prevention has been effective? A) "I will limit my fluid intake to 40 ounces per day." B) "I will use bubble bath when bathing." C). "I will wait to wear my tight jeans until after my urine is clear." D) "I will wipe from the front to back after voiding."

D

A man arrives at the clinic for his annual wellness physical. He is experiencing no acute health problems. Which question or statement by the nurse is most appropriate when beginning the interview? A) How is your family? B) How is your job? C) Tell me about your hypertension. D) How has your health been since your last visit?

D

A man has been admitted to the observation unit for observation after being treated for a large cut on his forehead. As the nurse works through the interview, one of the standard questions has to do with alcohol, tobacco, and drug use. When the nurse asks him about tobacco use, he states, I quit smoking after my wife died 7 years ago. However, the nurse notices an open pack of cigarettes in his shirt pocket. Using confrontation, the nurse could say: A) Mr. K., I know that you are lying. B) Mr. K., come on, tell me how much you smoke. C) Mr. K., I didnt realize your wife had died. It must be difficult for you at this time. Please tell me more about that. D) Mr. K., you have said that you dont smoke, but I see that you have an open pack of cigarettes in your pocket.

D

A man has come in to the clinic for a skin assessment because he is afraid he might have skin cancer. During the skin assessment the nurse notices several areas of pigmentation that look greasy, dark, and "stuck on" his skin. Which is the best prediction? He probably has: A) Senile lentigines, which do not become cancerous B) Actinic keratoses, which are precursors to basal cell carcinoma C) Acrochordons, which are precursors to squamous cell carcinoma D) Seborrheic keratoses, which do not become cancerous

D

A man who has had gout for several years comes to the clinic with a problem with his toe. On examination, the nurse notices the presence of hard, painless nodules over the great toe; one has burst open with a chalky discharge. This finding is known as: A) a callus. B) a plantar wart. C) a bunion. D) tophi.

D

A mother brings her 2-month-old daughter in for an examination and says, My daughter rolled over against the wall, and now I have noticed that she has this spot that is soft on the top of her head. Is something terribly wrong? The nurses best response would be: A) Perhaps that could be a result of your dietary intake during pregnancy. B) Your baby may have craniosynostosis, a disease of the sutures of the brain. C) That soft spot may be an indication of cretinism or congenital hypothyroidism. D) That soft spot is normal, and actually allows for growth of the brain during the first year of your babys life.

D

A mother brings her 4-month-old infant to the clinic with concerns regarding a small pad in the middle of the upper lip that has been there since 1 month of age. The infant has no health problems. On physical examination, the nurse notices a 0.5-cm, fleshy, elevated area in the middle of the upper lip. No evidence of inflammation or drainage is observed. What would the nurse tell this mother? A) "This area of irritation is caused from teething and is nothing to worry about." B) "This finding is abnormal and should be evaluated by another health care provider." C) "This area of irritation is the result of chronic drooling and should resolve within the next month or two." D) "This elevated area is a sucking tubercle caused from the friction of breast feeding or bottle-feeding and is normal."

D

A mother brings her child in to the clinic for an examination of the scalp and hair. She states that the child has developed some places where there are irregularly shaped patches with broken-off, stub-like hair and she is worried that this could be some form of premature baldness. The nurse tells her that it is: A) Folliculitis that can be treated with an antibiotic. B) Traumatic alopecia that can be treated with antifungal medications. C) Tinea capitis that is highly contagious and needs immediate attention. D) Trichotillomania; her child probably has a habit of absentmindedly twirling her hair

D

A newborn infant has Down syndrome. During the skin assessment, the nurse notices a transient mottling in the trunk and extremities in response to the cooler examination room temperature. The infant's mother also notices the mottling and asks what it is. The nurse knows that this mottling is called: A) Caf au lait. B) Carotenemia. C) Acrocyanosis. D) Cutis marmorata.

D

A patient comes into the clinic complaining of facial pain, fever, and malaise. On examination, the nurse notes swollen turbinates and purulent discharge from the nose. The patient also complains of a dull, throbbing pain in his cheeks and teeth on the right side and pain when the nurse palpates the areas. What do these findings indicate? A) Nasal polyps B) Frontal sinusitis C) Posterior epistaxis D) Maxillary sinusitis

D

A patient comes into the emergency department after an accident at work. A machine blew dust into his eyes, and he was not wearing safety glasses. The nurse examines his corneas by shining a light from the side across the cornea. What findings would suggest that he has suffered a corneal abrasion? A) Smooth and clear corneas B) Opacity of the lens behind the cornea C) Bleeding from the areas across the cornea D) Shattered look to the light rays reflecting off the cornea

D

A patient comes to the clinic and states that he has noticed that his skin is redder than normal. The nurse understands that this condition is due to hyperemia and knows that it can be caused by: A) Decreased amounts of bilirubin in the blood B) Excess blood in the underlying blood vessels C) Decreased perfusion to the surrounding tissues D) Excess blood in the dilated superficial capillaries

D

A patient complains that while studying for an examination he began to notice a severe headache in the frontotemporal area of his head that is throbbing and is somewhat relieved when he lies down. He tells the nurse that his mother also had these headaches. The nurse suspects that he may be suffering from: A) Hypertension. B) Cluster headaches. C) Tension headaches. D) Migraine headaches.

D

A patient has a long history of chronic obstructive pulmonary disease (COPD). During the assessment, the nurse will most likely observe which of these? A) Unequal chest expansion B) Increased tactile fremitus C) Atrophied neck and trapezius muscles D) Anteroposterior-to-transverse diameter ratio of 1:1

D

A patient has a normal pupillary light reflex. The nurse recognizes that this reflex indicates that: A) The eyes converge to focus on the light. B) Light is reflected at the same spot in both eyes. C) The eye focuses the image in the center of the pupil. D) Constriction of both pupils occurs in response to bright light.

D

A patient has been admitted to the hospital with vertebral fractures related to osteoporosis. She is in extreme pain. This type of pain would be classified as: A)Referred. B) Cutaneous. C) Visceral. D) Deep somatic.

D

A patient has been diagnosed with venous stasis. Which of these findings would the nurse most likely observe? A) Unilateral cool foot B) Thin, shiny, atrophic skin C) Pallor of the toes and cyanosis of the nail beds D) Brownish discoloration to the skin of the lower leg

D

A patient has finished giving the nurse information about the reason he is seeking care. When reviewing the data, the nurse finds that some information about past hospitalizations is missing. At this point, which statement by the nurse would be most appropriate to gather these data? A) Mr. Y., at your age, surely you have been hospitalized before! B) Mr. Y., I just need permission to get your medical records from County Medical. C) Mr. Y., you mentioned that you have been hospitalized on several occasions. Would you tell me more about that? D) Mr. Y., I just need to get some additional information about your past hospitalizations. When was the last time you were admitted for chest pain?

D

A patient has had a "terrible itch" for several months that he has been scratching continuously. On examination, the nurse might expect to find: A) A keloid. B) A fissure. C) Keratosis. D) Lichenification.

D

A patient has had arthritic pain in her hips for several years since a hip fracture. She is able to move around in her room and has not offered any complaints so far this morning. However, when asked, she states that her pain is bad this morning and rates it at an 8 on a 1-to-10 scale. What does the nurse suspect? The patient: A) Is addicted to her pain medications and cannot obtain pain relief. B) Does not want to trouble the nursing staff with her complaints. C) Is not in pain but rates it high to receive pain medication. D) Has experienced chronic pain for years and has adapted to it.

D

A patient is complaining of severe knee pain after twisting it during a basketball game and is requesting pain medication. Which action by the nurse is appropriate? A) Completing the physical examination first and then giving the pain medication B) Telling the patient that the pain medication must wait until after the x-ray images are completed C) Evaluating the full range of motion of the knee and then medicating for pain D) Administering pain medication and then proceeding with the assessment

D

A patient is experiencing acute renal failure. What is the most common cause of this critical illness? A) Hypovolemia B) Cardiogenic shock C) Nephrotoxic substances D) Urethral obstruction

D

A patient is unable to read even the largest letters on the Snellen chart. The nurse should take which action next? A) Refer the patient to an ophthalmologist or optometrist for further evaluation. B) Assess whether the patient can count the nurse's fingers when they are placed in front of his or her eyes. C) Ask the patient to put on his or her reading glasses and attempt to read the Snellen chart again. D) Shorten the distance between the patient and the chart until the letters are seen, and record that distance.

D

A patient recently diagnosed with peripheral artery disease reports both legs hurt when walking. what should the nurse instruct the patient to do? A) stop walking when the pain occurs B) rest frequently and elevate the legs C) wear support stockings D) enroll in a supervised training program at a rehabilitation facility

D

A patient states, Whenever I open my mouth real wide, I feel this popping sensation in front of my ears. To further examine this, the nurse would: A) Place the stethoscope over the temporomandibular joint, and listen for bruits. B) Place the hands over his ears, and ask him to open his mouth really wide. C) Place one hand on his forehead and the other on his jaw, and ask him to try to open his mouth. D) Place a finger on his temporomandibular joint, and ask him to open and close his mouth.

D

A patient visits the clinic because he has recently noticed that the left side of his mouth is paralyzed. He states that he cannot raise his eyebrow or whistle. The nurse suspects that he has: A) Cushing syndrome. B) Parkinson disease. C) Bell palsy. D) Experienced a cerebrovascular accident (CVA) or stroke.

D

A patient with chronic obstructive pulmonary disease (COPD) reports steady weight loss and being "too tired from just breathing to eat". Which of the following nursing diagnoses would be most appropriate when planning nutritional interventions for this patient? A) activity intolerance related to dyspnea B) weight loss related to COPD C) ineffective breathing pattern due to hypoventilation D) altered nutrition: less than body requirements related to fatigue

D

A patients abdomen is bulging and stretched in appearance. The nurse should describe this finding as: A) Obese. B) Herniated. C) Scaphoid. D) Protuberant.

D

A pregnant woman states that she is concerned about her gums because she has noticed theyare swollen and have started to bleed. What would be an appropriate response by the nurse? A) "Your condition is probably due to a vitamin C deficiency." B) "I'm not sure what causes swollen and bleeding gums, but let me know if it's notbetter in a few weeks." C) "You need to make an appointment with your dentist as soon as possible to havethis checked." D) "Swollen and bleeding gums can be caused by a change in hormonal balanceduring pregnancy."

D

A professional tennis player comes into the clinic complaining of a sore elbow. The nurse will assess for tenderness at the: A) olecranon bursa. B) annular ligament. C) base of the radius. D) medial and lateral epicondyle.

D

A woman has just entered the emergency department after being battered by her husband. The nurse needs to get some information from her to begin treatment. What is the best choice for an opening phase of the interview with this patient? A) Hello, Nancy, my name is Mrs. C B) Hello, Mrs. H., my name is Mrs. C. It sure is cold today! C) Mrs. H., my name is Mrs. C. How are you? D) Mrs. H., my name is Mrs. C. Ill need to ask you a few questions about what happened.

D

A woman is leaving on a trip to Hawaii and has come in for a checkup. During the examination the nurse notices that she is diabetic and takes oral hypoglycemic agents. The patient needs to be concerned about which possible effect of her medications? A) Increased possibility of bruising B) Skin sensitivity as a result of exposure to salt water C) Lack of availability of glucose-monitoring supplies D) Importance of sunscreen and avoiding direct sunlight

D

A woman who is in the second trimester of pregnancy mentions that she has had "more nose bleeds than ever" since she became pregnant. What is the likely reason for this? A) Inappropriate use of nasal sprays B) A problem with the patient's coagulation system C) Increased susceptibility to colds and nasal irritation D) Increased vascularity in the upper respiratory tract as a result of the pregnancy

D

A young swimmer comes to the sports clinic complaining of a very sore shoulder. He was running at the pool, slipped on some wet concrete, and tried to catch himself with his outstretched hand. He landed on his outstretched hand and has not been able to move his shoulder since then. The nurse suspects: A) joint effusion. B) tear of rotator cuff. C) adhesive capsulitis. D) dislocated shoulder.

D

After assessing a female patient, the nurse notices flesh-colored, soft, pointed, moist, papules in a cauliflower-like patch around her introitus. This finding is most likely: A) Urethral caruncle. B) Syphilitic chancre. C) Herpes simplex virus. D) Human papillomavirus.

D

An assessment of a 23-year-old patient reveals the following: an auricle that is tender and reddish-blue in color with small vesicles. The nurse would need to know additional information that includes which of these? A) Any change in the ability to hear B) Any recent drainage from the ear C) Recent history of trauma to the ear D) Any prolonged exposure to extreme cold

D

An examiner is using an ophthalmoscope to examine a patients eyes. The patient has astigmatism and is nearsighted. The use of which of these techniques would indicate that the examination is being correctly performed? A) Using the large full circle of light when assessing pupils that are not dilated B) Rotating the lens selector dial to the black numbers to compensate for astigmatism C) Using the grid on the lens aperture dial to visualize the external structures of the eye D) Rotating the lens selector dial to bring the object into focus

D

An example of subjective data is A) decreased range of motion B) crepitation in left knee joint C) arthritis D) left knee has been swollen and hot for past 3 days

D

An ophthalmic examination reveals papilledema. The nurse is aware that this finding indicates: A) Retinal detachment. B) Diabetic retinopathy. C) Acute-angle glaucoma. D) Increased intracranial pressure.

D

Axillary nodes drain the: A) anterior abdominal wall. B) lower extremities. C) hand and lower arm. D) breast and upper arm.

D

Before auscultating the abdomen for the presence of bowel sounds on a patient, the nurse should: A) Warm the endpiece of the stethoscope by placing it in warm water. B) Leave the gown on the patient to ensure that he or she does not get chilled during the examination. C) Ensure that the bell side of the stethoscope is turned to the on position. D) Check the temperature of the room, and offer blankets to the patient if he or she feels cold.

D

CRIES is an appropriate pain assessment tool for A) cognitively impaired older adults B) children ages 2 to 8 years C) infants and children D) preterm and term neonates

D

CRIES is an inappropriate pain assessment tool for: A) Cognitively impaired older adults B) Children ages 2 to 8 years C) Infants D) Preterm and term neonates

D

Consider the below scenario phone conversation when answering the following three questions:"Dr. Jones, this is Mary Smith, RN, on the postsurgical unit at City Hospital. I'm calling about Tom King, your 46-year-old patient who had an inguinal hernia repair this morning. He has not voided since surgery, 8 hours ago. He has received 1900 mL Lactated Ringers IV and 720 mL oral fluids. He can't initiate a stream, but states that he "feels the need to urinate." His bladder is distended by palpation and shows a volume of 800 mL when scanned with the bladder scanner. We've tried standing him to void, providing privacy, and running water, but he is still unable to go. He appears to have urinary retention and I'd like to try using a straight catheter to relieve his retention, what do you think?"In the above scenario, what part of the SBAR communication tool is the underlined information? A) S B) B C) A D) R

D

During a cardiac assessment on a 38-year-old patient in the hospital for chest pain, the nurse finds the following: jugular vein pulsations 4 cm above the sternal angle when the patient is elevated at 45 degrees, blood pressure 98/60 mm Hg, heart rate 130 beats per minute, ankle edema, difficulty breathing when supine, and an S3 on auscultation. Which of these conditions best explains the cause of these findings? A) Fluid overload B) Atrial septal defect C) MI D) Heart failure

D

During a morning assessment, the nurse notices that the patients sputum is frothy and pink. Which condition could this finding indicate? A) Croup B) Tuberculosis C) Viral infection D) Pulmonary edema

D

During an abdominal assessment, the nurse tests for a fluid wave. A positive fluid wave test occurs with: A) Splenomegaly B) Distended bladder. C) Constipation. D) Ascites.

D

During an assessment of a 32-year-old patient with a recent head injury, the nurse notices that the patient responds to pain by extending, adducting, and internally rotating his arms. His palms pronate and his lower extremities extend with plantar flexion. Which of these statements about these findings is accurate? A) Indicates a lesion of the cerebral cortex. B) Indicates a completely nonfunctional brainstem. C) Is normal and will go away in 24 to 48 hours. D) Is a very ominous sign and may indicate brainstem injury.

D

During an assessment of a healthy adult, where would the nurse expect to palpate the apical impulse? A) Third left intercostal space at the midclavicular line B) Fourth left intercostal space at the sternal border C) Fourth left intercostal space at the anterior axillary line D) Fifth left intercostal space at the midclavicular line

D

During an assessment of the sclera of a black patient, the nurse would consider which of these an expected finding? A) Yellow fatty deposits over the cornea B) Pallor near the outer canthus of the lower lid C) Yellow color of the sclera that extends up to the iris D) Presence of small brown macules on the sclera

D

During an assessment, the nurse is unable to palpate pulses in the left lower leg. What should the nurse do next? A) Document that the pulses are not palpable. B) Reassess the pulses in 1 hour. C) Have the patient turn to the side and then palpate for the pulses again. D) Use a Doppler device to assess the pulses

D

During an assessment, the nurse notes that the patients apical impulse is laterally displaced and is palpable over a wide area. This finding indicates: A) Systemic hypertension. B) Pulmonic hypertension. C) Pressure overload, as in aortic stenosis. D) Volume overload, as in heart failure.

D

During an examination of a 3-year-old child, the nurse will need to take her blood pressure. What might the nurse do to try to gain the childs full cooperation? A) Tell the child that the blood pressure cuff is going to give her arm a big hug. B) Tell the child that the blood pressure cuff is asleep and cannot wake up. C) Give the blood pressure cuff a name and refer to it by this name during the assessment. D) Tell the child that by using the blood pressure cuff, we can see how strong her muscles are.

D

During an examination of the eye, the nurse would expect what normal finding when assessing the lacrimal apparatus? A) Presence of tears along the inner canthus B) Blocked nasolacrimal duct in a newborn infant C) Slight swelling over the upper lid and along the bony orbit if the individual has a cold D) Absence of drainage from the puncta when pressing against the inner orbital rim

D

During an examination, a patient has just successfully completed the finger-to-nose and the rapid- alternating-movements tests and is able to run each heel down the opposite shin. The nurse will conclude that the patients function is intact. A) Occipital B) Cerebral C) Temporal D) Cerebellar

D

During an examination, the nurse notices that a patient is unable to stick out his tongue. Which cranial nerve is involved with the successful performance of this action? A) I B) V C) XI D) XII

D

During an inspection of the precordium of an adult patient, the nurse notices the chest moving in a forceful manner along the sternal border. This finding most likely suggests a(n): A) Normal heart. B) Systolic murmur. C) Enlargement of the left ventricle. D) Enlargement of the right ventricle.

D

During an interview, a parent of a hospitalized child is sitting in an open position. As the interviewer begins to discuss his sons treatment, however, he suddenly crosses his arms against his chest and crosses his legs. This changed posture would suggest that the parent is: A) Simply changing positions. B) More comfortable in this position. C) Tired and needs a break from the interview. D) Uncomfortable talking about his sons treatment.

D

During an interview, the nurse states, You mentioned having shortness of breath. Tell me more about that. Which verbal skill is used with this statement? A) Reflection B) Facilitation C) Direct question D) Open-ended question

D

During an interview, the patient states he has the sensation that "everything around him is spinning." The nurse recognizes that the portion of the ear responsible for this sensation is the: A) Cochlea. B) CN VIII. C) Organ of Corti. D) Labyrinth.

D

During an otoscopic examination, the nurse notices an area of black and white dots on the tympanic membrane and the ear canal wall. What does this finding suggest? A) Malignancy B) Viral infection C) Blood in the middle ear D) Yeast or fungal infection

D

During ocular examinations, the nurse keeps in mind that movement of the extraocular muscles is: A) Decreased in the older adult. B) Impaired in a patient with cataracts. C) Stimulated by cranial nerves (CNs) I and II. D) Stimulated by CNs III, IV, and VI.

D

During percussion, the nurse knows that a dull percussion note elicited over a lung lobe most likely results from: A) Shallow breathing. B) Normal lung tissue. C) Decreased adipose tissue. D) Increased density of lung tissue.

D

During the aging process, the hair can look gray or white and begin to feel thin and fine. The nurse knows that this occurs because of a decrease in the number of functioning: A) Metrocytes. B) Fungacytes. C) Phagocytes. D) Melanocytes.

D

During the cardiac auscultation, the nurse hears a sound immediately occurring after the S2 at the second left intercostal space. To further assess this sound, what should the nurse do? A) Have the patient turn to the left side while the nurse listens with the bell of the stethoscope. B) Ask the patient to hold his or her breath while the nurse listens again. C) No further assessment is needed because the nurse knows this sound is an S3. D) Watch the patients respirations while listening for the effect on the sound.

D

During the examination of a patients mouth, the nurse observes a nodular bony ridge down the middle of the hard palate. The nurse would chart this finding as: A) Cheilosis. B) Leukoplakia. C) Ankyloglossia. D) Torus palatinus.

D

During the history of a 78-year-old man, his wife states that he occasionally has problems with short-term memory loss and confusion: "He can't even remember how to button his shirt." In doing the assessment of his sensory system, which action by the nurse is most appropriate? A) The nurse would not test the sensory system as part of the examination because the results would not be valid. B) The nurse would perform the tests, knowing that mental status does not affect sensory ability. C) The nurse would proceed with an explanation of each test, making certain that the wife understands. D) Before testing, the nurse would assess the patient's mental status and ability to follow directions.

D

During the interview portion of data collection, the nurse collects __________ data. A)Physical B)Historical C) Objective D) Subjective

D

Fibrous bands running directly from one bone to another that strengthen the joint and help prevent movement in undesirable directions are called: A) bursa. B) tendons. C) cartilage. D) ligaments.

D

For older adults postoperative patients, poorly controlled acute pain places them at higher risk for: A) Atelectasis B) Increased myocardial oxygen demand C) Impaired wound healing D) All the above

D

In an individual with otitis externa, which of these signs would the nurse expect to find on assessment? A) Rhinorrhea B) Periorbital edema C) Pain over the maxillary sinuses D) Enlarged superficial cervical nodes

D

In assessing a 70-year-old patient who has had a recent cerebrovascular accident, the nurse notices right-sided weakness. What might the nurse expect to find when testing his reflexes on the right side? A) Lack of reflexes B) Normal reflexes C) Diminished reflexes D) Hyperactive reflexes

D

In performing an examination of a 3-year-old child with a suspected ear infection, the nurse would: A) Omit the otoscopic examination if the child has a fever. B) Pull the ear up and back before inserting the speculum. C) Ask the mother to leave the room while examining the child. D) Perform the otoscopic examination at the end of the assessment

D

In using verbal responses to assist the patients narrative, some responses focus on the patients frame of reference and some focus on the health care providers perspective. An example of a verbal response that focuses on the health care providers perspective would be: A) Empathy. B) Reflection. C) Facilitation. D) Confrontation.

D

Mrs. Gorman comes to the ambulatory health centre for a routine health assessment. On examination, you perform the modified Allen test, which assesses: A) early clubbing B) .the presence of thrombophlebitis. C) the degree of pedal edema. D) the patency of the radial and ulnar arteries.

D

Pain in the aging adult is considered to be: A) Part of the normal degenerative process B) Perceives to lesser degree C) An expected finding D) Unrelated to the aging process

D

Pain in the older adult is considered to be A) part of the normal degenerative process B) perceived to a lesser degree C) an expected finding D) unrelated to the aging process.

D

Pain signals are carried to the central nervous system by way of A) perception B) modulation C) referred pain D) afferent fibers

D

Pulsus bigeminus is associated with: A) aortic valve regurgitation. B) hyperkinetic states C) .heart failure. D) conduction disturbance

D

Spirituality is defined as A) a social group that claims to possess variable traits. B) participating in religious services on a regular basis. C) the process of being raised within a culture. D) a personal effort to find meaning and purpose in life.

D

The electrical stimulus of the cardiac cycle follows which sequence? A) AV node SA node bundle of His B) Bundle of His AV node SA node C) SA node AV node bundle of His bundle branches D) AV node SA node bundle of His bundle branches

D

The findings from an assessment of a 70-year-old patient with swelling in his ankles include jugular venous pulsations 5 cm above the sternal angle when the head of his bed is elevated 45 degrees. The nurse knows that this finding indicates: A) Decreased fluid volume. B) Increased cardiac output. C) Narrowing of jugular veins. D) Elevated pressure related to heart failure.

D

The first step to cultural competency by a nurse is to: A) identify the meaning of health to the patient B) understand how a health care delivery system works C) develop a frame of reference to traditional health care practices D) understand your own heritage and its basis in cultural values.

D

The inguinal nodes drain the: A) hand and lower arm. B) upper arm and breast. C) head and neck. D) anterior abdominal wall.

D

The most reliable indicator of pain in the adult is A) the degree of physical functioning B) nonverbal behaviors C) the MRI findings D) the patient's self report

D

The mother of a 2-year-old is concerned because her son has had three ear infections in the past year. What would be an appropriate response by the nurse? A) "It is unusual for a small child to have frequent ear infections unless something else is wrong." B) "We need to check the immune system of your son to determine why he is having so many ear infections." C) "Ear infections are not uncommon in infants and toddlers because they tend to have more cerumen in the external ear." D) "Your son's eustachian tube is shorter and wider than yours because of his age, which allows for infections to develop more easily."

D

The mother of a 2-year-old toddler is concerned about the upcoming placement of tympanostomy tubes in her son's ears. The nurse would include which of these statements in the teaching plan? A) The tubes are placed in the inner ear. B) The tubes are used in children with sensorineural loss. C) The tubes are permanently inserted during a surgical procedure. D) The purpose of the tubes is to decrease the pressure and allow for drainage.

D

The mother of a 3-month-old infant states that her baby has not been gaining weight. With further questioning, the nurse finds that the infant falls asleep after nursing and wakes up after a short time, hungry again. What other information would the nurse want to have? A) Infants sleeping position B) Sibling history of eating disorders C) Amount of background noise when eating D) Presence of dyspnea or diaphoresis when sucking

D

The nurse asks, I would like to ask you some questions about your health and your usual daily activities so that we can better plan your stay here. This question is found at the __________ phase of the interview process. A)Summary B) Closing C) Body D) Opening or introduction

D

The nurse documents that a patient has coarse, thickened skin and brown discoloration over the lower legs. Pulses are present. This finding is probably the result of: A) Lymphedema. B) Raynaud disease. C) Chronic arterial insufficiency. D) Chronic venous insufficiency.

D

The nurse educator is preparing an education module for the nursing staff on the dermis layer of skin. Which of these statements would be included in the module? The dermis: A) Contains mostly fat cells. B) Consists mostly of keratin. C) Is replaced every 4 weeks. D) Contains sensory receptors.

D

The nurse has completed the musculoskeletal examination of a patient's knee and has found a positive bulge sign. The nurse interprets this finding to indicate: A) irregular bony margins. B) soft tissue swelling in the joint. C) swelling from fluid in the epicondyle. D) swelling from fluid in the suprapatellar pouch.

D

The nurse has just completed an examination of a patients extraocular muscles. When documenting the findings, the nurse should document the assessment of which cranial nerves? A) II, III, and VI B) II, IV, and V C) III, IV, and V D) III, IV, and VI

D

The nurse hears bilateral loud, long, and low tones when percussing over the lungs of a 4-year-old child. The nurse should: A) Palpate over the area for increased pain and tenderness. B) Ask the child to take shallow breaths, and percuss over the area again. C) Immediately refer the child because of an increased amount of air in the lungs. D) Consider this finding as normal for a child this age, and proceed with the examination.

D

The nurse is assessing a 48 year old patient with a 50 pack year history during a routine physical examination. the patient is obese but exercises regularly and reports having pain in the claves that disappears at rest. Which findings requires further evaluation or follow up by the nurse? A) heart rate 54 beats per minute B) SpO2 94% on room air C) systolic blood pressure 135 mmHg D) ankle brachial index of 0.65

D

The nurse is assessing a patient for possible peptic ulcer disease. Which condition or history often causes this problem? A) Нуpertension B) Streptococcal infections C) Recurrent constipation with frequent laxative use D) Frequent use of nonsteroidal antiinflammatory drugs

D

The nurse is assessing a patient with a history of intravenous drug abuse. In assessing his mouth, the nurse notices a dark red confluent macule on the hard palate. This could be an early sign of what disease or disorder? A) Measles B) Leukemia C) A carcinoma D) Acquired immunodeficiency syndrome (AIDS)

D

The nurse is assessing a patient's eyes for the accommodation response and would expect to see which normal finding? A) Dilation of the pupils B) Consensual light reflex C) Conjugate movement of the eyes D) Convergence of the axes of the eyes

D

The nurse is assessing an older adult's functional ability. Which definition correctly describes one's functional ability? Functional ability: A) Is the measure of the expected changes of aging that one is experiencing. B) Refers to the individual's motivation to live independently. C) Refers to the level of cognition present in an older person. D) Refers to one's ability to perform activities necessary to live in modern society.

D

The nurse is assessing for clubbing of the fingernails and would expect to find: A) Nail bases that are firm and slightly tender. B) Curved nails with a convex profile and ridges across the nails. C) Nail bases that feel spongy with an angle of the nail base of 150 degrees. D) Nail bases with an angle of 180 degrees or greater and nail bases that feel spongy.

D

The nurse is assessing the forms of support an older patient has before she is discharged. Which of these examples is an informal source of support? A) The local senior center B) Her Medicare check C) Meals on Wheels meal delivery service D) Her neighbor, who visits with her daily

D

The nurse is assessing the lungs of an older adult. Which of these changes are normal in the respiratory system of the older adult? A) Severe dyspnea is experienced on exertion, resulting from changes in the lungs. B) Respiratory muscle strength increases to compensate for a decreased vital capacity C) Decrease in small airway closure occurs, leading to problems with atelectasis. D) Lungs are less elastic and distensible, which decreases their ability to collapse and recoil.

D

The nurse is aware that one change that may occur in the gastrointestinal system of an aging adult is: A) Increased salivation. B) Increased liver size. C) Increased esophageal emptying. D) Decreased gastric acid secretion.

D

The nurse is aware that the four areas in the body where lymph nodes are accessible are the: A) Head, breasts, groin, and abdomen B) Arms, breasts, inguinal area, and legs. C) Head and neck, arms, breasts, and axillae. D) Head and neck, arms, inguinal area, and axillae.

D

The nurse is bathing an 80-year-old man and notices that his skin is wrinkled, thin, lax, and dry. This finding would be related to which factor? A) Increased vascularity of the skin B) Increased numbers of sweat and sebaceous glands C) An increase in elastin and a decrease in subcutaneous fat D) An increased loss of elastin and a decrease in subcutaneous fat

D

The nurse is conducting a child safety class for new mothers. Which factor places young children at risk for ear infections? A) Family history B) Air conditioning C) Excessive cerumen D) Passive cigarette smoke

D

The nurse is describing a scaphoid abdomen. To the horizontal plane, a scaphoid contour of the abdomen depicts a _________profile A). Flat B) Convex C) Bulging D) Concave

D

The nurse is examining a 2-month-old infant and notices asymmetry of the infant's gluteal folds. The nurse should assess for other signs of what disorder? A) Fractured clavicle B) Down syndrome C) Spina bifida D) Hip dislocation

D

The nurse is examining a 2-year-old child and asks, May I listen to your heart now? Which critique of the nurses technique is most accurate? A) Asking questions enhances the childs autonomy B) Asking the child for permission helps develop a sense of trust C) This question is an appropriate statement because children at this age like to have choices D) Children at this age like to say, No. The examiner should not offer a choice when no choice is available

D

The nurse is examining a 3-month-old infant. While holding the thumbs on the infant's inner mid thighs and the fingers outside on the hips, touching the greater trochanter, the nurse adducts the legs until the nurse's thumbs touch and then abducts the legs until the infant's knees touch the table. The nurse does not notice any "clunking" sounds and is confident to record a: A) positive Allis test. B) negative Allis test. C) positive Ortolani's sign. D) negative Ortolani's sign.

D

The nurse is explaining to a patient that there are "shock absorbers" in his back to cushion the spine and to help it move. The nurse is referring to his: A) vertebral column. B) nucleus pulposus. C) vertebral foramen. D) intervertebral disks.

D

The nurse is giving report to the next shift and is using the SBAR framework for communication. Which of these statements reflects the Background portion of the report? A) "I'm worried that his gastrointestinal bleeding is getting worse." B) "We need an order for oxygen." C) "My name is Ms. Smith and I'm giving report on Mrs. X in room 1104." D) "He is four days postoperative and his incision is open to air."

D

The nurse is obtaining a health history on a 3-month-old infant. During the interview, themother states, "I think she is getting her first tooth because she has started drooling a lot."What is the best response by the nurse? A) "You're right, drooling is usually a sign of the first tooth." B) "It would be unusual for a 3-month-old to be getting her first tooth." C) "This could be the sign of a problem with the salivary glands." D) "She is just starting to salivate and hasn't learned to swallow the saliva."

D

The nurse is performing an assessment. Which of these findings would cause the greatest concern? A) A painful vesicle inside the cheek for 2 days B) The presence of moist, nontender Stensen's ducts C) Stippled gingival margins that snugly adhere to the teeth D) An ulceration on the side of the tongue with rolled edges

D

The nurse is performing an eye-screening clinic at a daycare center. When examining a 2-year-old child, the nurse suspects that the child has a "lazy eye" and should: A) Examine the external structures of the eye. B) Assess visual acuity with the Snellen eye chart. C) Assess the child's visual fields with the confrontation test. D) Test for strabismus by performing the corneal light reflex test.

D

The nurse is preparing to assess a patients abdomen by palpation. How should the nurse proceed? A) Palpation of reportedly tender areas are avoided because palpation in these areas may cause pain. B) Palpating a tender area is quickly performed to avoid any discomfort that the patient may experience. C) The assessment begins with deep palpation, while encouraging the patient to relax and to take deep breaths. D) The assessment begins with light palpation to detect surface characteristics and to accustom the patient to being touched.

D

The nurse is preparing to assess the ankle-brachial index (ABI) of a patient. Which statement about the ABI is true? A) Normal ABI indices are from 0.5 to 1.0. B) Normal ankle pressure is slightly lower than the brachial pressure. C) The ABI is a reliable measurement of peripheral vascular disease in individuals with diabetes. D) An ABI of 0.9 to 0.7 indicates the presence of peripheral vascular disease and mild claudication.

D

The nurse is preparing to assess the dorsalis pedis artery. Where is the correct location for palpation? A) Behind the knee B) Over the lateral malleolus C) In the groove behind the medial malleolus D) Lateral to the extensor tendon of the great toe

D

The nurse is preparing to perform a physical assessment. The correct action by the nurse is reflected by which statement? The nurse: A) Performs the examination from the left side of the bed. B) Examines tender or painful areas first to help relieve the patients anxiety. C) Follows the same examination sequence, regardless of the patients age or condition. D) Organizes the assessment to ensure that the patient does not change positions too often.

D

The nurse is preparing to use an otoscope for an examination. Which statement is true regarding the otoscope? The otoscope: A) Is often used to direct light onto the sinuses. B) Uses a short, broad speculum to help visualize the ear. C) Is used to examine the structures of the internal ear. D) Directs light into the ear canal and onto the tympanic membrane.

D

The nurse is providing patient education for a man who has been diagnosed with a rotator cuff injury. The nurse knows that a rotator cuff injury involves the: A) nucleus pulposus. B) articular process. C) medial epicondyle. D) glenohumeral joint.

D

The nurse is reviewing causes of increased intraocular pressure. Which of these factors determines intraocular pressure? A) Thickness or bulging of the lens B) Posterior chamber as it accommodates increased fluid C) Contraction of the ciliary body in response to the aqueous within the eye D) Amount of aqueous produced and resistance to its outflow at the angle of the anterior chamber

D

The nurse is reviewing the anatomy and physiologic functioning of the heart. Which statement best describes what is meant by atrial kick? A) The atria contract during systole and attempt to push against closed valves. B) Contraction of the atria at the beginning of diastole can be felt as a palpitation. C) Atrial kick is the pressure exerted against the atria as the ventricles contract during systole. D) The atria contract toward the end of diastole and push the remaining blood into the ventricles.

D

The nurse is reviewing the development of the newborn infant. Regarding the sinuses, which statement is true in relation to a newborn infant? A) Sphenoid sinuses are full size at birth. B) Maxillary sinuses reach full size after puberty. C) Frontal sinuses are fairly well developed at birth. D) Maxillary and ethmoid sinuses are the only sinuses present at birth.

D

The nurse is reviewing the principles of pain. Which type of pain is due to an abnormal processing of the pain impulse through the peripheral or central nervous system? A) Visceral B) Referred C) Cutaneous D) Neuropathic

D

The nurse is reviewing the structures of the ear. Which of these statements concerning the eustachian tube is true? A) The eustachian tube is responsible for the production of cerumen. B) It remains open except when swallowing or yawning. C) The eustachian tube allows passage of air between the middle and outer ear. D) It helps equalize air pressure on both sides of the tympanic membrane.

D

The nurse is taking the history of a patient who may have a perforated eardrum. What would be an important question in this situation? A) "Do you ever notice ringing or crackling in your ears?" B) "When was the last time you had your hearing checked?" C) "Have you ever been told that you have any type of hearing loss?" D) Is a normal finding, and no further follow-up is necessary.

D

The nurse is testing the function of cranial nerve XI. Which of these best describes the response the nurse should expect if the nerve is intact? The patient: A) Demonstrates the ability to hear normal conversation. B) Sticks out the tongue midline without tremors or deviation. C) Follows an object with his or her eyes without nystagmus or strabismus. D) Moves the head and shoulders against resistance with equal strength.

D

The nurse needs to complete an admission health history and physical assessment for a patient diagnosed with fever of unknown origin. The patient tells the nurse, " I don't feel good. I'm hot and feel sick to my stomach." What would be the best response by the nurse? A) it will not take too long. I will hurry B) we need the information to compete your admission paperwork C) i can come back in a few minutes D) let me see if I can get you something for your nausea and then we can talk later

D

The nurse notices that a patient has a solid, elevated, circumscribed lesion that is less than 1 cm in diameter. When documenting this finding, the nurse would report this as a: A) Bulla. B) Wheal. C) Nodule. D) Papule.

D

The nurse notices that a patients submental lymph nodes are enlarged. In an effort to identify the cause of the node enlargement, the nurse would assess the patients: A) Infraclavicular area. B) Supraclavicular area. C) Area distal to the enlarged node. D) Area proximal to the enlarged node.

D

The nurse notices that a school-aged child has bluish-white, red-based spots in her mouth that are elevated about 1 mm to 3 mm. What other signs would the nurse expect to find in this patient? A) Pink, papular rash on the face and neck B) Pruritic vesicles over her trunk and neck C) Hyperpigmentation on the chest, abdomen, and back of the arms D) Red-purple, maculopapular, blotchy rash behind the ears and on the face

D

The nurse notices that a woman in an exercise class is unable to jump rope. The nurse knows that to jump rope, one's shoulder has to be capable of: A) inversion. B) supination. C) protraction. D) circumduction.

D

The nurse suspects that a patient has a distended bladder. How should the nurse assess for this condition? A) Percuss and palpate in the lumbar region. B) Inspect and palpate in the epigastric region. C) Auscultate and percuss in the inguinal region. D) Percuss and palpate the midline area above the suprapubic bone.

D

The nurse understands that medication absorption is affected by the administration route. Which route for medications has the fastest absorption rate? A) Cream applied to the skin B) Enteric-coated capsules C) Subcutaneous injection D) Intravenous injection

D

The nurse uses health promotion activities with a new patient. What would this focus include? A) the nurse would try to change the patient's perception of disease. B) the nurse would search for identification of biomedical model interventions C) the nurse would help to identify negative health acts of the patient D) the nurse would empower the patient to choose a

D

The patient is ordered an ultrasound to determine the size, shape, and location of the kidneys. The nurse knows that prior to the test the patient will A) be required to have a bowel cleansing enema B) be checked for any allergies to shellfish. C) be required to drink a large amount of fluids before the test. D) have no pretest requirements.

D

The term culturally competent implies that the nurse A) is prepared in nursing B) possesses knowledge of the traditions of diverse peoples C) applies underlying knowledge to providing nursing care. D) understands the cultural context of the patient's situation.

D

What action should the nurse take immediately after instilling the prescribed eyedrops into the patient's eye? A) Apply a sterile eye patch to each eye receiving drops B) Maintain light pressure on the lower eyelid to keep it pulled down. C) Wipe the eyelid toward the inner canthus area. D) Press gently on the inner canthus area.

D

What is the source of deep somatic pain? A) pancreas B) intestine C) skin and subcutaneous tissue D) bones and joints

D

What is the tissue that connects the tongue to the floor of the mouth called? A) Uvula B) Palate C) Papillae D) Frenulum

D

What symptom is most likely to be exhibited by the patient who complains of voiding small amounts of urine in relation to his fluid intake? A) Nocturia B) Polyuria C) Anuria D) Oliguria

D

What type of database is most appropriate when rapid collection of data is required and often compiled concurrently with lifesaving measures? A) complete B) focused C) follow up D) emergency

D

When assessing a 75-year-old patient who has asthma, the nurse notes that he assumes a tripod position, leaning forward with arms braced on the chair. On the basis of this observation, the nurse should: A) Assume that the patient is eager and interested in participating in the interview. B) Evaluate the patient for abdominal pain, which may be exacerbated in the sitting position. C) Assume that the patient is having difficulty breathing and assist him to a supine position. D) Recognize that a tripod position is often used when a patient is having respiratory difficulties.

D

When assessing a patient's pulse, the nurse notes that the amplitude is weaker during inspiration and stronger during expiration. When the nurse measures the blood pressure, the reading decreases 20 mm Hg during inspiration and increases with expiration. This patient is experiencing pulses: A) Alternans. B) Bisferiens. C) Bigeminus. D) Paradoxus.

D

When assessing muscle strength, the nurse observes that a patient has complete range of motion against gravity with full resistance. What Grade should the nurse record using a 0 to 5 point scale? A) 2 B) 3 C) 4 D) 5

D

When auscultating over a patient's femoral arteries the nurse notices the presence of a bruit on the left side. The nurse knows that: A) Are often associated with venous disease. B) Occur in the presence of lymphadenopathy. C) In the femoral arteries are caused by hypermetabolic states. D) Occur with turbulent blood flow, indicating partial occlusion.

D

When examining a 16-year-old male teenager, the nurse should: A) Discuss health teaching with the parent because the teen is unlikely to be interested in promoting wellness. B) Ask his parent to stay in the room during the history and physical examination to answer any questions and to alleviate his anxiety. C) Talk to him the same manner as one would talk to a younger child because a teens level of understanding may not match his or her speech. D) Provide feedback that his body is developing normally, and discuss the wide variation among teenagers on the rate of growth and development.

D

When examining the nares of a 45-year-old patient who is experiencing rhinorrhea, itching of the nose and eyes, and sneezing, the nurse notices the following: pale turbinates, swelling of the turbinates, and clear rhinorrhea. Which of these conditions is most likely the cause? A) Nasal polyps B) Acute rhinitis C) Acute sinusitis D) Allergic rhinitis

D

When inspecting the anterior chest of an adult, the nurse should include which assessment? A) Diaphragmatic excursion B) Symmetric chest expansion C) Presence of breath sounds D) Shape and configuration of the chest wall

D

When measuring a patients weight, the nurse is aware of which of these guidelines? A) The patient is always weighed wearing only his or her undergarments. B) The type of scale does not matter, as long as the weights are similar from day to day. C) The patient may leave on his or her jacket and shoes as long as these are documented next to the weight. D) Attempts should be made to weigh the patient at approximately the same time of day, if a sequence of weights is necessary.

D

When observing a patients verbal and nonverbal communication, the nurse notices a discrepancy. Which statement is true regarding this situation? The nurse should: A) Ask someone who knows the patient well to help interpret this discrepancy. B) Focus on the patients verbal message, and try to ignore the nonverbal behaviors. C) Try to integrate the verbal and nonverbal messages and then interpret them as an average. D) Focus on the patients nonverbal behaviors, because these are often more reflective of a patients true feelings.

D

When performing an assessment of a patient, the nurse notices the presence of an enlarged right epitrochlear lymph node. What should the nurse do next? A) Assess the patient's abdomen, and notice any tenderness. B) Carefully assess the cervical lymph nodes, and check for any enlargement. C) Ask additional health history questions regarding any recent ear infections or sore throats. D) Examine the patient's lower arm and hand, and check for the presence of infection or lesions.

D

When reviewing the musculoskeletal system, the nurse recalls that hematopoiesis takes place in the: A) liver. B) spleen. C) kidneys. D) bone marrow.

D

When the nurse asks a 68-year-old patient to stand with feet together and arms at his side with his eyes closed, he starts to sway and moves his feet farther apart. The nurse would document this finding as a(n): A) Ataxia. B) Lack of coordination. C) Negative Homans sign. D) Positive Romberg sign.

D

When the nurse performs the confrontation test, the nurse has assessed: A) Extraocular eye muscles (EOMs). B) Pupils (pupils equal, round, reactive to light, and accommodation [PERRLA]). C) Near vision. D) Visual fields.

D

Which category is appropriate in a cultural assessment? A) family history B) chief complaint C) past medical history D) health-related beliefs.

D

Which is an example of objective data? A) patient's history of allergies B) patient's use of medications at home C) last menstrual period one month ago D) 2- x 5 cm scar present on the right forearm.

D

Which is considered a common physiologic change that occurs with pain? A) Polyuria B) Hyperventilation C) Hyperactive Bowel Sounds D) Tachycardia

D

Which is considered a common physiologic change that occurs with pain? A) polyuria B) hyperventilation C) hyperactive bowel sounds D) tachycardia

D

Which of the following statements regarding cultural/racial differences in the treatment of pain is true? A) Neurotransmitters are more concentrated in white individuals than in black and Hispanic individuals. B) White individuals receive more analgesic therapy than black or Hispanic individuals with similar symptoms. C) Pain modulation is more highly developed in black and Hispanic individuals. D) Black and Hispanic individuals have been found to have a higher pain tolerance than white individuals.

D

Which of the social determinants of health has the greatest influence on a person's health? A) work environment B) neighborhood C) education D) poverty

D

Which of these statements about the peripheral nervous system is correct? A) The CNs enter the brain through the spinal cord. B) Efferent fibers carry sensory input to the central nervous system through the spinal cord. C) The peripheral nerves are inside the central nervous system and carry impulses through their motor fibers. D) The peripheral nerves carry input to the central nervous system by afferent fibers and away by efferent fibers

D

Which of these veins are responsible for most of the venous return in the arm? A) Deep B) Ulnar C) Subclavian D) Superficial

D

Which statement is most appropriate to use when initiating an assessment of cultural beliefs with an older American Indian patient? A) "Are you of Christian faith?" B) "Do you want to see a medicine man? C) "How often do you seek help from medical providers?" D) "What cultural or spiritual beliefs are important to you?"

D

Which structure is located in the left lower quadrant of the abdomen? A) Liver B) Duodenum C) Gallbladder D) Sigmoid colon

D

Which would be included in the database for a new patient admission to a surgical unit? A) all subjective and objective data gathered by a health practitioner from a patient B) all objective data obtained from a patient through inspection, percussion, palpation, and auscultation C) A summary of a patient's record, including laboratory studies. D) All subjective and objective data, data gathered from a patient, and the results of any laboratory or diagnostic studies completed.

D

While caring for a preterm infant, you are aware that A) inhibitory neurotransmitters are in sufficient supply by 15 weeks' gestation B) the fetus has less capacity to feel pain C) repetitive blood draws have minimal long-term consequences D) the preterm infant is more sensitive to painful stimuli

D

While caring for a preterm infant, you are aware that: A) Inhibitory neurotransmitters are in sufficient supply y 15 weeks gestation. B) The fetus has less capacity to feel pain C) Repetitive blood draws have minimal long-term consequences. D) The preterm infant is more sensitive to painful stimuli.

D

While examining a 48-year-old patients eyes, the nurse notices that he had to move the handheld vision screener farther away from his face. The nurse would suspect: A) Myopia. B) Omniopia. C) Hyperopia. D) Presbyopia.

D

While measuring a patients blood pressure, the nurse recalls that certain factors, such as __________, help determine blood pressure. A) Pulse rate B) Pulse pressure C) Vascular output D) Peripheral vascular resistance

D

While obtaining a health history from the mother of a 1-year-old child, the nurse notices that the baby has had a bottle in his mouth the entire time. The mother states, "It makes a greatpacifier." What is the best response by the nurse? A) "You're right. Bottles make very good pacifiers." B) "Using a bottle as a pacifier is better for the teeth than thumb-sucking." C) "It's okay to use a bottle as long as it contains milk and not juice." D) "Prolonged use of a bottle can increase the risk for tooth decay and ear infections."

D

While the nurse is taking the history of a 68-year-old patient who sustained a head injury 3 days earlier, he tells the nurse that he is on a cruise ship and is 30 years old. The nurse knows that this finding is indicative of: A) Great sense of humor. B) Uncooperative behavior. C) Inability to understand questions. D) Decreased level of consciousness.

D

a patient is requesting pain medication and expresses a pain level of 9/10; however, the patient is up and smiling. How should you proceed? A) complete a full physical examination B) reposition patient but withhold pain medication based on behavior C) call the provider and suggest a substance abuse consult D) use therapeutic communication techniques to determine the patient's pain scale goal and history of chronic pain.

D

for older adult postoperative patients, poorly controlled acute pain places them at higher risk for: A) atelectasis B) increased myocardial oxygen demand C) impaired wound healing D) all of the above

D

he findings from an assessment of a 70-year-old patient with swelling in his ankles include jugular venous pulsations 5 cm above the sternal angle when the head of his bed is elevated 45 degrees. The nurse knows that this finding indicates: A) Decreased fluid volume. B) Increased cardiac output. C) Narrowing of jugular veins. D) Elevated pressure related to heart failure.

D

he physician comments that a patient has abdominal borborygmi. The nurse knows that this term refers to: A) Loud continual hum. B) Peritoneal friction rub. C) Hypoactive bowel sounds. D) Hyperactive bowel sounds

D

which of the following findings in an abdominal assessment should be reported to the physician immediately? A) continuous bowel sounds over the ileocecal valve B) irregular pattern of bowel sounds in the right lower quadrant C) absence of bowel sounds for 60 seconds D) rigid, board-like abdomen

D

Mr. Jankowski comes to your clinic with his child, who has been studying about the thymus gland in school. She is correct when she tells you the following is/are true about the thymus gland. (Select all that apply.) A) Develops T-lymphocytes B) The gland atrophies after puberty. C) Is vital to the immune system D) Develops B-lymphocytes

D, B, C

Which of the following changes may occur in the gastrointestinal system of an aging adult? Select all that apply A) increased salivation B) hepatomegaly C) dysphagia D) increased gastric acid secretion E) none of the above

Dysphasia

A 4-year-old boy is brought to the emergency department by his mother. She says he points to his stomach and says, It hurts so bad. Which pain assessment tool would be the best choice when assessing this childs pain?

FACES

True or False: Clubbing is a sign of increased capillary perfusion

False

True or False: Increased turgor is seen in patients experiencing dehydration

False

height decreases with aging because a thickening of the intervertebral disk develops True False

False

method of pain assessment

PQRST

Dark, tarry stools can indicate a gastrointestinal bleed True False

True

neuropathic pain

Type of pain caused from shingles, HIV/AIDS, scabies, chemotherapy, stroke, tumor

A patient is requesting pain medication and expresses a pain level of 9/10; however, the patient is up and smiling. How should you proceed?

Use therapeutic communication Technique to determine the patient's current prescription drug use.

spirituality

a broad term focused on a connection to something larger than oneself, and a belief in transcendence.

race

a means of self-identification, refers to a group of people who share similar physical characteristics.

ethnicity

a social group within the social system that claims to possess variable traits such as common geographic origin, migratory status, and religion.

_____________ pain is short and self-limiting, often follows a predictable trajectory, and dissipates after an injury heals (examples: surgery, trauma, kidney stones)

acute

pain can be classified by its duration as ____________ or _____________.

acute, chronic

hot cold theory of health

an explanatory model with origins in the ancient greek humoral theory.

religion

an organized system of beliefs concerning the cause, nature, and purpose of the universe, as well as attendance at regular services.

during modulation, pathways from the brainstem to the spinal cord release a set of neurotransmitters with an _________________ effect. (serotonin, norepinephrine, neurotensin, and endogenous opioids)

analgesic

the holistic health model

assesses the whole person because it views the mind, body, and spirit as interdependent and functioning as a whole within the environment.

ADOPIE stands for

assessment, diagnosis, outcome identification, planning, implementation, evaluation

biomedical theory of illness

assumes that all events in life have a cause and effect (example: germ theory)

________________ pain is a transient spike in pain level, moderate to severe in intensity, in an otherwise controlled pain syndrome.

breakthrough

culturally competent

caregivers understand all aspects of the patient's cultural being, and this include this information in the patient's care.

collaborative problems

certain physiologic conditions in which the approach to treatment involves multiple disciplines.

_____________ pain does not stop when the injury heals, it persists after the predicted trajectory, outlasts its protective purpose, and the level of pain does not correspond with physical findings.

chronic

______________ pain is diagnosed when the pain continues for 6 months or longer.

chronic (persistent)

assessment

collection of subjective and objective data about a patient's health

subjective data

consists of info provided by the affected individual

What are you looking for when completing an abdomen-inspect and palpate?

contour/symmetry guarding/organ size

_____________________ pain is derived from the skin surface and subcutaneous tissues.

cutaneous

______________ ________________ pain comes from sources such as the blood vessels, joints, tendons, muscles, and bone.

deep somatic

magicoreligious perspective of illness

depends on supernatural forces of good and evil.

first-level priority problems

emergent, life threatening, and immediate, such as establishing an airway or supporting breathing.

linguistic competence

ensures that non-native English speaking patients receive care in a manner in which they linguistically understand

follow up database

evaluates the status of any identified problem at regular intervals (short term and chronic)

true or false: dementia impacts the ability to feel pain

false

true or false: pain is a normal process of aging

false

The nurse is caring for a black child who has been diagnosed with marasmus. The nurse would expect to find the:

hair to be less kinky and to be a copper-red color.

current evidence suggests that repetitive and poorly controlled pain in infants can result in changes in the CNS that lead to pain _______________________ later in life.

hypersensitivity

culturally appropriate

implies that caregivers apply their knowledge of culture to provide the best patient care possible.

third-level priority problems

important to the patients health but can be addressed after more urgent problems. Examples include lack of knowledge or family coping.

a complete database

includes a complete health history and full physical examination, yielding the first diagnosis.

objective data

includes information obtained by the health care provider through observation and inspecting, percussing, palpating, and auscultating during the physical examination.

evidence based practice

is a systematic approach to practice that uses research evidence, clinical expertise, clinician knowledge, and the patient's preferences and values to make decisions about care and treatment.

naturalistic theory of illness

is the belief that the forces of nature must be kept in natural balance or harmony (example: ying yang, hot cold)

critical thinking

is the multidimensional thinking process needed for sound diagnostic reasoning and clinical judgment.

diagnostic reasoning

is the process of analyzing health data and drawing conclusions to identify diagnoses. (consists of attending to cues, formulating hypothesis, gathering data, and diagnosis)

database

is the totality of information available about the patient, including subjective data, objective data, and the patient's medical record and laboratory studies.

if chronic pain is cancer related, it is called ________________ pain, and is induced by tissue necrosis or stretching of an organ by the growing tumor.

malignant

culturally sensitive

means that caregivers are aware of different cultures in the health care setting.

the last phase of nociception is ______________, in which build in mechanisms eventually slow down and stop the processing of a painful stimulus.

modulation

__________________ pain does not adhere to typical and predictable phases.

neuropathic

__________________ pain is due to a lesion or disease in the somatosensory system, and implies an abnormal processing of the pain message from an injury to the nerve fibers.

neuropathic

________________ pain develops when functioning and intact nerve fibers in the periphery and CNS are stimulated.

nociceptive

normal ________________ pain is protective, predictable, and time limited based on the extent of an injury.

nociceptive

specialized nerve endings called ___________ are designed to detect painful sensations from the periphery and transmit them to the CNS.

nociceptors

The lymph nodes located at the base of the skull are the ______ lymph nodes

occipital

cutaneous pain

pain is derived from the skin surface and subcutaneous tissues.

referred pain

pain is felt at one site but originates from another location.

the third phase of nociception is ______________, which signifies the conscious awareness of a painful sensation

perception

pain that is felt at a particular site but originates from another location is known as ___________________ pain.

referred

second-level priority problems

require prompt intervention to prevent further deterioration and may include mental status change, acute pain, or abnormal laboratory values.

__________________ pain originates from musculoskeletal tissues or the body surface.

somatic

is pain a subjective or objective experience?

subjective

The nurse is assessing a patients pain. The nurse knows that the most reliable indicator of pain would be the:

subjective report

ADOPIE

the nursing process includes six phases which can be remembered using the acronym

acculturation

the process of adopting the culture and behavior of the majority culture.

socialization

the process of being raised within a culture and acquiring the characteristics of that group

diagnosis

the purpose of assessment is to make a judgement

yin yang

theory of health, health exists when all aspects of the person are in perfect balance.

the first phase of nociception is _______________, in which a noxious stimulus in the form of a traumatic or chemical injury, burn, incision, or tumor takes place in the periphery

transduction

nociception can be divided into four phases:

transduction, transmission, perception, modulation

the second phase of nociception is _______________, in which the pain impulses moves from the level of the spinal cord to the brain.

transmission

true or false: infants have the same capacity for pain as adults.

true

focused database

used for a limited or short term problem. It is smaller in scope and more targeted than the complete database.

Because hair for humans is no longer needed for protection from cold or trauma, it is called:

vestigial

______________ pain originates from the larger internal organs (stomach, gallbladder, pancreas) and is often described as dull, deep, squeezing, or cramping.

visceral

_______________ pain is transmitted along the fibers of the autonomic nervous system, so it often presents with responses such a vomiting, nausea, pallor, and diaphoresis.

visceral

When assessing the quality of a patients pain, the nurse should ask which question?

what does your pain feel like?


Ensembles d'études connexes

stock market and investing test review!! (test on 3/8)

View Set

States and Capitals: Lesson 24, California, Oregon, Washington

View Set

Taylors Fundamentals PrepU Ch. 40 Oxygenation and Perfusion

View Set

THE STOCK MARKET: HOW IT STARTED

View Set

Principles of Management (MGT201)

View Set

Supply chain management Final Spring 2019

View Set

ch 10 quiz real estate principles

View Set